الواقع و الزمن 2023

 المحتويات

تشكل الثقب الدودى من انصهار ثقبين ابيضين ثم تهافته السريع الى ثقبين اسودين 
اسئلة و اجوبة حول خياراتنا فى الفيزياء النظرية
تجربة ستارن-غارلاش
الميكانيك الكمومى المصفوفى و الميكانيك الكمومى على الشبكة
الميكانيك الكمومى المصفوفى
ماذا يمكن ان يقوم به النظريون الشبكيون لنظرية الاوتار الممتازة و النظرية المصفوفية؟
تكامل الطريق فايمان, تدوير وييك, الشبكة و الميكانيك الكمومى المصفوفى
الميكانيك الكمومى على الشبكة 2
الميكانيك الكمومى المصفوفى هو ميكانيك اغشية او ميكانيك جسيمات
الثنائية الثقالية-المعيارية
كورس فى الثنائية الثقالية-المعيارية
العامل الاجتماعى فى البحث العلمى
الاعداد الغراسمانية
مستويات النجاح
طرق مونتى كارلو الهجينة
الفيزياء النظرية فى اطار الفيزياء الحاسوبية
محاضرات فى الميكانيك الكمومى
الهزاز التوافقى على الشبكة
الميكانيك الكمومى على الشبكة و الثقالة الكمومية المصفوفية
مبدأ التكافؤ و تساوى الكتلتين الثقالية و العطالية و الفكرة الاسعد فى حياة اينشتاين.
كورس الميكانيك الاحصائى 2023
تناقض جيبس
الثقوب السوداء هى ترموديناميك
تبخر الثقب الاسود و اشعاع هاوكينج
مدخل الى النسبية العامة من الميكانيك الكلاسيكى
الهزاز التوافقى المصفوفى
اقرب الى الحقيقة
حل معضلة ضياع المعلومات فى الثقب: خلاصة
الايونات المحبوسة
حل بولشينسكى لمعضلة ضياع المعلومات فى الثقب
العقل و السفر فى الزمن
كورس الفيزياء الذرية 2023
من الفيزياء الذرية الى الحاسوبية الكمومية
نموذج الابعاد الاضافية الكبيرة
الغرافيتون الثقيل كحل للمادة المظلمة
معضلة الهايراركى او التراتب فى فيزياء الجسيمات الاولية
ثلاثة كتب فى نظرية الحقل الكمومى على الشبكة
التضميم
الاشخاص الذين تحصلوا على نوبل مرتين
تدوير وييك
الحاسوبية الكمومية و محاضرات فايمان حول الحاسوبية
الحاسوبية الكمومية
معضلة المضاعفة الفرميونية
سبينورات ديراك, ماجورانا و وايل
تعجيز ديراك لاولر
مبرهنات التناظر الممتاز
هندسة الفيزياء
طريقة مونتى كارلو تصيب-أو-تخيب
جداول التكاملات
ديراك و فون نيومان
 


تشكل الثقب الدودى من انصهار ثقبين ابيضين ثم تهافته السريع الى ثقبين اسودين

 فى الفيلم تشكل ثقب-دودى wormhole من انصهار ثقبين ابيضين (مفرد ثقب ابيض white hole).

ثم تهافت الثقب-الدودى بعد ذلك و بسرعة الى ثقبين اسودين (مفرد ثقب اسود black hole).
الدوائر الحمراء و الوردية هى افق حدث event horizon و السهم الاصفر هو اتجاه عبور افق الحدث.
بالنسبة للثقب الاسود افق الحدث هى الدوائر الوردية و اتجاه عبور افق الحدث هو نحو الداخل لان الثقب الاسود هو كائن يبتلع كل شيء دون ان يشبع.
اما اتجاه عبور افق الحدث (الدوائر الحمراء) بالنسبة للثقب الابيض هو نحو الخارج لان الثقب الابيض هو كائن يقيء كل شيء دون ان يستريح.
الثقب الابيض هو العكس فى الزمن time reversal للثقب الابيض.
هذا الثقب-الدودى هو اشهر ثقب-دودى على الاطلاق و اولها اكتشافا (الثقب-الدودى لشوارشيلد Schwarzschild wormhole) الذى اكتشفه اينشتاين و روزن و لهذا فهو يسمى ايضا جسر اينشتاين-و-روزن Einstein-Rosen bridge لانه يربط بين عالمين كما سنبين.
هناك فى الحقيقة شخص آخر غير معروف اكتشف هذا الثقب-الدودى قبل اينشتاين و روزن ب 20 سنة يسمى لودويغ فلام Ludwig Flamm لكن لم يأخذ حظه من الشهرة الا فى الدوائر الضيقة لتاريخ الفيزياء. اذن كل شخص و حظه فى العلم كما فى الدنيا.
هذا الثقب-الدودى هو حل مضبوط لمعادلات اينشتاين للنسبية العامة و الا لما كنا تكلمنا عنه اصلا.
مخطط بنروز Penrose diagram لهذا الثقب-الدودى موجود فى المربع الى يمين الصورة. مخطط بنروز هو المخطط الذى يختزل جميع الفضاء-زمن فى مربع و هو احد انجازات بنروز استاذ هاوكينغ و الذى تحصل على نوبل العام الماضى.
الخطوط الخضراء المقوسة فى مخطط بنروز هى المفردة singularity. هناك مفردة للثقب الابيض (اسفل) و هناك مفردة للثقب الاسود (اعلى).
الخطوط المستقيمة الوردية فى مخطط بنروز هى افق حدث الثقب الاسود و الخطوط المستقيمة الحمراء هى افق حدث الثقب الابيض.
لاحظوا الخط الابيض المتحرك الذى هو مفردة الثقب-الدودى كيف كانت منطبقة على مفردة الثقب الابيض فى بداية التفاعل ثم تطورت حتى تصبح منطبقة على مفردة الثقب الاسود فى نهاية التفاعل.
هذا الثقب-الدودى غير-قابل-للعبور non-traversable لانه كما يبينه التفاعل هو غير مستقر و يتهافت بسرعة شديدة الى ثقبين اسودين.
لكنه من الناحية المبدأية يمكن ان يربط العالم الذى نعيش فيه (الربع فى مخطط بنروز الموجود على اليمين) مع عالم آخر مكافئ لعالمنا لكن منقطع سببيا بالكامل عن عالمنا (الربع فى مخطط بنروز الموجود على اليسار).
هذان العالمان لا يمكن ان يتواصلا الا عبر الثقب-الدودى و من هنا تكمن اهمية جعل هذه الثقوب-الدودية مستقرة قابلة-للعبور traversable.
مرجع الفيلم: اندرو هاميلتون Andrew Hamilton.





تجربة ستارن-غارلاش

تجربة ستارن-غارلاش Stern-Gerlach experiment هى واحدة من اعظم تجارب الفيزياء الكمومية.

هى التجربة التى حسمت المعركة نهائيا لمصلحة الميكانيك الكمومى على حساب الميكانيك الكلاسيكى و لم يبقى ادنى شك فى صحة نظرية الميكانيك الكمومى.
حتى ان باولى قال بخصوص ستارن بعد خروج نتيجة التجربة (لعله اقتنع الآن).
بالفعل فان اكبر المتشككين فى الميكانيك الكمومى كان اوطو ستارن Otto Stern نفسه الذى فكر و خطط و نفذ هذه التجربة بعبقرية و نجاح باهرين.
بالخصوص فان ستارن كان متشككا كثيرا ببوهر Bohr و خاصة فى شرط تكميم العزم الحركى angular momentum quantization condition الذى شرح به بوهر طيف spectrum ذرة الهيدروجين فى نموذجه الشهير باسم نموذج بوهر Bohr model و الذى يسمى ايضا النظرية الكمومية القديمة old quantum theory و الذى سبق نظرية الميكانيك الكمومى التى نعرفها اليوم باكثر من 10 سنوات.
هذا الشرح او التفسير الذى قدمه بوهر للتجربة هو فعلا خاطئ كما نعلم اليوم لكن بوهر فى المحصلة لم يكن حدسه خاطئ فهو تنبأ بنتيجة التجربة اعتماد على النظرية القديمة للميكانيك الكمومى او نموذج بوهر.
ستارن اصر على ان بوهر خاطئ و راهن على ترك الفيزياء اذا كان شرط تكميم العزم الحركى (الذى كان يسمى آنئذ شرط التكميم الفضائى space quantization condition) خاطئا.
وكل من راهن ضد بوهر خسر.
حتى اينشتاين راهن ضد بوهر فيما بعد وخسر رهانه. لكن ستارن لم يكن يعلم بعد بصعوبة تخطئة بوهر (حتى لو كان خاطئا).
اذن ستارن راهن على ان حزمة beam مشكلة من ذرات الفضة اذا مرت عبر حقل مغناطيسى غير منتظم non-uniform فهى لن تنشطر. لان كل الزوايا مسموح بها.
وراهن بوهر على انها يجب ان تنشطر و يجب ان تنشطر الى قسمين لان الزوايا المسموح بها متقطعة discrete و فى هذه الحالة فهى تأخذ قيمتين ممكنتين فقط.
ستارن راهن على عدم الانقسام لانه كان مازال كلاسيكيا الى حد بعيد لا يصدق بعد بمعجزات الميكانيك الكمومى.
وبوهر راهن على الانقسام لانه هو نبى الكمومى.
حزمة ذرات الفضة فعلا انشطرت الى شطرين. هذا هو الذى نراه فى التجربة واول من رأى ذلك كان غارلاش فى فيفرى من عام 1922.
وهذا لم يفاجئ بقية الفيزيائيين لان الجميع كان يشاطر بوهر فى قناعاته.
لكن الذى فاجئ الجميع انها انشطرت الى قسمين بالضبط كما قال بوهر. رغم ان تفسير بوهر للامر بانه راجع الى تكميم العزم الحركى بقيمتين كان خاطئا و هذا كان يفهمه العباقرة الكبار مثل ديراك و باولى و هازينبرغ بشكل او بآخر.
وكل الذرات الاخرى المُعَدة فى الحالة الاساسية ground state سوف تنشطر الى شطرين ايضا.
بوهر كان يظن ان الحالة الاساسية تتميز بعزم حركى يساوى واحد. اذن التكميم يعطى الحزمتين +1 او -1 فى اى اتجاه من الفضاء.
هذا خاطئ. العزم الحركى للحالة الاساسية صفر فلماذا اذن يقع الانشطار.
وحتى لو كان العزم الحركى هو واحد فانه كان يجب ان يتم الانشطار الى ثلاثة حزمات +1 و -1 و 0.
هذه هى اكبر نتيجة فى هذه التجربة على الاطلاق: انشطار الحزمة الى شطرين.
ستارن و غارلاش نتيجتهما دقيقة جدا لا غبار عليها.
بوهر صحيح فى ان الانشطار هو الى قسمين.
لكن ليس للسبب الذى كان يظنه.
اذن ستارن لم يخسر الرهان لكن لم يربحه ايضا.
وبوهر يصعب تخطيئه كما ذكرنا و لو كان مخطئا.
فحدسه الكمومى عميق جدا لم يستطع اختراقه حتى اينشتاين وتذكروا فان مدرسة كوبنهاغن تسمى كذلك لان بوهر من كوبنهاغن.
اذن حدس بوهر بخصوص هذه التجربة كان صحيحا الى حد كبير رغم خطأ ما يسمى نموذج بوهر.
للملاحظة فان ستارن (وهو يهودى-المانى) تحصل على نوبل عام 1943 عندما اصبح استاذا فى امريكا لكن لم يتحصل معه غارلاش على الجائزة لانه كان رئيس البرنامج النووى الالمانى النازى رعم انه من ابعد ما يكون عن النازية.
بكل بساطة غارلاش كان استاذ و عالم و المانى لا تهمه القضايا السياسية حتى لو كانت حرب عالمية.
اما تفسير التجربة فقد جاء به طالبا دكتوراة اصبحا فيما بعد فيزيائيين كبيرين و التفسير هو ما نسميه اليوم عزم-اللف الذاتى او السبين spin.
ذرات الفضة فى الحالة الاساسية تتميز بالكترون منفرد فى المستوى الطاقوى الاعلى highest energy level لان بقية الالكترونات الداخلية تأتى كلها فى ازواج.
هذا الالكترون الوحيد يتميز بعزم لف يساوى نصف وهذا يؤدى الى ثنائى قطب مغناطيسى او دايبول مغناطيسى magnetic dipole moment مكمم quantized بقيمتين و لهذا فان تفاعل هذا الدايبول المغتاطيسى (الذى يمكنكم تصور ه كمغناطيس) مع الحقل المغناطيسى يؤدى الى انشطار الحزمة الى حزمتين.
سبين الالكترون ليس له تفسير فى اطار الميكانيك الكلاسيكى يقينا فهو ليس له اى تفسير حركى (وتصورنا انه راجع الى الدوران الذاتى للجسيم حول نفسه هو خاطئى لان الالكترون هو جسيم نقطى اذن هو لا يستطيع الدوران حول نفسه لانه ليس لديه محور اصلا).
بل ان سبين الالكترون ليس له اى تفسير حقيقى فى اطار الميكانيك الكمومى بل نحتاج الى الميكانيك الكمومى النسبى وهذا ما فعله ديراك عندما اكتشف معادلته الشهيرة (معادلة ديراك).
السبين او عزم اللف يمكن تفسيره بالكامل باستخدام نموذج الكيوبت qubit model او البت الكمومى.
بل ان فكرة الكيوبت نفسها لا تجد لها تفسير فيزيائى حقيقى الا السبين الالكترونى او العكس فان فكرة السبين الالكترونى لا تجد لها اى تفسير فيزيائى حقيقى الا فكرة الكيوبت.
فى هذا المنشور اشرح كل هذه الامور و اعطى ثلاثة تمرينات لتدعيم الفهم لم يسعنى الوقت لعرض حلهما.

الميكانيك الكمومى المصفوفى و الميكانيك الكمومى على الشبكة

من الصعب جدا ايجاد منفذ الى البحث فى الفيزياء النظرية و بالضبط فانه يصعب جدا ايجاد منفذ سريع او اكيد الى البحث.

اقترح هنا موضوع (الميكانيك الكمومى المصفوفى matrix quantum mechanics) و موضوع (الميكانيك الكمومى على الشبكة lattice quantum mechanics) وهما توجهين غير مرتبطين بالضرورة لكن حسب ما اقترحه هنا فان هناك ارتباط وثيق جدا بينهما.
هذا مجال جديد وحتى المصطلح فانه غير معروف جدا.
لكن اهم و اول (بل هو اشهر من نار على علم) نموذج ميكانيك كمومى مصفوفى هو نموذج ال BFSS المعروف ايضا باسم نموذج النظرية-المصفوفية او M-(atrix) theory الذى اقترحه ساسكيند Susskind و زملائه فى اواخر التسعينات كتعريف لما يسمى (النظرية المصفوفية Matrix theory او اختصارا M-theory) التى تعيش فى 11 بعد و الموحدة للنظريات الوترية الخمسة التى تعيش فى عشرة ابعاد.
النظرية-المصفوفية M-(atrix) theory تلعب فى الطاقات العليا الدور الذى تلعبه فى الطاقات الدنيا نظرية الثقالة الممتازة فى 11 بعد بالنسبة للنظرية المصفوفية المجهولة M-theory و التى هى رسميا (نظرية الثقالة الكمومية).
هذا النموذج هو نموذج مصفوفى يانغ-و-ميلز Yang-Mills matrix model وهو يهتم بالثقالة الكمومية quantum gravity و براينات ديرشليه Dirichlet branes و الثنائية الثقالية-المعيارية gauge-gravity duality.
نموذج النظرية-المصفوفية موجود فى ابعاد 10 و 6 و 4 و 3 و 2. وهذه النماذج هى نماذج ميكانيك كمومى مصفوفى يانغ-و-ميلز وهذا هو سر قوتها الاكبر بالاضافة الى اهم ما يميزها و هو التناظر الممتاز supersymmetry. هذه النماذج من الميكانيك الكمومى المصفوفى الاهم موجودة فى الجدول مع تحديد التناظر الدورانى و التناظر الممتاز فى كل حالة.
واما الميكانيك الكمومى على الشبكة فأهميته برزت عندما تم و لأول مرة وضع نموذج ميكانيك كمومى متناظر-بامتياز على الشبكة lattice وضعه بنجاح كاترال Catteral و زملائه على الشبكة فى اواخر التسعينات.
هذا النموذج هو نموذج واس-و-زومينو Wess-Zumino model الشهير. وهو كما ذكرت اول نموذج تم وضعه على الشبكات مع الحفاظ على قدر كبير من التناظر الممتاز عبر ما يسمى اليوم التناظر الممتاز الملتوى twisted supersymmetry. وهو ليس نموذج مصفوفى لكن كيف نضع التناظر الممتاز على الشبكة كان فى ذلك الوقت هو الهدف.
عندما نرجع الى نموذج يانغ-و-ميلز فاننا نجده يتميز بالخاصيتين مصفوفى و تناظرى ممتاز و الخاصية الثانية اصعب على الشبكة دائما.
هذا كان و مازل انجاز كبير لان التناظر الممتاز هو تناظر مفصلى من الناحية الفيزيائية و هو ايضا من اصعب التناظرات من ناحية التقريب الحاسوبى و الحسابى.
و اذكر هنا اننى قد كنت هناك فى تلك الفترة فى جامعة سيراكيوس Syracuse University عندما بدأ كاترال هذا المشروع و حقق خطواته الاولى فيه.
اذن كنت هناك لكننى لم افكر فى العمل مع كاترال رغم توفر الامكانية امامى و رغم اننى ذهبت اختياريا و اخذت معه كورس بعنوان العلوم و الحاسوب computer and science و كان هذا اول تعرض لى شخصيا للفيزياء الحاسوبية computational physics.
اذن كدت افهم الموضوع فى ذلك الوقت لكن لم افهم الموضوع تماما. وقد اتضح فيما بعد ان هذا هو الموضوع الذى اريد العمل عليه.
و كما قلت فان الاختيار الاول الذى يختاره الطالب فى البداية سوف يحدد نجاحه الساحق من فشله الساحق.
اليوم كاترال هو الرائد الاول فى العالم فى دراسة النظريات المتناظرة بامتياز -بجميع انواعها- على الشبكات.
واهم النظريات المتناظرة بامتياز هى نماذج الميكانيك المصفوفى ليانغ-و-ميلز. وهو احد اكبر دارسى هذا النوع ايضا.
اذن الميكانيك الكمومى المصفوفى ليانغ-و-ميلز (و ايضا الميكانيك الكمومى لواس-و-زومينو) بل الميكانيك الكمومى المصفوفى بصفة عامة هو شيء محورى فى الثقالة الكمومية و نظريات الحقول المعيارية وهو مجال خصب جدا و اهم شيء يميز هذا الميكانيك الكمومى هو التناظر الممتاز.
و التناظر الممتاز لهذه النماذج مع كونها مصفوفية هو المدخل الى الثقالة الكمومية الوترية-المصفوفية و من اهم وسائل دراستها هى الميكانيك الكمومى على الشبكة.
الميكانيك الكمومى على الشبكة هو حالة خاصة من نظرية الحقول على الشبكة lattice field theory و على هذا فهو اسهل بكثير رغم اهميته الاكبر بالنسبة للثقالة الكمومية.
و كمدخل لدراسة الميكانيك الكمومى على الشبكة انظر هذا المقال لسايمون كاترال فى كيفية وضع التناظر الممتاز على الشبكة بشكل مضبوط.



الميكانيك الكمومى المصفوفى

 فى الصورة الاولى نكتب الميكانيك الكمومى المصفوفى BFSS وهو ميكانيك كمومى درجات حريته degrees of freedom هى 9 مصفوفات هرميتية Hermitian matrices هى الاحداثيات غير-التبديلية non-commutative coordinates للجسيمات الممتازة super-particles المعروفة باسم براينات (مفرد براين) D0.

و البراينات branes هى جسيمات تحقق الشروط الحدية لدريشليه Dirichlet boundary conditions و من هنا اتى الحرف D (فهى ليست جسيمات عادية).
هذا الميكانيك الكمومى المصفوفى يحتوى ايضا على حقل معيارى gauge field متضمن فى المشتقة الكوفارينتية covariant derivative فى الزمن D_t. هذا الميكانيك الكمومى هو اذن نظرية معيارية gauge theory.
اهم من هذا فان هذا الميكانيك الكمومى المصفوفى يمتاز بتناظر ممتاز supersymmetry كبير جدا يعطى ب 16 شحنة ممتازة supercharges وهو اقصى تناظر ممتاز ممكن و هو التناظر الممتاز الذى يميز فضاء-زمن ب 11 بعد.
هذا احد الاسباب التى تجعلنا نفهم هذا الميكانيك الكمومى المصفوفى على انه يصف النظرية M فى 11 بعد وهى النظرية الموحدة للنظريات الوترية الممتازة الخمسة التى تعيش فى 10 ابعاد.
فى الصورة الاولى لم نكتب القسم الفرميونى fermionic part للفعل action و هو قسم ضرورى من اجل ان يكون لدينا تناظر ممتاز.
فى الصورة الثانية نعطى مترية البراين الاسود black brane الثنوى dual لهذا الميكانيك الكمومى المصفوفى.
البراين الاسود هو ثقب اسود متشكل من ذرات هى عبارة عن الجسيمات الممتازة التى يصفها هذا الميكانيك الكمومى المصفوفى اى تلك البراينات D0.
هذه هى الثنائية الثقالية-المعيارية gauge-gravity duality التى يوفرها هذا الميكانيك الكمومى المصفوفى.
اذن يمكننا حساب خواص هذا الثقب الاسود (او بالاحرى هذا البراين الاسود) انطلاقا من نظرية معيارية متناظرة بامتياز هى هذا الميكانيك الكمومى المصفوفى.
وهذا الحساب هو حساب غير-اضطرابى non-perturbative بالاساس يعتمد على محاكاة هذا الميكانيك الكمومى المصفوفى باستخدام طرق مونتى كارلو Monte Carlo methods.
اذن يمكننا حساب تبخر الثقب الاسود black hole evaporation و التأكد من ضياع المعلومات information loss فى الثقب او عدم ضياعها و هذا انطلاقا من نظرية حقلية هى هذا الميكانيك الكمومى المصفوفى.
هذا هو المقصود من الثنائية الثقالية-المعيارية. نستخدم النظرية المعيارية بصورة غير-اضطرابية لحساب النظرية الثقالية (وهذا باستخدام طرق مونتى كارلو او اى طريقة اخرى غير-اضطرابية).
و قد نستخدم النظرية الثقالية لحساب النظرية المعيارية (وهذا مثلا يستخدم فى اجراء حسابات فى الكروموديناميك الكمومى quantum chromodynamics انطلاقا من النظرية الثقالية الثنوية له).
لكن يبقى الاتجاه الاول هو الاكثر اهمية لان النظرية الثقالية الكمومية هى أكبر مجهول فى الفيزياء النظرية.




ماذا يمكن ان يقوم به النظريون الشبكيون لنظرية الاوتار الممتازة و النظرية المصفوفية?


هذا مدخل ممتاز و فى المتناول جدا لفكرة الميكانيك الكمومى المصفوفى matrix quantum mechanics و الميكانيك الكمومى على الشبكة lattice quantum mechanics التى اقترحتهما البارحة للبحث فى مستوى الدكتوراة.
المقترح الذى يتناوله هنا ماسانورى هانادا Masanori Hanada هو اعم من اقتراحى لانه يقترح دراسة مواضيع تخرج من اطار الميكانيك الكمومى المصفوفى الى الاطار الاعم اطار نظرية التقابل الثقالى-المعيارى gauge-gravity duality و بالتالى فان وسيلة الدراسة تخرج من اطار الميكانيك الكمومى على الشبكة الى الاطار الاعم نظرية الحقل على الشبكة lattice field theory.
النظريات الاربعة الاساسية للتقابل الثقالى-المعيارى التى يناقشها المؤلف هى:
-التقابل AdS5/CFT4 وهى نظرية كونفورمالية conformal theory تعيش فى اربعة ابعاد وهى اول نظريات ال AdS/CFT اكتشافا.
-نظرية ABJM وهى تعيش فى ثلاثة ابعاد وتصف الفضاء-زمن حول البراين-الثنائى الاسود black two-brane مثلما ان الحالة الاولى تصف الفضاء-زمن حول البراين-الثلاثى الاسود black three-brane.
-التحول الطورى phase transition من الثقب الاسود black hole الى الوتر الاسود black string وهى تعيش فى بعدين. و الوتر الاسود هنا هو كينونة مختلفة عن البراين-الاحادى الاسود black one-brane.
-نموذج BFSS او النظرية-المصفوفية M-(atrix) theory و هذا هو الميكانيك الكمومى المصفوفى الذى يعيش فى بعد واحد.


تكامل الطريق فايمان, تدوير وييك, الشبكة و الميكانيك الكمومى المصفوفى

 الجميع يعرف ان الميكانيك الكمومى ينطلق من معادلة شرودينغر.

لكن بالنسبة للمسائل الاكثر أساسية (الثقالة الكمومية quantum gravity, الحقول المعيارية gauge fields, الثقوب السوداء black holes, كوسمولوجيا الانفجار الاعظم big bang cosmology, التحولات الطورية phase transition للمادة و الفضاء-زمن و ما شابه ذلك) فان نقطة انطلاق الميكانيك الكمومى هى ليست معادلة شرودينغر بل نقطة الانطلاق هى تكامل الطريق فايمان Feynman path integral الذى اكتشفه فايمان فى رسالته للدكتوراة فى الاربعينات.
تكامل الطريق فايمان مكافئ تماما لمعادلة شرودينغر وهما مكافئان تماما لمعادلة هايزنبرغ.
لكن تكامل الطريق فايمان يعطينى المنتشر propagator بين نقطتين فى الفضاء-زمن بدلالة جميع الطرق الرابطة بين النقطتين وهذا هو مبدأ التراكب الخطى superposition principle فى ابهى صوره والذى يقوم عليه جميع بناء الميكانيك الكمومى.
الجسيم الكمومى بين نقطتين سوف يمر عبر جميع الطرق الرابطة بين النقطتين فى نفس الوقت (لا محالة) وهذا عكس كل الوهم الكلاسيكى المترسخ فى الانسان.
فى هذه المحاضرة القصيرة نبرهن على تكامل الطريق فايمان. هذه هى الخطوة الاولى.
فى الخطوة الثانية نقوم بتدوير وييك Wick rotation للزمن من اجل ادخال درجة الحرارة. سنرى ان الطرق التى يسلكها الجسيم اصبحت مغلقة و ان مقلوب درجة الحرارة inverse temperature هو بالضبط دور الزمن الاقليدى period of Euclidean time ويتحول بذلك تكامل طريق فايمان الى دالة تقسيم بولتزمان Boltzmann partition function اى يتحول الميكانيك الكمومى الى ميكانيك احصائى.
اذن الميكانيك الكمومى الاقليدى (المدور) هو فى الحقيقة ميكانيك كمومى حرارى اى ان درجة الحرارة لا تنعدم فيه.
رغم اننا قمنا بتدوير الزمن نحو متغير اقليدى الا ان الميكانيك الكمومى مازال غير معرف رياضيا بالشكل الكافى.
فى الخطوة الثالثة نقوم بوضع الميكانيك الكمومى على الشبكة و بهذا نحصل على ميكانيك كمومى على الشبكة lattice quantum mechanics و هو كما سترون ليس الا نوع من انواع نموذج ايزينغ Ising model و بالتالى فهو معرف رياضيا بالكامل.
نؤكد هنا اننا لم نحدد بعد طبيعة التفاعل فالكمون potential مازال كيفى الى اقصى حدود لكن الميكانيك الكمومى على الشبكة يتصرف بالكامل مثل جمل السبين او جمل عزوم-اللف spin system وهذا معرف فيزيائيا و رياضيا بالكامل يمكننا ان نطبق عليه مباشرة طرق مونتى كارلو Monte Carlo methods.
فى الخطوة الرابعة نطبق خوارزمية ميتروبوليس Metropolis algorithm على هذا الميكانيك الكمومى على الشبكة. و سترون فان الامر سهل جدا. مرة اخرى الكمون مازال غير محدد و القضية عامة جدا.
فى الخطوة الاخيرة نعمم نحو الميكانيك الكمومى المصفوفى و سترون ان كل ما ذكر سابقا مازال صالحا.
  
الميكانيك الكمومى على الشبكة 2

 الفيزياء هى أم العلوم و اقدمها. بل الفيزياء هى التى جاءت بالطريقة العلمية.

والفيزياء النظرية هى قلب و عقل الفيزياء.
لكن الفيزياء النظرية قد توغل فى الرياضيات الى الحد الذى ننسى فيه الطريقة العلمية و لا يتبقى معنا الا الطريقة الرياضية.
لكن هناك مجال فى الفيزياء النظرية يتميز بالامرين معا: هو لا يخشى ان يوغل فى الرياضيات لكنه لا ينسى ابدا الطريقة العلمية.
وللأسف فان كثير من الطلبة لا يفهم هذا الامر و يعتقد ان هذا المجال هو (فيزياء عددية) او بالاحرى (بريكولاج عددى) لا اقل و لا اكثر.
هذه فيزياء نظرية فى ذروة الرياضيات (الطريقة الرياضية) و الفيزياء (الطريقة العلمية) فى آن معا.
هذا هو مجال (نظرية الحقول على الشبكة lattice field theory).
ونظرية الحقول على الشبكة صعبة جدا لانها تتعامل بكل بساطة مع حقول نسبية كمومية تحقق الكثير من التناظرات.
اما الميكانيك الكمومى على الشبكة lattice quantum mechanics فهو حالة خاصة من نظرية الحقول على الشبكة و هو اسهل نسبيا منها لكنه ليس سهل ابدا و الا فان الجميع كان قد قام به.
الميكانيك الكمومى على الشبكة لا يتعامل مع حقول نسبية بل يتعامل مع جسيمات نسبية (وهذا اسهل نسبيا) لكنه ينطوى على نفس القدر من التناظرات وهذا يجعله صعب فى حد ذاته الى حد كبير.
نظرية الحقول على الشبكة تسمح لنا باجراء التجارب الافتراضية virtual experiments على الحواسيب باستخدام خوارزميات مونتى كارلو Monte Carlo algorithm وهذا يسمح لنا بسبر غور اعقد النظريات الرياضية للحقول التى تصف التفاعلات الكونية الثلاثة (كهرومغناطيسية و النووية اللونية القوية strong nuclear color و الديناميك الذوقى الضعيف weak flavor dynamics).
وكنا نعتقد سابقا انه من اجبل سبر اغوار نظرية الثقالة الكمومية theory of quantum gravity فاننا نحتاج الى نظرية الحقول على الشبكة او شيء اعقد منها مثلا نظرية الاوتار على الشبكة lattice string theory.
لكن اليوم نحن نعرف ان جزء معتبر جدا من نظرية الثقالة الكمومية يكفى من اجل سبر اغواره استخدام الميكانيك الكمومى على الشبكة.
الميكانيك الكمومى على الشبكة هو اسهل من نظرية الحقول على الشبكة لكنه ليس سهل ابدا و الا فان الجميع كان تمكن من القيام به.
لب صعوبة نظرية الحقول على الشبكة و كذا الميكانيك الكمومى على الشبكة هو كيفية وضع التناظرات الفيزيائية physical symmetries على الشبكة.
والشبكة lattice للتذكير هو الفضاء-زمن مقطع discrete و متناهى finite لان هذا هو الشيء الذى يمكن ان يفهمه الحساب و الحواسيب و المنطق و الخوارزميات و لغات التشفير.
واهم التناظرات التى يجب الحفاظ عليها على الشبكة هى التناظرات المعيارية gauge symmetries و التناظرات الممتازة supersymmetries و التناظرات الكايرالية chiral symmetries.
فى هذه المحاضرة لن نواجه التناظرات المعيارية و هى الاسهل وضعا على الشبكة كما بين ذلك العبقرى ويلسون Wilson فى السبعينات.
فى هذه المحاضرة الثانية نقدم (الميكانيك الكمومى المتناظر-بامتياز على الشبكة supersymmetric quantum mechanics on the lattice).
هذا النموذج هو من نوع نماذج واس-زومينو Wess-Zumino models و هو اول نموذج متناظر بامتياز تم وضعه على الشبكة بنجاح و محاكاته باستخدام طرق مونتى كارلو.
سنواجه معضلة المضاعفة الفرميونية fermion doubling problem و نقوم بحلها. وهذا موضوع ذى علاقة بالتناظر الكايرالى على الشبكة و قد كنا ناقشناه من قبل و نقوم فى هذه المحاضرة بانهائه.
وسنواجه ايضا معضلة وضع التناظر-الممتاز على الشبكة و نقوم بحلها عبر ما يسمى التناظر-الممتاز الملتوى twisted supersymmetry الذى يمت بصلة وثيقة لنظرية الحقول الطوبولوية topological field theory.
الفكرة هو انه يكفى الحفاظ على جزء فقط من التناظر-الممتاز على الشبكة من اجل الحصول على النهاية المستمرة continuum limit الصحيحة بدون الحاجة الى دوزنة fine tuning المؤثرات التى تتطلبها معادلة زمرة اعادة-التنظيم renormalization group equation.
ثم نقوم ايضا بحل معضلة الاشارة الفرميونية fermion sign problem لمحدد determinant مؤثر ديراك Dirac operator و قد كنا اشرنا الي هذه المعضلة عدة مرات. اذن نشرح هنا هذه المعضلة بشكل دقيق و نبين كيف اننا نتفادها بالكامل فى اطار الميكانيك الكمومى الممتاز (وهذا لحسن الحظ).
نقدم ايضا ملخص قصير حول الاعداد الغراسمانية Grassmann numbers و المحدد determinant و الفافيان Pfaffian التى تلعب دورا اساسيا فى تكميم الحقول الفرميونية fermion fields و منها هذه الحقول-الممتازة superfields.
هذه المحاضرة الثانية موجودة على هذه الصفحة على البوابة البحثية للتحميل. المحاضرة الاولى موجودة ايضا على هذه الصفحة على البوابة البحثية للتحميل.

الميكانيك الكمومى المصفوفى هو ميكانيك اغشية او ميكانيك جسيمات

 الفرق بين الميكانيك الكلاسيكى classical mechanics و النسبية العامة general relativity هو الفرق بين الهندسة السمبليكتية symplectic geometry و الهندسة الريمانية Riemannian geometry.

نحن نعلم ان تكميم الهندسة السمبليكتية يعطينا جبريات المؤثرات operator algebras و فضاءات هيلبرت Hilbert spaces و الميكانيك الكمومى quantum mechanics و الهندسة غير-التبديلية non-commutative geometry.
لكننا لا نعلم ماهو تكميم النسبية العامة.
علينا اعادة صياغة الهندسة الريمانية على شكل هندسة سمبليكتية ثم التركيز على الهيئة السمبليكتية symplectic form عوض المترية metric و بالتالى التركيز على المساحات areas عوض الاطوال lengths فنصل الى ضرورة اعتبار الاغشية membranes عوض الاوتار strings و الجسيمات particles.
من هذه الملاحظة نقول ان تكميم النسبية العامة هو الميكانيك الكمومى بشكل او بآخر وهى نتيجة وصلت اليها نظرية الاوتار الممتازة super string theory منذ عشرين سنة لكنها لم تركز عليها بالشكل الصحيح وهو الميكانيك الكمومى المصفوفى matrix quantum mechanics.
بل ان الميكانيك الكمومى المصفوفى توصل اليه الفيزيائيون فى اواخر الثمانينات قبل ثورة الاوتار الثانية فى اواسط التسعينات عندما اكتشفوا ان الغشاء النسبى الممتاز relativistic super-membrane يجب ان يتحرك فى 11 بعد مثلما ان الوتر النسبى الممتاز relativistic super-string يجب ان يتحرك فى 10 ابعاد.
واكتشفوا ايضا ان هذا الغشاء النسبى الممتاز يصفه بالضبط الميكانيك الكمومى المصفوفى.
النظرية الوحيدة الاخرى التى تعيش فى 11 بعد هى الثقالة الممتازة supergravity فى 11 بعد وهى ايضا تحتوى على اغشية مشهورة باسم البراينات M2-brane (الجسيم الكهربائى) و M5-brane (الجسيم المغناطيسى).
اذن الميكانيك الكمومى المصفوفى هو نظرية الطاقات العليا مثلما ان نظرية الثقالة الممتازة هى نظرية الطاقات الدنيا وهما نهايتان لنفس النظرية التى يجب ان تعيش ايضا فى 11 بعد و التى تسمى النظرية M و يرمز لها M-theory و الحرف M كان يعتقد انه يرمز الى الاغشية membrane لكن اليوم يعتقد انه يرمز الى المصفوفات matrices.
بالفعل فان الاكتشافات التى جاءت فيما بعد جعلت الفيزيائيين يغيرون موقفهم (درجات الحرية الاساسية للنظرية M هى ليست اغشية كما كان يعتقد بل هى جسيمات موصوفة بمصفوفات).
بالفعل جاءت نظرية الاوتار الممتازة فى اواخر التسعينات و اكتشفت ان حركة الجسيمات الممتازة super particles فى مايسمى الفضاءات-زمن الموجية pp-wave spacetime تعطى بالضبط بهذا الميكانيك الكمومى المصفوفى.
نظرية الاوتار اكتشفت ايضا ان هذه الجسيمات الممتازة هى فى الحقيقية جسيمات دريشليه Dirichlet particles او البراينات (مفرد براين) D0-branes التى تشكل الجسيمات الذرية المشكلة للثقوب السوداء.
اذن الميكانيك الكمومى المصفوفى يصف الثقوب السوداء. وهذا كان نموذج آخر للثنائية الثقالية-المعيارية gauge-gravity duality مختلف عن نموذج ال AdS/CFT.
بالفعل هذا الميكانيك الكمومى المصفوفى نحصل عليه ايضا من الاختزال البعدى dimensional reduction لنظريات يانغ-ميلز Yang-Mills theories فى عشرة ابعاد.
هذه البراينات D0-branes هى فى الحقيقة انماط كالوزا-كلاين Kaluza-Klein modes (الناجمة عن الاختزال البعدى) للغرافيتون graviton فى 11 بعد.
اذن الميكانيك الكمومى المصفوفى هو ميكانيك جسيمات (حركة جسيمات-ممتازة) لكنه ايضا ميكانيك اغشية نسبية وهو ايضا يصف الثقوب السوداء و الغرافيتون و الحقول المعيارية يانغ-ميلز و الهندسة غير-التبديلية للفضاء-زمن و اشياء اخرى كثيرة.


الثنائية الثقالية-المعيارية

 ماهى الثنائية الثقالية-المعيارية gauge-gravity duality?

هذا امر صعب جدا بل ربما هو من اصعب الامور لان نظرية الاوتار الممتازة هى ليست بالامر الهين ابدا.
لكنه امر رغم صعوبته فهو امر يمكن شرحه ببساطة كما يمكن فهمه ايضا فهما تاما ببعض الجهد و التوجيه.
بكل بساطة هذه الثنائية -التى تصدرت ميدان الفيزياء النظرية خلال ال 25 سنة الماضية منذ ظهرت عام 1998 -تنص على ان النظريات المعيارية-الممتازة هى مكافئة للنظريات الوترية-الممتازة.
النظريات المعيارية-الممتازة supersymmetric gauge theories هى تعميم لكهرومغناطيسية ماكسويل وهى تتميز بوسيطين parameters هما العدد الطبيعى N (رتبة الزمرة المعيارية rank of the gauge group) و ثابت الاقتران المعيارى gauge coupling constant الذى يرمز له ب gYM (الذى هو تعميم للشحنة الكهربائية).
اما النظريات الوترية-الممتازة superstring theory فهى تعميم لنظريات الثقالة-الممتازة supergravity theories التى هى فى حد ذاتها تعميم للنسبية العامة لاينشتاين. هذه النظريات تتميز ايضا بوسيطين: طول الوتر string length الذى يرمز له ب 'α و ثابت الاقتران الوترى string coupling constant الذى يرمز له ب gs الذى يرتبط بحقل الديلاطون dilaton field وهو حقل سلمى scalar field يرمز له ب φ عبر الدالة الاسية. اى ان gs=exp(φ).
من ناحية النظرية المعيارية لما يذهب N الى مالانهاية مع الحفاظ على ثابت اقتران توهفت t Hooft' المعرف ب λ=gYM**2.N ثابت -وهذه تسمى نهاية الاقتران القوي strong coupling limit- فان النظرية الوترية-الممتازة (التى هى نظرية كمومية تصف اوتار) تختزل الى نظرية الثقالة-الممتازة (التى هى نظرية كلاسيكية تصف جسيمات).
التصحيحات الوترية stringy corrections المتناسبة مع 'α (اى كون درجات الحرية هى فعلا اوتار و الجسيمات هى تقريب فقط) نحصل عليها باخذ الحدود المتناسبة مع مقلوب ثابت اقتران توهفت λ.
اما التصحيحات الكمومية quantum corrections المتناسبة مع gs (اى كون الفيزياء هى كمومية فعلا و الكلاسيكى هو تقريب فقط) فاننا نحصل عليها باخذ الحدود المتناسبة مع مقلوب رتبة الزمرة المعيارية N.
النظرية المعيارية هى نظرية N من براينات Dp (و البراين brane هو جرم بعده p يتحرك فى الفضاء-زمن). فعل action هذه النظرية (الذى يعرف تكامل طريق فايمان Feynman path integral) معطى بالمعادلة فى الصورة الاولى.
اما النظرية الوترية فهى نظرية ثقالة-ممتازة حول البراين الاسود black brane الذى تعطى متريته metric بالمعادلة فى الصورة الثانية.
البراين الاسود هو ثقب اسود معمم بعده p (الثقب الاسود الذى نعرفه من النسبية العامة هو ثقب اسود نقطى بعده 0).
البراين الاسود هو عبارة عن حالة مرتبطة bound state ل N براين Dp (مثلما ان الثقب الاسود العادى هو حالة مرتبطة من N براين D0).
العلاقات بين النظرية المعيارية-الممتازة و النظرية الوترية-الممتازة -التى شرحناها اعلاه- ملخصة فى المعادلة فى الصورة الثالثة وهذه هى بالضبط الثنائية الثقالية-المعيارية.
اذا وضعنا p=0 فى كل هذه المعادلات فاننا نحصل على الميكانيك الكمومى المصفوفى BFSS الذى يقابل براين اسود هو فعلا ثقب اسود عادى مشكل من N جسيم نقطى هى براينات D0.




كورس فى الثنائية الثقالية-المعيارية

 رغم كل شيء و رغم كل الظروف فانه دائما هناك الكثير من الطلبة الجيدين ممن يريد ان يدرس الفيزياء النظرية.

ولو علم الطلبة او كان لهم الثقافة الكافية فى مستوى الثانوى لكان اختيار الكثير منهم من الممتازين بعد النجاح فى البكالوريا هو الذهاب الى الفيزياء من اجل التخصص فى الفيزياء النظرية و ليس اختيار الطب او اى شيء آخر لاسباب غير موضوعية.
الطلبة فقط لا يعرفون و المجتمع و الثقافة يشكلان غشاءا غليظا امام اعينهم قل من يستطيع ان ينظر عبره او من خلاله بدون الحد الادنى من الثقافة و الاطلاع.
و اكرر مرة اخرى اننى لا انصح اى احد بالتخصص فى الفيزياء النظرية بل على العكس من ذلك.
اذن من يتوهم -خاصة من الطلبة عندنا- اننى ادعوا الى هذا التخصص فهو واهم اكثر من وهمه بل اننى اريد فعلا من الجميع ان يذهب الى اشياء اخرى و يتركنا مع الفيزياء النظرية فاننا فعلا نحبها رغم كل شيء و لا نريد لكل من هب و دب ان يدخل علينا و يشاركنا فيها.
لكن الطلبة بلغ جميعهم سن الرشد و هم احرار فى الاختيار و يتحملون عواقب اختياراتهم. واننى اتحدث هنا الى من سيأتى الى الفيزياء النظرية رغم تحذيرى اياهم.
وعلى كل حال فان اغلب هؤلاء ممن سيأتى الينا سوف يندم و سوف يحاول ان يغير الوجهة بعد فوات الاوان -مثلا الدفعة السابقة عندنا لا احد منها اختار العمل على مواضيع الفيزياء النظرية العميقة و بسبب ذلك لم نقدم مسابقة دكتوراة هذا العام و اؤكد ان لا احد منهم سوف يمر الى الدكتوراة فى المستقبل بسبب ذلك الاختيار الخاطئ ثم الاختيار الاكثر خطأ-. فنحن لسنا نلعب هنا.
اذن من يختار الفيزياء النظرية فعليه ان يكون متيقنا تماما من ان اختياره موضوعى و ليس عاطفى او اجتماعى -تأثير الحشد-.
ونحن فى الاخير لا نلوم هؤلاء الطلبة ممن يريد الفيزياء النظرية على اختيارهم للفيزياء النظرية.
فان اعظم الفيزيائيين فى التاريخ هم فيزيائيون نظريون و على رأسهم الآباء نيوتن -الميكانيك الكلاسيكى- و اينشتاين -النسبية- و بولتزمان -الترموديناميك- و ماكسويل -الكهرومغناطيسية- و بوهر -الميكانيك الكمومى-.
الفيزياء فعلا تقودها التجربة لكن تحكمها فى الاخير النظرية. التجربة هى جوارح الفيزياء لكن عقل و قلب و روح الفيزياء هى الفيزياء النظرية.
الآن الى من يريد ان يتخصص فى الفيزياء النظرية.
اقول اولا ان اهم مادتين فى الفيزياء النظرية اذا تم تدريسها بشكل صحيح هما نظرية الحقول الكمومية -التى هى توحيد للميكانيك الكمومى و الكهرومغناطيسية - و نظرية النسبية العامة -التى هى توحيد للميكانيك الكلاسيكى و النسبية الخاصة-.
وتدريس هاتين المادتين من اصعب ما يكون بالنسبة للاستاذ قبل الطالب. وهذا لا يعيه لا الاستاذ -اغلبهم- و لا الطالب.
شخصيا اذا تم فتح دفعة ماستر نظرى جديدة العام القادم فى الفيزياء النظرية فاننى لن ادرس لا نظرية النسبية العامة و لا نظرية الحقول الكمومية.
بل اقترح فى مكانهما تدريس الثنائية الثقالية-المعيارية gauge-gravity duality التى هى توحيد للنسبية العامة و نظرية الحقول الكمومية مع تأثير واسع جدا لنظرية الاوتار الممتازة و الحاسوبية الكمومية.
البعض قد يقول هذا امر صعب جدا.
الجواب ان نظرية الحقول الكمومية فى حد ذاتها صعبة جدا و نظرية النسبية العامة اصعب منها.
و كفانا بحثا عن التسهيل للطلبة فهذا لم ينفعهم و هم لم يحترمونا على تسهيلنا المستمر للامور لهم.
لندرس الامور على حقيقتها و من اراد ان يلتحق بالركب فعليه الالتزام و بذل الجهد.
فالفهم يبقى مسؤولية الطالب فى الاخير و ليست مسؤولية الاستاذ. الاستاذ مسؤوليته تقريب الموضوع و قيادة الطالب الى الافكار و المراجع و اسس الموضوع و محاولة شرحها بدقة و بتبسيط قدر الامكان.
اذن اننى ادعوا الى تدريس الثنائية الثقالية-المعيارية بالموازاة مع نظرية الحقول الكمومية و النسبية العامة فى مستوى الماستر.
واننى أدعى عن اقتناع تام ان الثنائية الثقالية-المعيارية لو تأملنا فيها ليست اصعب من النسبية العامة و نظرية الحقول الكمومية بل هى فى مثل صعوبتهما.
فى الرابط مرجع ممتاز عن الثنائية الثقالية-المعيارية و المؤلف هو فيزيائى نظرى مشهور اسمه ناستايسه Nastase و هو احد مؤسسى الميكانيك الكمومى المصفوفى BMN.
المؤلف يقسم الكتاب الى ثلاثة اقسام.
القسم الاول مدخل.
القسم الثانى الثنائية الثقالية-المعيارية الاولى التى اكتشفها مالداسينا Maldacena عام 1998 وهى التقابل AdS5/CFT4.
القسم الثالث تعميم للتقابل AdS/CFT و للثنائية الثقالية-المعيارية.
القسم الاول يحتوى على 9 فصول تحضيرية:
-نظرية الحقول الكمومية و النظرية المعيارية.
-النسبية العامة و فضاءات AdS.
-التناظر-الممتاز supersymetry.
-الثقالة-الممتازة supergravity.
-الاختزال البعدى dimensional reduction.
-الثقوب السوداء و البراينات branes.
-نظرية الاوتار.
-نظرية الحقول الكونفورمال conformal.
-البراينات Dp.
القسم الثانى يتناول بالدراسة الثنائية الثقالية-المعيارية الاولى المكتشفة تاريخيا و هو التقابل AdS5/CFT4.
هذا التقابل مهم جدا لانه يسمح لنا بدراسة الثقالة الكمومية فى 10 ابعاد -بالضبط نظرية الاوتار الممتازة التى تعيش فى فضاء AdS5- بدلالة نظرية حقل كمومى كونفورمالية فى اربعة ابعاد -بالضبط نظرية يانغ-ميلز Yang-Mills المعيارية الممتازة التى تعيش على فضاء-زمن مينكوفسكى-.
هذا القسم يحتوى ايضا على 9 فصول.
-التقابل AdS/CFT.
-وصفة ويتن Witten.
-الهولوغرافيا holography.
-السوليتونات solitons.
-الكوراك وحلقة ويلسون Wilson loop.
-درجة الحرارة و البلازما.
التصادمات scattering.
-الميكانيك الكمومى المصفوفى.
-نماذج السبين spin models.
القسم الثالث يحتوى على 9 فصول اضافية و هو يحتوى على تعميم للثنائية الثقالية-المعيارية و التقابل AdS/CFT للابعاد الدنيا.
-الامثلة الكونفورمالية الاخرى للتقابل AdS/CFT.
-النموذج ABJM.
-الثنائيات الثقالية gravity duals.
-الاستنظام الهولوغرافى holographic renormalization.
-تدفق معادلة زمرة الاستنظام RG.
-فينومينولوجيا phenomenology الثنائية الثقالية-المعيارية: التقابل AdS/QCD. اى التقابل مع الكروموديناميك الكمومى quantum chromodynamics اى QCD.
-فينومينولوجيا الثنائية الثقالية-المعيارية: التقابل AdS/CMT. اى التقابل مع فيزياء المادة المكثفة CMT.
-تصادم الغليونات gluons.
-التشابك الكمومى الهولوغرافى holographic entanglement entropy. وصفة ريو-تاكاياناغى Ryu-Takayanagi.


العامل الاجتماعى فى البحث العلمى

 أهم شيء فى العلم هو البحث.

و اهم شيء فى البحث هو النشر الذى يعتد به.
و اهم شيء فى النشر هو النجاح و النجاح هنا نقصد به ان يكون لعملك قبول من الاقران اى من الباحثين من امثالك فى ذلك المجال او الموضوع.
أما علامة النجاح مع الاقران و القبول من الاقران فهو ان يكون لديك عدد محترم من الاستشهادات citations. اى ان يرجع الى عملك و يستشهد به اكبر عدد ممكن من الاقران.
هذا هو ما نصارعه يوميا كباحثين. العبرة ليست بالنشر و لا بالدورية التى تنشر فيها و لا بعدد المنشورات. بل العبرة كل العبرة بعدد الاستشهادات. ذلك هو العامل الحاسم النهائى.
و قد يحدث ان يكون لباحث مزيف عدد هائل من الاستشهادات من المزيفين من امثاله او فى مجال مزيف. هذا ايضا موجود. و علينا ان نحذر من كل هذا الزيف و التزييف الذى نجده حتى فى العلم.
لكن كل مجال لديه دوريات مرموقة و باحثين مرموقين (يعرفهم الجميع فى ذلك المجال) و الاستشهادات من هؤلاء هى التى نتكلم عنها هنا.
اذن هذا ما نصارعه فى آخر المطاف القبول و النجاح عبر تحقيق اكبر عدد ممكن من الاستشهادات من الاقران المعترف بهم فى المجال.
و هذا الطريق هو ليس طريق علمى بحت بل هو ايضا للأسف طريق اجتماعى بامتياز.
اذن الباحث يحتاج الى مجتمع علمى يعرفونه و يعرفهم حتى يحقق هذا النجاح.
و بدون العامل الاجتماعى (بسلبياته و ايجابياته) فان صراعه يتحول الى صراع دون كيشوت مع طواحين الهواء اى لن يجنى من وراءه اى نجاح ملموس او يذكر.
هذا العنصر الاجتماعى هو اكبر عامل يتضرر منه الباحث الجزائرى او العربى الذى يعمل من الداخل -داخل الوطن- اما الجانب العلمى فى حد ذاته فقد يحقق فيه الانسان اعلى المستويات لكن يبقى امرا بدون اثر اذا لم يكن هناك تفاعل اجتماعى مع الخارج -خارج الوطن-.
و العكس ايضا صحيح فان الباحث الجزائرى الذى يعمل من الخارج -خارج الوطن- قد يبلغ اعلى المستويات فقط من جهة تفاعلاته الاجتماعية مع ذلك الخارج وليس بالضرورة من امتلاكه اى قوة علمية حقيقية.
هذا هو كيف يتم تقييم الباحث العلمى فى مجاله.
لكن على مستويات الطلبة و طلبة الدكتوراة و طلبة ما بعد الدكتوراة فان معرفة المستوى العلمى هو امر اسهل بكثير.
مثلا على مستوى طلبة ما بعد الدكتوراة او البوست-دوك post-docs فان مسابقات البوست-دوك هى معيار ممتاز.
واذكر هنا جزء من سيرتى الذاتية كمثال.
مثلا اتذكر اننى دخلت عام 2005 مسابقة (متابع مارى كورى الدولى Marie Curie International Fellow) التابعة للاتحاد الاوروبى European Union و قد شارك فى المسابقة حوالى 79 فيزيائى نظرى فى مستوى البوست-دوك من مختلف انحاء العالم و قد ترتبت شخصيا بينهم فى الرتبة 20.
اللجنة لجنة المسابقة قررت اخذ ال 19 مترشح الاول و بذلك كنت الاول فى قائمة الانتظار waiting list. اذن فى الانتقاء الاول first selection لم انجح برتبة واحدة و هذا امر لا استطيع تصديقه الى غاية يومنا هذا.
لحسن الحظ لم يتمكن البعض من ال 19 الاوائل الذين اختارتهم اللجنة ان يلتحق بهذا المنصب و بهذا دخلت قائمة المترشحين الناجحين فى المسابقة بعد معاناة نفسية مع الانتظار القاتل.
مشروع البحث الذى قدمته للجنة كان -محاكاة نظرية الحقول غير-التبديلية فى اربعة ابعاد Simulation of noncommutative field theory in 4 dimensions- و هو موجود فى الرابط.
هذا موضوع مازال يصلح اليوم لمن كان مهتما بالامر لانه موضوع لم يحسم بعد رغم كل هذه السنوات التى مرت.
السؤال الآن اذا كنت قد ترتب ال 20 من 80 على مستوى البوست-دوك اى فى ال 25 بالمائة الاولى قبل كل تلك السنوات الطوال فهل سأترتب اليوم ايضا فى ال 25 بالمائة الاولى على مستوى الاساتذة من اقرانى?
الجواب هو اكيد لا.
اذن شخصيا رغم ان العلم الذى حققته اليوم اكبر بكثير من العلم الذى كان لدى فى ذلك الوقت الا ان ترتيبى العلمى قد تقهقر بكثير وهذا كله بسبب العامل الاجتماعى الذى كنت قد ذكرته و الذى يبدو اننى قد قصرت فيه او على الاقل لم ابذل فيه ما بذلت من جهد فى الجانب العلمى.
اذن يجب ابدا عدم التقصير فى العامل الاجتماعى -او نسيانه خلال صراعك مع الجانب العلمى- و محاولة بناء تعاون علمى و تواصل بحثى مع الخارج لانه عامل حاسم فى التطور المستمر و النجاج المستقبلى ان شاء الله.
الاعداد الغراسمانية

مؤثر ديراك Dirac operator عندما يظهر فى تكامل الطريق فايمان Feynamn path integral فانه يظهر ضرورة مضروب فى اعداد غراسمانية Grassmann numbers و ليس اعداد مركبة complex numbers كما تبينه المعادلة 7.17 او 7.20 و النهاية الكلاسيكية 7.18.

الاعداد الغراسمانية هى اعداد فرميونية عكس الاعداد المركبة التى هى اعداد بوزونية.
هذا يعنى ان اهم خاصية للاعداد الغراسمانية هى انها اعداد ضد-تبادلية anti-commuting و بالتالى فان مربعها صفر بالضرورة (المعادلة 7.18).
هناك خواص اخرى كثيرة مثلا فان دالة كيفية فى عدد غراسمانى هى بالضرورة دالة خطية و التكامل على العدد الغراسمانى مكافئى تماما للتفاضل على العدد الغراسمانى.
الاعداد الغراسمانية يمكن ان تكون مركبة و قد تكون حقيقية بالمعنى المتعارف عليه. اى انه اذا كان الارفاق المركب complex conjugation للعدد الغراسمانى يعطينى نفس العدد فان هذا العدد الغراسمانى هو عدد غراسمانى حقيقى و الا فهو عدد غراسمانى مركب.
أهم شيء بالنسبة للتكامل الفرميونى على الاعداد الغراسمانية (تكامل طريق فايمان على سبينورات ديراك) هى النتيجة 7.40 و قارنوا هذه النتيجة مع التكامل البوزونى على الاعداد المركبة النتيجة 7.41.
المصفوفة M تلعب دور مؤثر ديراك اذن نحصل على محدد determinant مؤثر ديراك اى det M عكس التكامل البوزونى اين نحصل على محدد مقلوب M وهذا فرق هائل جدا بين الحقول الفرميونية (سبين نصف صحيح) و الحقول البوزونية (سبين صحيح).
لو كانت الاعداد الغراسمانية هى اعداد غرامسانية حقيقية فاننا نحصل على الفافايان Pffafian عوض المحدد و الفافيان هو الجذر التربيعى للمحدد (تقريبا). حساب الفافيان اصعب عموما من حساب المحدد و حساب المحدد هو اصعب عملية مصفوفية قاطبة.
هذا المحدد او هذا الفافيان الديراكى هو نقطة انطلاق نظرية الحقول على الشبكة التى تحتاج الى محاكاة الجسيمات و الحقول الفرميونية او محاكاة التناظر الممتاز supersymmetry.

مستويات النجاح

 النجاح مستويات متعددة و البعض يتكلم عن النجاح دون ان يدرك و يعى ان للنجاح مستويات مختلفة بل ان النجاح الحقيقى لا يستطيع البعض ادراكه و رؤيته على حقيقته بل هو شيء مضبب غامض عندهم ليس له كينونة محددة ثابتة.

واننى هنا اتكلم عن العلم اما الحياة فهى شيء اكبر من العلم و ليس لى اى دخل بها بل اننى اصراحكم و اقول اننى لا افهم فيها فعلا.
هنا ادعى و اقول ان هناك على الاقل اربعة مستويات مختلفة من النجاح: النجاح الوظيفى و النجاح الاكاديمى و النجاح العلمى و النجاح الابداعى.
اولا هناك النجاح الوظيفى فى تحقيق النقاط و تحقيق الشهادات و تحقيق الوظائف و تحقيق الرتب. هذا كله نوع واحد لا فرق بين النقطة و الشهادة و الوظيفة و الرتبة. هذا نجاح ادارى قانونى لا اقل و لا اكثر. وهو سهل جدا لكل جاد مجد و جيد وهذا بشرط ان لا توضع عثرات غير طبيعية من المعرقلين الطفيلين المدخولين فى طريق الجاد و المجد و الجيد.
ثانيا لكن هناك ايضا النجاح الاكاديمى فى تحقيق الابحاث من الناحية الكمية و النوعية. وهذا نجاح جاف ميكانيكى قد يكون صعب التحقيق لكنه فى متناول كل جاد و مجد و جيد. وهو ضرورى و خطوة مهمة فى الطريق لكنه ليس نهاية الطريق كما يعتقد الجميع. و من اجل تحقيق هذا النجاح يكفى ان تجد الدائرة الاكاديمية التى تُنتج فيها و معها و لها فتتحقق بذلك الاستمرارية العقيمة.
ثالثا لكن هناك ايضا النجاح العلمى الذى يتميز بتحقيق تخصص موسوعى و ثقافة شاملة و فهم عميق مترابط غائى لذلك المجال العلمى. وهو قد يترافق و قد لا يترافق مع النجاح الاكاديمى. لكن اذا ترافق مع النجاح الاكاديمى الجاف الميكانيكى فانه ينتج عنه نجاح علمى خصب و حيوى رائع يمكنك ان تقدم عبره لغيرك الكثير. غير ذلك فاننى شخصيا افضل النجاح العلمى على النجاح الاكاديمى ويمكنك ان تقدم ايضا عبر النجاح العلمى لغيرك الكثير و بشكل افضل بكثير. لكن فى الواقع قد يصعب جدا الجمع بين النجاح الاكاديمى الميكانيكى و النجاح العلمى الحيوى و النجاح العلمى هو بطبيعته اصعب بكثير فى التحقيق لانه يتميز بمصداقية و صدق كاملين و ليس فيه تنازل عن اى قيمة من قيم العلم.
رابعا اما المستوى الاخير فهو النجاح الابداعى. وهو ان تُبدع الجديد و تأتى بالاصيل فى ذلك العلم و تصبح حجة فى المجال يرجع اليك الجميع من اصجاب النجاح الوظيفى و النجاح الاكاديمى و النجاح العلمى. الناجح ابداعيا هو فى العموم ذلك من استطاع تحقيق النجاح الاكاديمى بسهولة شديدة ثم استطاع تحقق النجاح العلمى بصعوبة قليلة ثم تعدى هذه المرحلة بمراحل الى مرحلة الابداع وهى اصعب بكثير.
هذه الفئة كما ذكرت فى منشور قديم هم 1 فى المائة -او ربما اقل بكثير- فى كل جيل. وتذكروا سلم لانداو الذى تكلمت عنه فى منشور آخر قديم الذى يرتب فيه اصحاب هذا المستوى فى مراتب. فمثلا نيوتن و اينتشاين و بولتزمان و بوهر هم مستوى لوحدهم و غيرهم يأتون فى مستويات ادنى منهم لكنهم كلهم حققوا النجاح الابداعى.
اذا قارنا بما نعرفه فاننى اقول ان النجاح الوظيفى هو مثل الفقيه او المتكلم المقلدان اما النجاح الاكاديمى فهو مثل المجتهد العالم فى الشريعة اما النجاح العلمى فهو مثل المتصوف العارف بالحقيقة اما النجاح الابداعى فهو مثل حالة النبى (وللنبى المثل الاعلى) الذى عنده حبل من الله مباشرة.
طرق مونتى كارلو الهجينة

هذا مقال آخر من ماسانورى هانادا Masanori Hanada و هو فيزيائى نظرى يابانى أشاركه الفلسفة العامة فى الفيزياء النظرية و هى محاولة (ارجاع نظرية الاوتار الممتازة superstring theory الى حضن الطريقة العلمية للفيزياء و الفيزياء النظرية وهذا عن طريق اخضاع نتائجها الى طرق نظرية الحقل على الشبكة lattice field theory).

فى هذا المقال يقدم المؤلف مدخل سريع جدا و فعال الى طرق مونتى كارلو Monte Carlo method المستخدمة فى نظرية الحقل على الشبكة.
اولا يقدم سلاسل ماركوف Markov chain بصفة عامة جدا. وهى التأسيس الرياضى-الاحصائى لطرق مونتى كارلو.
ثم يقدم خوارزمية ميتروبوليس Metropolis algorithm التى تقع فى قلب طرق مونتى كارلو.
ثم يقدم خوارزمية مونتى كارلو الهجينة hybrid Monte Carlo algorithm فى اطار النماذج البوزونية bosonic models.
ثم يقدم خوارزمية مونتى كارلو الهجينة العقلانية rational hybrid Monte Carlo algorithm فى اطار النماذج الفرميونية fermionic models.
وتذكروا فان الحقول البوزونية هى الحقول ذات عزم-اللف spin الصحيح integer و الحقول الفرميونية هى الحقول ذات عزم-اللف نصف-الصحيح half-integer.
مثال عن الفرميونات الالكترون و الميون و الكوارك ( اى المادة) و مثال عن البوزونات الفوتون و الغليون و الغرافيتون (اى الاشعاع).
و خوارزمية مونتى كارلو الهجينة و خوارزمية مونتى كارلو الهجينة العقلانية هما الخوارزميتان الاساسيتان المستخدمتان فى الكروموديناميك الكمومى quantum chromodynamics و فى النظريات المعيارية gauge theories و فى نظريات يانغ-ميلز Yang-Mills theories و فى نظريات التناظر-الممتاز supersymmetric theories و فى النماذج المصفوفية matrix models و فى الميكانيك الكمومى على الشبكة lattice quantum mechanics و فى نظرية الحقل على الشبكة lattice field theory بصفة عامة.
المؤلف يقارن بالخصوص بين الكروموديناميك الكمومى (وهو المثال التاريخى الانجح فى هذا المجال و الذى ابتدا فى السبعينات و ابتدأت به نظرية الحقول على الشبكة) و بين نظريات يانغ-ميلز الممتازة (لانه المثال الاهم بالنسبة لنظرية الاوتار الممتازة التى تحتويها كجزء اساسى فى بنيتها الرياضية) و كيفية تطبيق خوارزمية مونتى كارلو الهجينة العقلانية على هذه الاخيرة.
هذه الخوارزمية هى خوارزمية معقدة جدا يقع فى قلبها خوارزمية ميتروبوليس التى يتم على اساسها رفض reject او قبول accept التحديث update الذى نقترحه.
لكن من اين يأتى الاقتراح proposal الذى يتم قبوله او رفضه. هنا تفترق خوارزمية ميتروبوليس العادية عن خوارزمية مونتى كارلو الهجينة فى ان هذه الاخيرة تستخدم حل معادلات هاميلتون للحركة Hamilton equations of motion من اجل تقديم حل هذه الاخيرة كاقتراح لخوارزمية ميتروبوليس حتى تقبله او ترفضه.
معادلات هاميلتون للحركة يتم حلها عن طريق الديناميك الجزيئى molecual dynamics وهذا باستخدام مثلا خوارزمية قفزة-الضفدع leap-frog algorithm.
و كما تعلمون فان خوارزمية الديناميك الجزيئى تحتوى على اخطاء منهجية systematic errors تقوم خوارزمية ميتروبوليس باعدامها بالكامل وهذا هو النجاح الخارق للعادة الذى تم اكتشافه عندما تم اكتشاف خوارزمية مونتى كارلو الهجينة التى هى فى الحقيقة مزيج فى غاية القوة و الدقة لخوارزمية ميتروبوليس مع خوارزمية الديناميك الجزيئى.
اذن خوارزمية مونتى كارلو الهجينة مثلها مثل خوارزمية ميتروبوليس لا تحتوى على اى اخطاء منهجية systematic errors (التى هى اخطاء رياضية ناجمة عن التقريب لا تريدها الفيزياء) بل تحتوى فقط على اخطاء احصائية statistical errors (وهذه اخطاء مقبولة جدا لانها اخطاء من نوع الاخطاء التجريبية).
يتم حساب هذه الاخطاء باستخدام طرق اومبيركية empirical methods اهمها على الاطلاق طريقة الجاكنايف Jackknife method و هى طريقة يشرحها ايضا المؤلف.
خوارزمية مونتى كارلو الهجينة تحتوى ايضا على خوارزمية الخزان الحرارى heat bath algorithm التى تسمح لنا بتوليد كميات الحركة momentum من توزيع غوسى Gaussian distribution و هذا من اجل ضرورة تفادى ما يسمى المعضلة الارغودية ergodic problem (وهو عجز بعض الخوارزميات عن سبر جميع اجزاء فضاء الطور phase space).
الآن عندما نقوم بادخال الفرميونات الى الموضوع اى ادخال محدد مؤثر ديراك determinant of Dirac operator الى الموضوع فان درجة تعقيد الموضوع تزيد مثل الذى بين السماء و الارض. هذه فعلا هى اصعب نقطة ولولا هذه النقطة لما كان هناك اى صعوبة فى الموضوع.
عند الاضطرار الى حساب محدد مؤثر ديراك (وهذه هى حالة جميع النظريات التى ذكرتها فى البداية بدون استثناء) فان خوارزمية مونتى كارلو الهجينة تصبح تسمى خوارزمية مونتى كارلو الهجينة العقلانية لانها تستخدم 3 خوارزميات اضافية من اجل حساب محدد ديراك.
اولا محدد ديراك يأتى عموما مرفوعا الى اس power معطى بعدد عقلانى rational number اى عدد كسرى. هذا صعب جدا للحساب فى حد ذاته و اذن نستخدم ما يسمى التقريب المينيماكس minimax approximation من اجل التعبير عن هذا الاس الكسرى بدلالة كثير حدود polynomial. من الناحية العملية هذا التقريب المينيماكس يتم حسابi باستخدام خوارزمية ريماز Remez algorithm الشهيرة.
ثانيا محدد ديراك لا يتم حسابه مباشرة فهذا هو لب صعوبة هذا الامر الصعب. ولهذا فاننا نقوم بالتعبير عنه باستخدام حقول تسمى الفرميونات-المزيفة pseudo-fermions وهى حقول تمتلك جميع الاعداد الكمومية quantum numbers للفرميونات (سبينورات ديراك Dirac spinors) لكنها ليست سبينورات بل هى بوزونات سلمية scalar bosons. هذه الطريقة تسمى طريقة الفرميونات-المزيفة pseudo-fermion method.
ثالثا نجد بعد الحساب ان هذه الفرميونات-المزيفة ترتبط بحقول غوسية Gaussian fields (وهذا من اسهل الحقول على الاطلاق) عن طريق مصفوفة تساوى مؤثر ديراك مرفوع لاس كسرى آخر.
هنا نستخدم التقريب المينيماكس و خوارزمية ريماز مرة اخرى.
ونستخدم ايضا خوارزمية الخزان الحرارى مرة اخرى لتوليد الحقول الغوسية.
لكن اهم من كل هذا -وهو اللب الآخر لخوارزمية مونتى كارلو الهجينة العقلانية- هو ضرورة استخدام ما يسمى خوارزمية التدريج المرافق conjugate gradient method وهى خوارزمية معقدة و قوية جدا تسمح لنا بحساب مقلوب inverse مصفوفة (هنا مؤثر ديراك) بدون ان نحسب فعلا هذا المقلوب.
و بعد كل هذا فانه لا يجب ان ننسى انه يجب ايضا استخدام خوارزمية الديناميك الجزئى و خوارزمية ميتروبوليس على محدد ديراك بعد اجراء كل تلك العمليات الآنفة الذكر.
كل هذه الخوارزميات مركبة مع بعضها البعض -هى عبارة عن ذكاء اصطناعى artificial intelligence فى غاية الذكاء- يستعمل منذ التسعينات فى نظريات الحقول المعيارية و مازال يستخدم اليوم فى نظريات التناظر-الممتاز و نظريات الثقالة الكمومية وهذا قبل ان تخرج اصلا موضة الذكاء الاصطناعى فى علوم الحاسوب فى هذا الزمان.


الفيزياء النظرية فى اطار الفيزياء الحاسوبية

هناك عدة خطوات عند القيام بالفيزياء النظرية -فى اطار الفيزياء الحاسوبية- كما افهمها شخصيا.

اولا علينا فهم الفكرة و التصور و المفهوم الفيزيائى.
ثانيا علينا التحكم التام فى الرياضيات التى نؤسس عليها الفكرة الفيزيائية الاساسية.
ثالثا علينا تحويل النظرية الفيزيائية و الرياضيات التى تقوم عليها النظرية الفيزيائية الى نموذج منطقى حسابى عددى.
رابعا علينا التحكم الرياضى فى الخوارزميات التى سوف تقوم بحل النموذج المنطقى الحسابى العددى. اذا كان الهدف هو اجراء محاكاة عددية (اى تجربة افتراضية) للنظرية الفيزيائية فليس هناك مناص من سلاسل ماركوف و طرق مونتى كارلو.
خامسا علينا تشفير الخوارزميات باستعمال احدى لغات البرمجة. و اذا كان الهدف هو اجراء محاكاة عددية (اى تجربة افتراضية) فانه يجب استعمال احدى لغات التشفير الاعلى او ما يسمى الاوتوكود.
سادسا الانجاز و التنفيذ. هل الشفرة تؤدى الى فيزياء يمكن فهمها و تفسيرها ام ان العملية قد فشلت.
وجدت طلبة و اساتذة يأتون اليك بعد أن يكتبوا شفرة و يقولون لك (قد كتبنا شفرة). الهدف ليس هو كتابة الشفرة فى حد ذاتها. الهدف هو التحصل على فيزياء يمكن فهمها و تفسيرها. الهدف هو الانجاز و التنفيذ. اما الكتابة فالجميع يكتب. الهدف هو كتابة شيء يحمل معنى و غاية و قيمة.

محاضرات فى الميكانيك الكمومى

 هذه مجموعة من المحاضرات فى الميكانيك الكمومى وهى غير منشورة كنت قد قدمتها منذ اكثر من عشر سنوات مع احدى دفعات الماستر النظرى الاولى.

أهم ميزة لهذه المحاضرات هى السرعة الشديدة التى غطيت بها اهم مواضيع الميكانيك الكمومى (الذى هو موضوع شاسع جدا) فى مساحة ضيقة جدا من الوقت و الصفحات.
لاحظوا ايضا ان هذه المحاضرات هى مكتوبة باللغة الانجليزية لاننا درسنا للطلبة باللغة الانجليزية منذ البداية فى الماستر (و درسنا بالعربية فى الليسانس و قد كنت الوحيد الذى يقوم بذلك لمدة تزيد الآن على 13 سنة).
وهم مازالوا فى عام 2023 يتناقشون هل هى فرنسية ام انجليزية -وكأنها قضية هل انا افكر اذن هل انا موجود- و هذا من شدة التخلف الذهنى للثقافة الجزائرية و الجامعة الجزائرية.
هذه المحاضرات بالانجليزية هى الاصل الذى بنيت عليه بعد ترجمته الى اللغة العربية القسم الثالث من كتاب (الفيزياء الاساسية).
الآن اهم شيء بالنسبة لشخص يريد ان يدرس الميكانيك الكمومى بسرعة هو الآتى:
-فضاء هيلبرت و الوصول الى معادلة شرودينغر.
-التناظر الدورانى و الوصول الى جبرية الزمرة SO(3) التى هى زمرة العزم الحركى.
-حل معادلة شرودينغر بالنسبة لذرة الهيدروجين و الوصول الى علاقة بوهر -التى اكتشفها بوهر قبل ان يكتشف الميكانيك الكمومى بأكثر من 7 سنوات-.
-نظرية الاضطراب غير-المتعلقة بالزمن حتى يمكننا حساب البنية الدقيقة لذرة الهيدروجين و طاقة ذرة الهيليوم. هذه النظرية تسمح لنا حتى بحساب البنية الهايبر-دقيقة لذرة الهيدروجين.
-نظرية الاضطراب المتعلقة بالزمن حتى يمكننا حساب كيفية امتصاص و ارسال الاشعاع من الذرة -الذى هو اساس ما يسمى التفاعل الكهرومغناطيسى بين المادة و الاشعاع-.
انظروا ايضا الى مجموعة الامتحانات فى آخر الكتاب.
فهى تحدى حقيقى عندما كانت الامتحانات امتحانات و كان الطلبة طلبة يقبلون التحدى و اذا ربحوا فرحوا و اذا خسروا فانهم يعيدون الكرة.
مثلا الامتحان الثالث هو امتحان منزلى قدمت فيه تجربة ال EPR و مبرهنة Bell بعد ان منهجت الفيزياء و الرياضيات بالكامل.
ورغم ان الطلبة أخذوا 15 يوم فى عطلة الشتاء فى المنزل و الامتحان معهم لكن لا احد استطاع الاجابة و الاجابة موجودة فى كل مكان من الواب و الكتب.
انظر مثلا الامتحان الرابع الذى يحتوى على تصادمات و من لم يفهم التصادمات فى الميكانيك الكلاسيكى فهو لن يفهمها فى الميكانيك الكمومى و من لم يفهم التصادمات فى الميكانيك الكمومى فهو لن يفهمها فى نظرية الحقول الكمومية (اذن تيقنوا من هذا الامر. و اذهبوا فى هذا الامر خطوة خطوة و بهدوء. هذا الامر هو ما يسمى مصفوفة التصادم فى نظرية الحقول و هو موضوع محورى و بعضهم -مثلا واينبرغ- قال ان نظرية الحقل بأجمعها هى مصفوفة تصادم. شخصيا لا اتفق مع هذا الرأى لكن هذا واينبرغ و ليس شخصا آخر).
انظروا ايضا الامتحان النهائى الذى يحتوى على زهرة من كل بستان.
امتحان الاستدراك كان هدية.
اما امتحان العام الموالى فقد كان تحديا حقيقيا لكنه فى المستوى لكن لم يستطع ان يجيب فيه اى احد رغم اننى اؤكد انه فى المتناول تماما. ليس تعجيزيا ابدا. امتحان مبرهنة بال قد يكون تعجيزيا و لهذا اعطيته فى البيت و مع هذا كان تعجيزيا.
هذه المحاضرات تصلح جدا جدا ان تنشر ككتاب و قد نسيتها تماما بسبب المشاغل الكثيرة.


الهزاز التوافقى على الشبكة

لا يوجد ابسط من مسألة الهزاز التوافقى harmonic oscillator problem فى الفيزياء.

بل ان هذه المسألة تقع فى قلب الميكانيك (بكل انواعه كلاسيكى و احصائى و نسبى و كمومى).
وتذكروا ايضا ان الفيزياء هى ميكانيك ليس شيئا آخر. و كل معضلات الميكانيك الكمومى هى مقاومة هذا الاخير فى ان يكون ميكانيك و رغبته الجامحة الى تحوله الى علم نفس و علم وعى. وهذا ما يرفضه الفيزيائيون و ترفضه الفيزياء.
اذن مسألة الهزاز تقع فى قلب الميكانيك. و النموذج الفيزيائى هو النواس البسيط simple pendulum او النابض المرن elastic spring.
بل اننى اؤكد لكم ان مسألة الهزاز التوافقى تقع فى قلب الطبيعة و الكون و العالم المادى.
فكل شيء (اضطرابى perturbative) فى الكون هو فى المحصلة اضطرابات حول هزاز توافقى بشكل او بآخر. الاستثناء هو الظواهر غير-الاضطرابية non-perturbative فى الطبيعة التى رغم اهميتها القصوى الا انها ظواهر تقع فى صف الاقلية بالمقارنة مع الظواهر الاضطرابية.
الجميع يعرف ايضا كيف يتم حل معضلة الهزاز التوافقى فى الميكانيك الكلاسكيى و فى الميكانيك الكمومى بالخصوص.
لكن اتوقع ان القليل هم من نظر الى حل معضلة الهزاز التوافقى على الشبكة lattice harmonic oscillator (اى هزاز توافقى فى زمن متقطع discrete دورى periodic حرارى thermal و ليس فى زمن مستمر continuous لانهائى infinite نسبى relativistic).
اقدم حل هذه المسألة هنا.
ثم اقدم حل هذه المسألة باستخدام خوارزمية ميتروبوليس Metropolis algorithm.
ثم باستخدام خوارزمية مونتى كارلو الهجين hybrid Monte Carlo algorithm.
هنا لدينا حل تحليلى مضبوط exact analytical solution على الشبكة اذن جميع المحاكيات العددية numerical simulations يجب ان تعطى هذه النتائج النظرية.
هذه المسألة تستخدم فى الحقيقة لموازنة calibration خوارزميات ميتروبوليس و مونتى كارلو الهجينة.

الميكانيك الكمومى الشبكى و الثقالة الكمومية المصفوفية

 

المحاضرة 4


نقدم المحاضرة الرابعة فى موضوع (الميكانيك الكمومى الشبكى و الثقالة الكمومية المصفوفية Lattice Quantum Mechanics and Matrix Quantum Gravity).
هذا موضوع ملموس و تطبيقى حول اعظم انجازات الفيزياء النظرية العميقة فى ال 20 سنة الاخيرة (اى موضوع الثنائية الثقالية-المعيارية gauge-gravity duality).
و هو ايضا موضوع فى متناول الجميع بعد الاجتهاد و الصبر و الالتزام رغم انه يخص احد اعماق الفيزياء النظرية و اكثر من هذا فهو موضوع يتزايد الاهتمام به بصورة مستمرة بل اننى اراهن على انه موضوع مستقبلى (واقصد به المستقبل القريب و ليس البعيد).
هذا الموضوع يتم فيه المزج بين افكار و فيزياء و رياضيات نظرية الاوتار الممتازة superstring theory و افكار و طرق و حاسوبيات نظرية الحقول على الشبكة lattice field theory.
نظرية الاوتار الممتازة فى حد ذاتها هى مزج هائل بين نظرية الحقول الكمومية quantum field theory من جهة و نظرية النسبية العامة general relativity من جهة اخرى.
نظرية الحقول الكمومية هى ميكانيك كمومى quantum mechanics معمق جدا.
اما نظرية الحقول على الشبكة فهى دراسة نظرية الحقول الكمومية بشكل غير-اضطرابى non-perturbative و بالخصوص عبر الطرق العددية numericals و طرق مونتى كارلو Monte Carlo.
فى هذه المحاضرة نقدم نموذج ال BFSS الذى حوله تدور كل هذه المحاضرات.
اول نناقش الثقالة الممتازة supergravity فى 11 بعد و علاقتها بالنظرية M (التى توحد النظريات الوترية الممتازة الخمسة) و علاقة هذه الاخيرة بنموذج ال BFSS.
سنبين ان نموذج ال BFSS هو فريد من نوعه (وهو فعلا كذلك) حيث يصف ثلاثة اشياء مختلفة:
-النظرية M اى تكميم quantization الجسيمات الممتازة superpartciles.
-نظرية الاوتار الممتازة من النوع IIA اى البراينات (مفرد براين brane) D0 و الثنائية الثقالية-المعيارية.
-تكميم الاغشية النسبية الممتازة super relativistic membranes و بالخصوص الغشاء M2 الذى هو محور النظرية M و نظرية الثقالة الممتازة فى 11 بعد.
فى هذه المحاضرة سنقوم ايضا باشتقاق هذا النموذج من الاختزال البعدى dimensional reduction. سنبين صراحة كيف ينزل هذا النموذج من النظرية المعيارية يانغ-ميلز الممتازة supersymmetric Yang-Mills gauge theory فى 10 ابعاد.
ثم نقوم بالتثبيت المعيارى gauge-fixing لهذا النموذج على الشبكة.
ثم نناقش مؤثر ديراك Dirac operator الخاص بهذا النموذج. هنا نقدم نبذة سريعة عن مصفوفات ديراك فى الابعاد المختلفة.
نقدم ايضا وصف لخوارزمية المونتى كارلو الهجينة hybrid Monte Carlo algorithm مطبقة على هذا النموذج فى التقريب البوزونى bosonic approximation اين نهمل مؤثر ديراك.
نقدم ايضا نبذة عن خواص الفافيان pfaffian (و طويلة الفافيان هى الى حد ما الجذر التربيعى للمحدد determinant) الخاص بمؤثر ديراك.
هذه المحاضرة الرابعة يمكن تحميلها من الرابط اين ستجدون ايضا المحاضرات الثلاثة السابقة و قائمة بالمراجع و فهرس المحتويات.
المحاضرة 5

المحاضرة الخامسة فى موضوع -الميكانيك الكمومى على الشبكة و الثقالة الكمومية المصفوفية-

هذا الموضوع هو مثال عن الثنائية الثقالية-المعيارية حيث ان الميكانيك الكمومى المصفوفى يقوم بدور -وهو فعلا كذلك- النظرية المعيارية.
اما الشبكة فهى تقدم لنا تعريف غير-اضطرابى للثقالة الكمومية.
نحن عندما ننطلق فى الفيزياء النظرية نبدأ بالميكانيك الكمومى ثم نظرية الحقول الكمومية ثم نظرية النسبية العامة ثم نظرية الاوتار الممتازة وهذا من اجل الوصول الى الثقالة الكمومية -التى هى نظرية كل شيء-.
الشيء المذهل بخصوص الثنائية الثقالية-المعيارية فى شكل الميكانيك الكمومى المصفوفى هى انها تقول لنا اننا سنجد الثقالة الكمومية فى الميكانيك الكمومى نفسه -اى فى الخطوة الاولى- و لا نحتاج الى الذذهاب الى نظرية الحقول الكمومية و لا الى النسبية العامة و لا الى نظرية الاوتار الممتازة.
فى هذه المحاضرة سنحاول الاجابة عن السؤال -لماذا نريد ان نقوم بالميكانيك الكمومى المصفوفى على الشبكة- ماهى الاسباب و المحفزات و الدواعى الفيزيائية التى تدفعنا الى ذلك.
اقدم ستة دوافع فيزيائية اساسية:
-من نظرية الثقالة الكمومية -الميكانيك الكمومى المصفوفى يقبل ثنائية ثقالية gravity dual هى عبارة عن ثقب اسود من النوع IIA يسمى البراين الاسود black brane. و بالتالى فان النظرية الكمومية حول هذا البراين الاسود تساوى تماما هذا الميكانيك الكمومى المصفوفى-
-من النسبية العامة و الثقوب السوداء - هذا الميكانيك الكمومى المصفوفى يسمح لنا بحساب تأثير هاوكينغ Hawking اى اشعاع الثقب الاسود اعلاه اى اشعاع البراين الاسود و نجد انه ليس هناك ضياع للمعلومات. هذا حل معضلة ضياع المعلومات information loss فى الثقب الاسود و فى نفس الوقت هو تحقق مباشر من الثنائية الثقالية-المعيارية-.
-من الكوسمولوجيا -هذا الميكانيك الكمومى المصفوفى يحتوى على كوسمولوجيا توسعية expanding cosmology مثل التى تميز الكون الذى نعيش فيه. هذه الكوسمولوجيا المصفوفية تقبل التكميم لانها ضمن نطاق نموذج ميكانيك كمومى. اذن دالة موجة الكون يمكن حسابها بشكل مباشر على الاقل من الناحية المبدأية. بصفة عامة هذا الميكانيك الكمومى يحتوى ايضا على الهندسة المنبعثة emergent geometry. و لان الاختزال البعدى لهذا الميكانيك الكمومى المصفوفى هو النموذج المصفوفى IKKT فان هناك ايضا امكانية من اجل انبعاث الزمن -.
-من نظرية الحقول الكمومية و فيزياء الجسيمات -هذا الميكانيك الكمومى المصفوفى يسمح لنا بدراسة التحول الطورى phase transition من المادة الهايدرونية hadronic matter النووية الى المادة الكواركية quark matter البلازمية المسمى تحول هايجدورن Hagedorn. اى متى تتبخر الجسيمات الهايدرونية وتتحول الى كواركات حرة. اذن هو يسمح لنا بدراسة مباشرة لخاصية الحبس confinement وهى الخاصيm الاساسية المميزة للكروموديناميك الكمومى QCD-.
-من الميكانيك الاحصائى و الترموديناميك -هذا الميكانيك الكمومى المصفوفى يسمح لنا بدراسة التحولات الطورية الطوبولوجية topological transitions من الثقب الاسود الى الوتر الاسود black string. التغير الطوبولوجى topology change هو واحد من اهم خواص الثقالة الكمومية. هذه التحولات الطورية من الثقب الاسود الى الوتر الاسود هى فى الحقيقة تميز الثنائية الثقالية-المعيارية فى بعدين فى درجات الحرارة العليا لكن يمكننا دراستها ايضا عبر الميكانيك الكمومى المصفوفى.
-هذا الميكانيك الكمومى المصفوفى فى التقريب البوززنى bosonic approximation و من اجل عدد ابعاد كبير جدا يتحول الى نظرية غوسيانية Gaussian تعطى بالضبط بمسألة الهزاز الكمومى. هذا يسمح لنا بحساب جميع خواص التحول الطورى من الحبس الى اللاحبس و كذلك خواص التحول الطورى من الثقب الاسود الى الوتر الاسود. لكن كون نظرية معقدة مثل الميكانيك الكمومى المصفوفى تتحول الى نظرية غوسيانية تبقى نقطة عميقة مازالت ابعادها غامضة تذكرنا بفكرة ان الكون رغم كل تعقيده تتحكم فيه ايضا الغوسيانية.
نقدم ايضا فى هذه المحاضرة خوارزميتين. اولا خوارزمية المونتى كارلو الهجينة مطبقة على هذا الميكانيك الكمومى المصفوفى فى التقريب البوززنى. ثم نقدم خوارزمية الخزان الحرارى مطبقة ايضا على التقريب البوزونى.
المحاضرة جاهزة للتحميل من صفحة الكورس على البوابة البحثية.



المحاضرة 6

نقدم هنا جميع نماذج الميكانيك الكمومى المصفوفى المعروفة فى جميع الابعاد.
هذا الميكانيك المصفوفى يانغ-ميلز يعيش فى الابعاد 10 و 6 و 4 و 3 و 2 مع تناظر-ممتاز قصوى و تناظر معيارى موضعى و تناظر دورانى-لورنتزى يتعلق على البعد.
كل نموذج من هذه النماذج يتميز على الاقل بثابت كوسمولوجى واحد يسمى معامل التشويه.
يمكن النظر الى هذه النماذج على انها تقريب لنموذج ال BFSS و تمديده الموجى BMN.
كون هذه النماذج تعيش فى بعد ادنى اذن هى اسهل للدراسة.
نقدم فى هذه المحاضرة بالخصوص البعدين 2 و 3. ونقوم بالتثبيت-المعيارى على الشبكة فى الحالتين.
هذه النماذج موجودة فى الصورة الاولى.
كل نموذج من هذه النماذج يصف ثلاثة اشياء اساسية موجودة فى الصورة الثانية.
-وصف للبراينات D0 و الثنائية الثقالية-المعيارية فى نظرية الاوتار الممتازة.
-وصف للاغشية الكمومية النسبية فى الثقالة الممتازة و النظرية M.
-وصف للجسيمات الممتازة فى الثقالة الممتازة و النظرية M.
نقدم ايضا فى هذه المحاضرة -ولأول مرة فهذا نموذج جديد تماما- الميكانيك الكمومى واس-زومينو المصفوفى.
هذه المحاضرة يمكنكم الآن تحميلها من هنا





 المحاضرة 7
فى هذه المحاضرة نقدم خوارزمية المونتى كارلو الهجينة العقلانية rational hybrid Monte Carlo algorithm بشكل بيداغوجى مع تفصيل فيزيائى و رياضى و حاسوبى كامل.
نقدم مثلا الشفرة الخاصة بالميكانيك الكمومى واس-زومينو المتناظر بامتياز supersymmetric Wess-Zumino quantum mechanics كنموذج نموذجى.
نشرح ايضا خوارزميتى ريماز Remez و التدريج المرافق conjugate gradient بتفصيل كاف.
هذا النموذج هو تحضير قبل الذهاب الى نماذج الميكانيك الكمومى المصفوفى التى تلعب دورا حاسما فى الثقالة الكمومية quantum gravity حسب الثنائية الثقالية-المعيارية gauge-gravity duality و التقابل correspondence الشهير ب AdS/CFT.
من اجل فهم هذه المحاضرة تحتاجون الى المحاضرة رقم 3 بالخصوص.
بهذه المحاضرة نكون قد ختمنا هذا الموضوع الى حد ما.
للاشارة هذا موضوع شائك يطول فيه النقاش لم ينتهى فهناك اشياء كثيرة جدا لم اذكرها.
لكن كمدخل للثنائية الثقالية-المعيارية على مستوى نماذج الميكانيك الكمومى المصفوفى فان هذا الذى قدمته يعتبر كافى جدا.
وبهذا اكون قد اكملت مهمتى التى كلفت بها نفسى فى تقديم دليل تفصيلى بيداغوجى للمهتمين لن تجدونه فى اى مكان يقينا.
مبدأ التكافؤ و تساوى الكتلتين الثقالية و العطالية و الفكرة الاسعد فى حياة اينشتاين


 الالهام فى الفيزياء و بالخصوص الفيزياء النظرية هو حقيقة حقيقية و ليس وهما.

وهذا ما سماه اينشتاين (الفكرة الاسعد فى حياتى the happiest thought of my life).
حيث ان اينشتاين عبر عن لحظة اكتشافه الطريق نحو النسبية العامة بالفكرة الاسعد فى حياته و هذا عندما فهم دور مبدأ التكافؤ equivalence principle فى صياغة نظرية نسبية لقوة الجذب الثقالى.
اذن الالهام فى الفيزياء هو (الفكرة الاسعد فى الحياة).
و الجميع ممن درس الفيزياء النظرية بحق و حقيقة يدرك هذا حتى لو لم يجرب ابدا (الفكرة الاسعد فى الحياة).
فالفيزياء النظرية من هذا الباب مثل الفن و الابداع و هذا الالهام الذى هو اسعد فكرة فى الحياة هو مثل الكشف الصوفى.
وهذا مخصوص جدا فى الفيزياء النظرية اين تتقاطع الرياضيات و الفيزياء و الوعى و الطبيعة و العقل و الخيال و اشياء اخرى كثيرة لا تلتقى فى جل الميادين الاخرى.
اذن الفيزيائى النظرى الملهم هو شيء نادر مثلما ان الصوفى الحقيقى الذى تتكشف له الحجب شيء أندر.
بل ان الامكانية العقلية لهذا الالهام او الفكرة الاسعد فى الحياة هى بالضبط ما جعلنى شخصيا اقبل امكانية الكشف الصوفى عقلا على منوال ما فعل الغزالى منذ قرون الذى انطلق من الامكانية العقلية للكشف الصوفى من اجل الاقرار بامكانية الوحى.
اذن عبقرية العلماء هى تقريب لكشف الصالحين الذى هو تقريب لوحى الانبياء او هكذ افهم شخصيا هذا الامر.
اذن الفكرة الاسعد فى حياة اينشتاين هو الالهام الذى تلقاه من العقل الفعال -على مصطلح ابن سينا- او من اللاوعى -على مصطلح جونغ- عندما ادرك اينشتاين الاهمية القصوى لمبدأ التكافؤ.
مبدأ التكافؤ كان يعرفه غاليليو الذى ادرك ان الحركة المستقيمة المنتظمة لا تصدر عن قوة مثلما ان السكون لا يصدر عن قوة.
ثم جاء الملهم الآخر نيوتن و اكتشف ان مبدأ التكافؤ كما فهمه غاليليو هو ليس الا ما يسمى اليوم القانون الاول لنيوتن.
نيوتن هنا فهم الامر بعمق كبير. لانه ادرك ان كل عمليات الرصد الفيزيائية تتطلب معلم او معالم اسناد reference frames و ان هذه المعالم قد تكون متحركة اذن القوانين الفيزيائية يجب ان تكتب ضرورة بالنسبة الى معالم اسناد مخصوصة تسمى معالم الاسناد العطالية inertial reference frames التى يكون فيها قانون غاليلو صالحا.
بعبارة اخرى ادرك نيوتن انه لا يمكن تعريف معالم الاسناد العطالية بشكل ذاتى مستقل بل تعريفها لا يمكن الا بالرجوع الى مبدأ غاليلو.
بكل بساطة المعلم العطالى هو المعلم الذى تكون فيه الحركة المستقيمة المنتظمة لا تصدر عن اية قوة. بعبارة اخرى نقول ان المعلم العطالى هو المعلم الذى يطبق فيه قانون نيوتن الاول.
اما المعلم غير-العطالى non-inertial frame فهو المعلم الذى لا يطبق فيه قانون نيوتن الاول. و اهم المعالم العطالية نذكر اولا المعالم الدورانية و نذكر ثانيا المعالم المتسارعة.
و ادرك نيوتن ايضا ان هذه المعالم العطالية يجب ان تتحرك بالنسبة لبعضها البعض بسرعات منتظمة.
وان هذه المعالم العطالية يجب ان تتحرك بسرعة منتظمة بالنسبة الى معلم عطالى ثابت لا يتحرك يسمى الفضاء المطلق absolute space.
نيوتن كان يعلم جيدا ان هذا المعلم المطلق غير موجود على الارض و لهذا فهو يُعرفه بالنسبة للنجوم الثابتة البعيدة عن الارض.
من هذا المعلم المطلق الافتراضى جاءت فكرة الفضاء المطلق و الزمن المطلق او الفضاء-زمن المطلق التى انتقدها ليبنيز بشراسة ثم جاء بعده ماخ Mach و انتقدها اكثر و من نقد ماخ انطلق اينشتاين.
هذا هو مبدأ التكافؤ عند نيوتن.
مبدأ التكافؤ عند اينشتاين انطلق من ملاحظة ان الكتلة العطالية يجب ان تكون مساوية للكتلة الثقالية.
و الكتلة العطالية inertial mass هى الكتلة التى تؤثر عليها الحركة اى الكتلة التى يتم تسريعها اثناء الحركة.
اما الكتلة الثقالية gravitational mass فهى الكتلة التى تؤثر عليها قوة الجذب الثقالى اى هى الكتلة الموزونة.
اذن تساوى هاتين الكتلتين هو تعبير آخر عن مبدأ التكافؤ.
وتساوى هاتين الكتلتين الثقالية و العطالية يعنى ان جميع الاجسام تسقط بنفس الطريقة فى الحقول الثقالية.
وهذا يعنى ايضا ان هناك تكافؤ تام بين الحقل الثقالى و التسارع فكل تسارع يمكن النظر عليه -عبر تغيير المعلم- على انه حقل جذب ثقالى و كل حقل جذب ثقالى يمكن الغاءه عبر تغيير المعلم الى معلم متسارع. وهذا هو لب الالهام او الفكرة الاسعد فى حياة اينشتاين. لان هذه النتيجة هى من اعمق النتائج فى الفيزياء.
هنا كما ترون يدخل النقاش نوعا آخر من المعالم هى المعالم الساقطة سقوطا حرا freely falling frames و هى معالم غير-عطالية.
والمعالم الساقطة سقوطا حرا يمكن تقسيهما الى عدد لانهائى من المعالم العطالية.
وهذا هو محتوى مبدأ التكافؤ باستخدام الهندسة التفاضلية differential geometry عندما نقول ان المتشعب المنحنى curved manifold (وهو فضاء-زمن النسبية العامة) هو فى كل نقطة عبارة عن متشعب مينكوفسكى المسطح flat Minkowski manifold (وهو فضاء-زمن النسبية الخاصة).
اذن المتشعب المنحنى يقابل المعالم الساقطة سقوطا حرا و المتشعب المسطح يقابل المعالم العطالية.
بل ان حركات السقوط الحر فى حقل ثقالى فى النسبية العامة هى ما يقابل الحركات المستقيمة المنتظمة فى النسبية الخاصة و كليهما هو ما نسميه فى اطار الهندسة التفاضلية بالجيوديزيات geodesics التى هى الخطوط المخصوصة فى الهندسة المشكلة للبنية التفاضلية و البنية السببية لمتشعب الفضاء-زمن.
اذن هذا هو مبدأ التكافؤ. فهو ينص على ضرورة تساوى الكتلتين الثقالية و العطالية.
وهذا هو محتوى تجربة غاليليو الاولى فى هذا المضمار.
وهو ايضا محتوى تجربة ايتوفوس Eotvos التى حسمت هذا الامر فى التاريخ.
فى تجربة ايتوفوس نثبت كتلتين A و B الى طرفى ساق صلدة rigid rod معلقة بخيط رفيع طويل الى السقف. انظروا الصورة الاولى.
الكتلتان سوف تشعران بقوة الجذب الثقالى نحو مركز الارض وهى قوة عمودية فى اتجاه نسميه y. هاتان القوتان تتجهان اذن نحو مركز الارض كما هو معروف وهما تتعلقان على الكتلتين الثقاليتين mgA و mgB للكتلتين A و B على التوالى.
لكن تذكروا فان الارض هى ليست معلم عطالى لانها تدور. اذن هى سوف تؤدى الى لوى اى التواء الساق الصلدة حول الخيط الرفيع المعلق الى السقف.
اى ان الكتلتين سوف تشعران ايضا بقوة طاردة مركزية centrifugal force عمودية على محور دوران الارض. هاتان القوتان يمكن تفكيكهما الى مركبة عمودية فى الاتجاه y و مركبة موازية فى اتجاه نسميه x وهما تتعلقان على الكتلتين العطاليتين miA و miB على التوالى.
ركزوا هنا فان المحور العمودى y هو الخيط وهو محور ذاهب الى مركز الارض اما المحور الموازى x فهو بالضبط الساق الصلدة وهو محور عمودى على الخيط و ليس عمودى على محور دوران الارض -انظروا الصورة الثانية-.
هذه التشكيلة تمثل ميزان لوى torsion balance يسمح لنا بقياس القوى الضعيفة جدا عبر قياس عزم اللوى torsion moment الخاص بها حول محور معين -هنا القوة النى نريد قياسها هى قوة اللوى الناجمة عن دوران الارض حول المحور المعطى بالخيط الرفيع-
القوة فى الاتجاه العمودى y متوازنة balanced اى عزم القوة المؤثرة على الجسم A يساوى عزم القوة المؤثرة على الجسم B. انظر المعادلة فى الصورة الثالثة.
القوة فى الاتجاه الموازى x غير متوازنة من الناحية المبدأية لان هناك قوة لوى او لف torsion force تؤدى الى عزم moment نرمز له ب τ ناجمة عن امكانية اختلاف الكتلتين الثقالية و العطالية. انظر المعادلة فى الصورة الرابعة.
تسارع جاذبية الارض نرمز له ب g كالعادة. اما التسارع الطارد المركزى فى الاتجاهين x و y فنرمز له ب gcx و gcy على التوالى.
بتعويض معادلة الصورة الثالثة فى معادلة الصورة الرابعة و باهمال التسارع الطارد المركزى امام تسارع جاذبية الارض فاننا نحصل على المعادلات فى الصورتين الخامسة و السادسة.
من المعادلة الاخيرة فى الصورة السادسة نرى مباشرة ان عزم اللوى او اللف torsion moment ينعدم لو كانت الكتلة الثقالية تساوى الكتلة العطالية.
من الناحية التجريبية فاننا الذى نقيسه فى هذا العصر هو المعامل η بدلالة التسارع و هذا المعامل يقيس مباشرة الفرق بين الكتلتين الثقالية و العطالية. انظر الصورة السابعة.
آخر القياسات التجريبية التى اجريت عام 1994 تعطى النتيجة فى الصورة الثامنة و هى صفر تجريبى بالدقة المتاحة فى هذا العصر.
اذن الكتلة الثقالية تساوى بالفعل الكتلة العطالية و هذا من المبادئ الفيزيائية الأدق تجريبيا وهذا هو بالضبط مبدأ التكافؤ الذى لاحظه اولا غاليليو ثم فهمه نيوتن بشكل فيزيائى عميق ثم فهمه ليبنيز بفهم فلسفى لا يقل عمقا ثم نقده ماخ نقدا هائلا انطلق منه اينشتاين نحو النسبية العامة فى الفكرة الاسعد فى حياته.









كورس الميكانيك الاحصائى 2023


 الميكانيك الاحصائى ماستر فيزياء جميع التخصصات عام 2023

المحاضرات من 1 الى 6 مع سلسلة التمرينات الاولى-الاسابيع الثلاثة الاولى 8 و 15 و 22 اكتوبر-

المحاضرتان 1 و 2

نناقش و بسرعة المبدأ الاول للترموديناميك ثم المبدأ الثانى للترموديناميك.
المبدأ الاول هو انعكاس لمبدأ انحفاظ الطاقة اذن هو قانون لا يتخلف مطلقا فى الطبيعة.
اما المبدأ الثانى فهو انعكاس للميكانيك الاحصائى الذى يحكم الترموديناميك على المستوى الميكروسكوبى.
بالفعل فان هناك تحولات فى الطبيعة لا يمنعها المبدأ الاول للترموديناميك لكننا لا نراها ابدا تقع فى الطبيعة و السبب ان المبدأ الثانى هو الذى يمنعها احتماليا و ليس مبدأيا.
مثلا انتقال الحرارة من البارد الى الساخن هذه تحولات غير ممنوعة الا احتماليا بالمبدأ الثانى فهى ليست محال.
فى المحاضرة الثانية نقدم ايضا برهان على مبرهنتى كارنو و كلوسيوس اللتان تؤسسان للمبدأ الثانى للترموديناميك.
بالفعل فانه كما ان الترموديناميك يهتم بوصف الجمل المتوازنة ماكروسكوبيا فان الميكانيك الاحصائى يهتم بوصف نفس الجمل لكن ميكروسكوبيا.
اذن الترموديناميك هو نهاية -اى حالة خاصة- من الميكانيك الاحصائى.
اذن هما يهتمان بالجمل المتوازنة حراريا-و-ميكانيكيا.
لكن الميكانيك الاحصائى هو اعم من الترموديناميك لانه يقبل التعميم الى الميكانيك الكمومى.
فى الحقيقة فان الميكانيك الاحصائى من هذا الجانب -جانب التعميم نحو الميكانيك الكمومى- هو اعم حتى من النظرية الحركية.
من الجهة الاخرى فان النظرية الحركية و هى نظرية كلاسيكية محضة هى اعم من الميكانيك الاحصائى الكلاسيكى من جهة انها تهتم باللاتوازن اكثر من اهتمامها بالتوازن.
بعض الناس قد يستخدم مصطلح الميكانيك الاحصائى غير-المتوازن للتعبير عن النظرية الحركية و هذا قد يؤدى الى الخلط لان الميكانيك الاحصائى لا يهتم ابدا بالتطور فى الزمن بل هو يهتم بالجمل المتوازنة و يدرسها بشكل قوى جدا لا مزيد عليه من جهة وصفها عبر الطرق الاحصائية و نظرية الاحتمالات.
اذن هناك الترموديناميك و هناك الميكانيك الاحصائى و هناك النظرية الحركية. الأولان يهتمان بالتوازن الكلاسيكى و الكمومى اما الاخير فيهتم باللاتوازن الكلاسيكى. الميكانيك الاحصائى يشترك مع النظرية الحركية فى شيء واحد هو دراسة الجمل من الناحية الميكروسكوبية باستخدام نظرية الاحتمالات. اما الترموديناميك فهو لا يهتم الا بالجانب الماكروسكوبى.

المحاضرات 3 و 4 و 5 و 6 يتم فيها شرح النظرية الحركية للغازات kinetic theory of gases من وجهة نظر الفيزياء النظرية الاساسية.
النظرية الحركية تعنى بكل بساطة محاولة تتبع حركة عدد هائل من الذرات او الجزئيات التى تتصادم بشكل عشوائى مستمر فيما بينها.
اذن نقدم اولا نبذة قصيرة عن كيفية حساب المقاطع التفاضلية للتصادم scattering cross sections التى تهم ايضا الفيزياء الذرية و الفيزياء النووية و فيزياء الجسيمات الاولية.
بعد ذلك نقوم بالاشتقاق التفصيلى لمعادلة بولتزمان Boltzmann equation التى تتحكم فى كيفية تصرف دالة كثافة الاحتمال probability density function فى الزمن.
هنا ستتاح لنا فرصة عرض اهم فرضيات ليس فقط الفيزياء النظرية بل فلسفة الفيزياء وهى فرضية الفوضى الجزيئية hypothesis of molecular chaos التى هى واحدة من عدد كبير من انجازات عملاق الفيزياء الثالث بولتزمان.
هذه الفرضية التى لا يعرف لها برهان تنص بكل بساطة على ان احتمال الحصول على جسيمين بزخمين p1 و p2 داخل حجم dV يساوى الى احتمال الحصول على الجسيم الاول فى الحجم dV بزخم p1 مضروب فى احتمال الحصول على الجسيم الثانى فى الحجم dV بزخم p2.
اذن كما ترون هى فرضية بديهية لكنها تبقى فرضية بدون برهان و هى حاسمة فى معادلة بولتزمان و النظرية الحركية و اكثر من هذا فهى حاسمة فى نقاشات فلسفة الميكانيك الاحصائى و فلسفة الزمن ولهذا فقد تعرض بولتزمان الى نقد شديد بسببها.
معادلة بولتزمان هى ربما من اعقد المعادلات فى الفيزياء فهى معادلة تكاملية-تفاضلية integro-differential equation عبارة عن انعكاس لمبرهنة ليوفيل Liuoville's theorem من الميكانيك الكلاسيكى و كيف تؤثر التصادمات المستمرة بين ذرات الغاز على مبرهنة ليوفيل التى تعبر عن انحفاظ الحجم فى الفضاء الطورى phase space.
بعد ذلك مباشرة نقوم باشتقاق ما يسمى المبرهنة H او H-theorem التى هى واحدة من اعظم مبرهنات الفيزياء حيث انها تحاول ان تفسر المبدأ الثانى للترموديناميك second law of thermodynamics بل اكثر من هذا تحاول ان تفسر السهم فى الزمن arrow of time كما حاول بولتزمان نفسه ذلك.
المبرهنة H تعطى فى الحقيقة الحل لمعادلة بولتزمان وهى التى اقنعت بولتزمان بحقيقة الذرة مما ادى الى حرب كبيرة عليه من قبل الفيزيائيين و الرياضيين و الفلاسفة الذين كانوا يرفضون فكرة الذرة فى ذلك الوقت (وما زال كثير من الفلاسفة يرفضون فكرة الذرة الى غاية يومنا هذا و هذا كله بسبب تأثير ارسطو و افلاطون و كانط و هيوم العميق).
المبرهنة H تصاغ بدلالة الدالة H او H-function التى ادت فيما بعد ببولتزمان الى مفهوم الانطروبى entropy. فالدالة H تسمى ايضا انطروبى بولتزمان Boltzmann's entropy و هى تختلف عن الانطروبى الحرارى thermodynamical entropy و لا تساويه الا عند التوازن.
الانطروبى هو اعظم انجازات بولتزمان وهو الذى يظهر على شاهد قبره و يمكن القول ان الانطروبى هو الذى تسبب فى انتحار بولتزمان فى الاخير بسبب الضغوط الذى تعرض اليها من قبل الجميع بسببه. بل يمكن القول ان الانطروبى هو الذى جعل بولتزمان يؤمن بالذرة و ليس العكس.
فى الفقرة الاخيرة نقوم باستخدام معادلة بولتزمان و المبرهنة H من اجل اشتقاق توزيع بولتزمان-ماكسويل Boltzmann-Maxwell distribution الشهير.
هذا التوزيع يعطينا احتمال ان تتميز ذرة معينة بطاقة حركية معينة اذن هو يسمح لنا بتوزيع جميع الذرات على جميع السرعات.
نقوم فى الاخير باستخدام توزيع بولتزمان-ماكسويل من اجل اشتقاق جميع الترموديناميك ابتداءا من معادلة الحالة equation of state الى الطاقة الداخلية internal energy الى السعة الحرارية specific heat الى المبدأ الاول first law الى الانطروبى entropy الى المبدأ الثانى second law.
نختم بمجموعة من التمارين النموذجية و قد قمنا بحل اغلبها فى القسم وهذا حتى يعلم المتغيبين.

 

المحاضرات من 7 الى 12 مع سلسلة التمرينات الثانية-الاسبوعان 29 اكتوبر و 5 نوفمبر-

فى هذه المحاضرات نناقش المجموعة الميكروقانونية microcanonical ensemble و هى اول المجموعات الاحصائية للميكانيك الاحصائى.
و المجموعة نقصد بها هنا مجموعة من الجمل الفيزيائية المتطابقة ماكروسكوبيا macroscopically identical اى من الناحية العيانية (مثلا هذه الجمل لها نفس الطاقة) لكن متمايزة ميكروسكوبيا microscopically distinct اى من الناحية المجهرية بمعنى ان ذراتها او جسيماتها أو ذراتها تتوزع فى كل منها بشكل مختلف لكن بشكل يعطى نفس القيمة من الطاقة (وهذه التوزيعات المختلفة للذرات تسمى الحالات الميكروسكوبية microscopic states).
المجموعة الميكروقانونية تصف الجمل المعزولة isolated systems اى الجمل ذات الطاقة الثابتة.
وهذا عكس المجموعة القانونية canonical ensemble التى تصف الجمل ذات درجة الحرارة الثابتة و عدد الجسيمات الثابت و المجموعة القانونية الكبرى grand-canonical ensemble التى تصف الجمل ذات درجة الحرارة الثابتة.
وهذه الثلاث مجموعات هى المجموعات الاحصائية الاساسية فى الميكانيك الاحصائى.
لكن ننبه ان استعمال المجموعة القانونية و المجموعة القانونية الكبرى اسهل من استعمال المجموعة الميكروقانونية لانه اسهل ان نثبت درجة الحرارة (عن طريق وضع الجملة فى توازن حرارى مع ما يسمى حمام حرارى heat bath) اذن اسهل ان نثبت درجة الحرارة من ان نتثبت الطاقة الذى يتطلب عزل الجمل تماما عن الوسط الخارجى (الذى يسمى ايضا البيئة environment) الذى هو امر اصعب بكثير.
لكن حتى من الناحية الرياضية فان استعمال القانونية و القانونية الكبرى اسهل من استعمال الميكروقانوينة.
اساس المجموعة القانونية هو دالة التقسيم partition function-التى سنتكلم عنها فى المستقبل ان شاء الله- لكن اساس المجموعة الميكروقانونية فهو عدد الحالات الميكروسكوبية number of microscopic states التى تتميز بطاقة معينة و الذى يتم حساب كل الترموديناميك بل كل الفيزياء انطلاقا منه.
فى المجموعة الميكروقانونية فان الانطروبى entropy هو متناسب بالضبط مع لوغاريتم عدد الحالات الميكروسكوبية microscopic states للجملة اى لوغاريتم عدد التشكيلات المختلفة different configurations التى تتوزع فيها الذرات او الجزيئات المشكلة للغاز بحيث تعطى تلك الطاقة المعينة المميزة للحالة الماكروسكوبية.
هذه العلاقة بين الانطروبى و عدد الحالات الميكروسكوبية هو ما يسمى معادلة بولتزمان للانطروبى Boltzmann's entropy formula.
فى المجموعة الميكروقانونية هذه نعتبر عوض الجملة الفيزيائية الاصلية المميزة بطاقة معينة عدد كبير من الجمل المتطابقة مع هذه الجملة الاصلية اى جمل لها نفس الطاقة لكن تختلف فيما بينها فى الحالة الميكروسكوبية microscopic state التى تحتلها كل جملة من المجموعة.
المجموعة الميكروقانونية تتميز باحتمال منتظم uniform يساوى الى مقلوب عدد الحالات الميكروسكوبية.
اذن احتمال الحصول على اى جملة فى حالة ميكروسكوبية ما هو ثابت وهذا هو نص المسلمة الاولى للميكانيك الاحصائى.
اذن المتوسط فى الزمن لأى مقدار فيزيائى للجملة الاصلية يساوى الى المتوسط على المجموعة لهذا المقدار الفيزيائى و هذا هو نص المسلمة الثانية للميكانيك الاحصائى التى تعرف ايضا بالفرضية الارغودية ergodic hypothesis.
اذن المتوسط فى الزمن (وهو صعب جدا فى الحساب يتطلب تتبع التطور فى الزمن للجملة الفيزيائية) يساوى المتوسط فى المجموعة (وهو اسهل فى الحساب يتطلب اخذ جمل فيزيائية متطابقة فى نفس اللحظة الزمنية) اذا كان عدد عناصر المجموعة كبير جدا. ومن هنا دخل الاحصاء.
هذه النتيجة فى الحقيقة هى احدى معجزات الميكانيك الاحصائى و هذا هو السبب لماذا يتم تحويل نظرية الحقول الكمومية quantum field theory (التى هى تعميم الميكانيك الكمومى نحو النسبية الخاصة) الى نظرية حقول اقليدية Euclidean field theory عندما يراد اعطاء محتوى حاسوبى للحقول الكمومية يمكن للرياضيات و الحاسوبية ان تفهماه بشكل مضبوط.
نظرية الحقول الاقليدية تسمى ايضا نظرية الحقول الاحصائية statistical field theory و كل هذه ليست الا اسماء لان نظرية الحقول الاقليدية او نظرية الحقول الاحصائية هى ليست الا ميكانيك احصائى لا اقل و لا اكثر.
و اننى شخصيا افضل مصطلح الميكانيك الاحصائى على هذه المصطلحات.
فى هذه المحاضرات نقوم بالخصوص بحساب عدد الحالات الميكروسكوبية فى مثال المشاء العشوائى random walker و فى مثال نموذج ايزينغ Ising model فى بعد واحد ثم نقوم بتعريف انطروبى المعلومات entropy of information.
نقوم بعد ذلك باشتقاق معادلة بولتزمان للانطروبى التى تنص على ان انطروبى المعلومات (و يسمى هنا فى اطار الميكانيك الاحصائى بالانطروبى الاحصائى statistical entropy) متناسب مع لوغاريتم عدد الحالات الميكروسكوبية و ثابت التناسب هو بالضبط ثابت بولتزمان Boltzmann's constant.
الانطروبى الاحصائى يصبح الانطروبى الترموديناميكى المعروف عندما نذهب الى التوازن equilibrium.
الانطروبى او عدد الحالات الميكروسكوبية يقيسان درجة اللانظام disorder فى الجملة.
نقوم بعد ذلك بمناقشة التوازن الاحصائى الذى يتميز بعدد اعظمى من الحالات الميكروسكوبية او قيمة اعظمية للانطروبى او درجة لانظام اعظمية.
عند التوازن الاحصائى فان احتمال الحصول على اى حالة ميكروسكوبية هو ثابت يساوى مقلوب عدد الحالات الميكروسكوبية.
نناقش بعد ذلك التوازن الترموديناميكى و نعطى الوصفة العامة لكيفية استخراج الترموديناميك من المجموعة الميكروقانونية.
ثم نقوم فى الاخير بتطبيق المجموعة الميكروقانونية على جملة الغاز المثالى ideal gas.
كل هذه الامور تجدونها مشروحة بشكل فيزيائى-نظرى و رياضى فى الفصل الثانى.
المسائل و التمرينات فى آخر المحاضرات مرفقة بحلول و سنناقشها فى منشور منفصل ان شاء الله لانه مازلنا لم نقم بحلها فى القسم و ايضا لان بعض تلك المسائل هى فى غاية الاهمية.

تناقض جيبس

 السر الأكبر وراء نجاح الفيزياء الساحق فى كشف اسرار الطبيعة هو التزامها الصارم بمحاولة فهم الكون على شكل ميكانيكى.

و دراسة الفيزياء و الرياضيات مع عدم خلط تلك الدراسة مطلقا بأى شيء آخر هو الضامن الوحيد على الحفاظ على نظافة التفكير الفيزيائى رغم عجز هذا النمط من التفكير عن تناول امور أخرى ليس لها حل على التحقيق.
فالفيزياء هى ميكانيك وليست شيئا آخر.
ولهذا فان مواضيع الوعى و الوجود -التى ليس لها حل على التحقيق- تجد فيها الفيزياء و غيرها من العلوم صعوبة شديدة لأن الوعى على ما يبدو ليس ميكانيكى.
اذن الأسلم هو دراسة الفيزياء و الرياضيات و الحاسوبية و غيرها من العلوم الميكانيكية-الرياضية الصارمة بدون اى خلط مع العلوم الاخرى لان كل شيء آخر لن يجدى شيئا و لن يصل الى اى شيء.
اذن الفيزياء هى ميكانيك و لهذا فان الفيزياء ناجحة فى مهمتها و فى موضوعها نجاحا لم يسبقها فيه اى علم آخر لا قبل و لا بعد.
و أنواع الميكانيك فى الفيزياء و الفيزياء النظرية هى الميكانيك الكلاسيكى classical mechanics و الميكانيك الاحصائى statistical mechanics و الميكانيك الكمومى quantum mechanics.
واى شيء فى الفيزياء بل ان كل ظاهرة فى الطبيعة يجب ان ننجح فى وصفها بأحد انواع الميكانيك هذه.
حتى نظرية كل شيء theory of everything التى يلهث وراءها الجميع فهى ليست الا نظرية ميكانيك كمومى.
اذن الميكانيك الكمومى هو الأساس.
و الميكانيك الكلاسيكى هو المنطق الرياضى الذى يريد ان يحتذى به لكن يفتقده الميكانيك الكمومى.
و أما الميكانيك الاحصائى فهو المنطق الحاسوبي الذى يلجأ له الميكانيك الكمومى حتى يقوم بالتجربة الافتراضية.
اليوم سأتكلم عن الميكانيك الاحصائى.
و سادة و كبراء الميكانيك الاحصائى من الناحية النظرية الاساسية هم ثلاثة: بولتزمان Boltzmann النمساوى و ماكسويل Maxwell الانجليزى و جيبس Gibbs الامريكى.
ومن الأزمات التى صادمت الميكانيك الاحصائى فى بداياته اذكر ما يسمى تناقض جيبس Gibbs paradox. فهذا تناقض كان اساسيا فى اعطاء الزخم التفسيرى للمجموعة الميكروقانونية micro-canonical ensemble و هى اول المجموعات الاحصائية للميكانيك الاحصائى.
المجموعة الميكروقانونية تصف الجمل المعزولة isolated systems اى الجمل ذات الطاقة الثابتة.
وفى هذه المجموعة نعتبر عوض الجملة الفيزيائية الاصلية المميزة بطاقة معينة عدد كبير من الجمل المتطابقة مع هذه الجملة الاصلية اى جمل لها نفس الطاقة لكن تختلف فيما بينها فى الحالة الميكروسكوبية microscopic state التى تحتلها كل جملة من المجموعة.
المجموعة الميكروقانونية تتميز باحتمال منتظم uniform يساوى الى مقلوب عدد الحالات الميكروسكوبية.
اذن احتمال الحصول على اى جملة فى حالة ميكروسكوبية ما هو ثابت وهذا هو نص المسلمة الاولى للميكانيك الاحصائى.
اذن المتوسط فى الزمن لأى مقدار فيزيائى للجملة الاصلية يساوى الى المتوسط على المجموعة لهذا المقدار الفيزيائى و هذا هو نص المسلمة الثانية للميكانيك الاحصائى التى تعرف ايضا بالفرضية الارغودية ergodic hypothesis.
تناقض جيبس Gibbs paradox هو فى الحقيقة تجربة يستحيل تفسيرها فى اطار نظرية الميكانيك الاحصائى الكلاسيكى لكن جيبس وجد لها تفسيرا من خارج هذه النظرية بالذهاب الى الخواص الكمومية للجزيئات و الذرات و الجسيمات قبل اكتشاف الميكانيك الكمومى بأكثر من 40 سنة.
فى هذه التجربة او التناقض نقوم بأخذ غازين مثاليين ideal gases مثلا الارغون و النيون و نقوم بوضعهما فى وعاء و نفصل بينهما بجدار.
نحسب بعد ذلك الانطروبى entropy الخاص بالجملة باستخدام علاقة تسمى ساكور-تترود Sakur-Tetrode relation.
فى المجموعة الميكروقانونية فان الانطروبى هو متناسب مع لوغاريتم عدد الحالات الميكروسكوبية microscopic states للجملة اى لوغاريتم عدد التشكيلات المختلفة different configurations التى تتوزع فيها الذرات او الجزيئات المشكلة للغاز.
هذه العلاقة او المعادلة (الانطروبى متناسب مع لوغاريتم عدد الحالات) تسمى معادلة بولتزمان و هى المعادلة التى اوصى بولتزمان -الذى انتحر فى المشفى بعد انهيار نفسى- بأن توضع على شاهد قبره.
اذن نحسب انطروبى غاز الارغون و انطروبى غاز النيون و نقوم بجمعهما. هذه القيمة الابتدائية.
بعد ذلك نقوم بنزع الجدار الفاصل بين الحجرتين اى بين غازى الارغون و النيون و نعيد حساب الانطروبى باستخدام نفس العلاقة. هذه هى القيمة النهائية.
نجد مباشرة ان التغير فى الانطروبى لا ينعدم لان هذا تحول غير-عكسى irreversible process اى انه لا يمككنا الرجوع الى الحالة الابتدائية التى كانت عليها الجملة قبل نزع الجدار الفاصل بين الغازين.
هذه النتيجة مؤكدة تجريبيا. اذن لحد الساعة لا توجد اى مشكلة.
نفترض الآن ان الغازين المثاليان هما من نفس النوع.
اذا قمنا بحساب التغير فى الانطروبى باتباع نفس الخطوات اعلاه فاننا سنجد ان هذا التغير لا ينعدم ايضا.
وهذه كانت كارثة بجميع المقاييس التجريبية و النظرية.
لانه بكل بساطة اذا كان الغازان فى الحجرتين متطابقين -اى نفس الغاز- فان نزع الجدار الفاصل لا يؤدى الى اى تغيير محسوس فى الجملة. فى هذه الحالة نقول ان التحول عكسى reversible process و اذن كان يجب ان يكون التغير فى الانطروبى معدوما.
اكثر من هذا فان جيبس انتبه الى انه فى هذه الحالة كان يمكن ان نتصور انه كان موجودا اكثر من جدار فاصل فى الوعاء اى ان الوعاء يتشكل من اكثر من حجرتين و كلما نزعنا جدارا بين حجرتين فان الانطروبى يزداد و اذن فان الانطروبى يمكنه ان يأخذ نظريا اى قيمة عددية.
بكل بساطة يستنتج جيبس انه لا يمكن تعريف الانطروبى فى هذه الحالة و كل هذا خطأ لاننا نعرف ان هذا التحول الترموديناميكى هو عكسى و اذن لا يوجد تغير فى الانطروبى.
اكثر من هذا فان الانطروبى مُعرف تماما بعلاقة بولتزمان التى تنص على ان الانطروبى فى المجموعة الميكروقانونية يجب ان يكون متناسبا مع لوغاريتم عدد الحالات الميكروسكوبية للجملة.
هنا يأتى جيبس بحل عبقرى ينص على انه يجب ان نقسم عدد الحالات الميكروسكوبية على عدد تبديلات number of permutations ذرات الغاز التى لا تغير من حالة الجملة و ان العدد الذى نحصل عليه هو العدد الصحيح و هذا العدد هو ما يسمى تعداد بولتزمان الصحيح correct Boltzmann counting.
اذن اذا كان الغازان متمايزين distinct -مثلا ارغون و نيون- و كان الارغون يحتوى على N1 جسيم و كان النيون يحتوى على N2 جسيم فان عدد تبديلات الجزيئات التى لا تغير من حالة الجملة هو N1!N2! اى معاملى N1 ضرب معاملى N2.
هذا واضح لان ذرات الارغون متطابقة identical اذن معاملى factorial عدد ذرات الارغون هو عدد التبديلات الخاصة بالارغون التى لا تغير من حالة الجملة وهذه هى ال N1!.
بالمثل فان ذرات النيون هى متطابقة identical و اذن معاملى factorial عدد ذرات النيون هو عدد التبديلات الخاصة بالنيون التى لا تغير من حالة الجملة وهذه هى ال N2!.
هذا من جهة.
اما من الجهة الاخرى فانه اذا كان الغازان متطابقين -مثلا ارغون و ارغون- فان عدد التبديلات يصبح (N1+N2)! اى معاملى المجموع.
اذن عدد التبديلات مختلف جدا فى الحالتين و هذا العدد يجب ان نقسم به عدد الحالات الميكروسكوبية لان هذه التبديلات لا تغير من الحالة الميكروسكوبية للجملة و اختلاف عدد التبديلات فى حالة الغازات المتمايزة و حالة الغازات المتطابقة يؤدى بالضبط الى النتيجة الصحيحة فى كلتا الحالتين.
فمثلا فى حالة الغازين المتطابقين فان هذه القاعدة -التى تسمى تعداد بولتزمان الصحيح- تؤدى الى تغير فى الانطروبى معدوم و هذا هو المشاهد تجريبيا فى هذه الحالة.
اذن جيبس العبقرى حل هذه المعضلة باستخدام خاصية يستحيل ان تكون صحيحة فى الميكانيك الاحصائى الكلاسيكى.
فالجسيمات المتطابقة identical particles غير موجودة فى الميكانيك الكلاسيكى و اما فى الميكانيك الكمومى فهى موجودة مع خاصية ان دالة الموجة الخاصة بها يجب ان تكون اما تناظرية symmetric (اذا كانت الجسيمات ذات عزم لف صحيح integer spin) او تناظرية-ضدية anti-symmetric (اذا كانت الجسيمات ذات عزم لف نصف صحيح half-integer spin).
هذه الخاصية الكمومية هى ذات علاقة بمبرهنة السبين-و-الاحصاء spin-statistic theorem و مبدأ الاستبعاد لباولى Pauli's exclusion principle التى هى ظواهر كمومية محضة ذات اصل طوبولوجى topological.
اذن جيبس اكتشف معضلة فى فيزياء الغازات ثم حل تلك المعضلة باستخدام خواص كمومية قبل اكتشاف الميكانيك الكمومى باكثر من 40 سنة.
حل هذه المعضلة يسمى تعداد بولتزمان الصحيح لان بولتزمان عندما قام بحل هذه المسألة حلها بشكل صحيح من البداية و لم يقع فى اى معضلة و هذا بكل بساطة لان بولتزمان هو اكثر ذكاءا و عبقرية و عمقا بسنوات ضوئية من جيبس.



الثقوب السوداء هى ترموديناميك

فى السبعينات انتبه الفيزيائيون الى التشابه الكبيربين الثقوب السوداء من جهة و بين الترموديناميك من جهة ثانية.

فمثلا من اجل ثقب اسود كتلته M دوار بسرعة زاوية Ω فان العلاقة بين M و Ω من جهة و العزم الحركى J للثقب الاسود و المساحة A الخاصة بسطح افق الحدث event horizon من جهة اخرى و العلاقة بين كل هذه الامور و ما يسمى ثقالة السطح surface gravity الخاصة بالثقب الاسود التى يرمز لها ب κ تعطى بالمعادلة فى الصورة.
لاحظوا فان التغير فى العزم الحركى مرفق بالسرعة الزاوية لاننا نعلم من الميكانيك الكلاسيكى ان هذين المتغيران هما متغيران مترفقان conjugate variables.
لكن لاحظوا ايضا ان التغير فى مساحة افق الحدث هو مرفق بثقالة سطح افق الحدث. اذن نستنتج ان هذين المتغيران يجب ان يكونا مترافقين.
فى نفس الفترة اى فى السبيعنات اكتشف هاوكينغ Hawking ان الثقب الاسود يصدر عنه اشعاع حرارى thermal radiation عند درجة حرارة تعرف اليوم باسم درجة حرارة هاوكينغ Hawking temperature التى وجدها هاوكينغ متناسبه تماما مع ثقالة سطح افق الحدث. انظر المعادلة الثانية فى الصورة.
اذن مساحة سطح افق الحدث هى فى الحقيقة متغير مرافق لدرجة الحرارة.
ونجن نعرف من الترموديناميك ان المتغير المرافق لدرجة الحرارة هو بالضبط الانطروبى entropy.
من هنا استنتج بكننشتاين Bekenstein فى السبيعنات مرة اخرى و باستخدام المعادلتين السابقتين اهم قانون على الاطلاق الى غاية يومنا هذا يخص الثقوب السوداء: انطروبى الثقب الاسود متناسب مباشرة مع مساحة افق الحدث. و هذا القانون يعنى مما يعنى ان القانون الذى يحكم الثقوب السوداء (مثلا الدوارة المعطاة فى الضورة الاولى) هو ليس الا القانون الاول للترموديناميك first law of thermodynamics حيث تلعب الكتلة دور الطاقة الداخلية internal energy و يلعب الحد الذى يحتوى على السرعة الزاوية و العزم الحركى دور العمل work الذى ينطوى فى الصور العادية للترموديناميك على الحجم و الضغط. انظر الصورة الثالثة.
قانون بكنشتاين-هاوكينغ هو قانون صالح من اجل جميع الثقوب السوداء وهو يعطى انطروبى الثقب الاسود على انه مجموع حدين. الحد الاول ينص على ان انطروبى الثقب الاسود متناسب مع مساحة سطح افق الحدث وهذا هو اهم حد على الاطلاق. اما الحد الثانى فهم يضم انطروبى الحقول الكمومية الموجودة فى الحقل الثقالى للثقب الاسود وهذا يسمى عموما الانطروبى الخارجى. انظر الصورة الرابعة.
اكثر من هذا فانه فى السبيعنات كان الفيزيائيون يعرفون قانون ثان خاص بالثقوب السوداء و الذى ينص على ان مساحة سطح افق الحدث A لا يمكن الا ان تزيد فى الزمن.
ولاننا نعرف من قانون بكنشتاين ان المساحة متناسبة مع الانطروبى فان قانون الازدياد المضطرد لمساحة الثقب الاسود مع الزمن يتحول الى قانون الازدياد المضطرد لانطروبى الثقب الاسود مع الزمن. وهذا هو بالضبط القانون الثانى للترموديناميك second law of thermodynamics كما يعرف الجميع. انظر الصورة الخامسة.
ولهذا فان الثقوب السوداء مكافئة تماما للترموديناميك او بعبارة اخرى لماذا يجب ان تكون النسبية العامة مكافئة للترموديناميك و ليس للميكانيك الكمومى كما يصر الجميع.







تبخر الثقب الاسود و اشعاع هاوكينغ


 الثقوب السوداء تتبخر كما بين ذلك هاوكينغ Hawking فى السبعينات.

اشعاع هاوكينغ Hawking radiation هو عملية تخلق زوج pair creation من الجسيمات الافتراضية virtual particles بالقرب من افق الحدث event horizon حيث ان احد الزوجين يسقط فى المفردة singularity -مفردة الثقب الاسود- اما الزوج الآخر فينتشر الى المالانهاية infinity حيث يوجد الراصد observer-اى نحن-.
هذا الزوج متشابك كموميا quantum entangled مما يعنى ان هناك ضياع للتشابك الكمومى او المعلومات فى المفردة.
اذن هذه المعضلة الصعبة جدا المعروفة تحت اسم -معضلة ضياع المعلومات فى الثقب الاسود black hole information loss paradox- تتلخص فى وجود افق الحدث الذى يفرض علينا تقسيم الحالة النقية pure state للجملة -الثقب الاسود هو ليس الا نجم قد انهار على نفسه- الى جزأين مما يؤدى الى ظهور اشعاع هاوكينغ بالنسبة لنا و كأنه اشعاع حرارى اى على شكل حالة مختلطة mixed state مثل اشعاع الجسم-الاسود blackbody radiation و هو ليس كذلك.
و لا يمكن ابدا -اى لا توجد اى قوة فى الطبيعة- يمكنها ان تأخذ حالة نقية الى حالة مختلطة. الاستثناء الآخر هو معضلة الرصد الكمومى quantum measurement problem وهى اعقد بكثير من هذه المعضلة.
يمكن شرح هذه المعضلة -معضلة ضياع المعلومات فى الثقب الاسود- من وجهة نظر اشعاع هاوكنيغ كما يلى.
نحن نعرف ان انطروبى entropy الثقب الاسود متناسب مع مساحة area سطح افق الحدث الذى يحيط بالثقب الاسود (قانون بكنشتاين-هاوكينغ Bekenstein-Hawking law).
هذا الانطروبى هو انطروبى ما يسمى التحبيب-الخشن coarse-grained (بعبارة اخرى هو الانطروبى الذى نستخدمه فى الترمودينامينك اى انطروبى بولتزمان Boltzamnn المتناسب مع عدد الحالات الميكروسكوبية microscopic states للجملة).

لكن الثقب الاسود يتبخر اذن الانطروبى الذى يصفه هو فى الحقيقة انطروبى فون نيومان von Neumann او ما يسمى انطروبى التحبيب-الدقيق fine-grained وهو الانطروبى الذى نستخدمه فى الميكانيك الكمومى لقياس التشابك الكمومى quantum entanglement و كمية المعلومات information.
انطروبى فون نيومان دائما اقل من انطروبى بولتزمان. اى ان انطروبى فون نيومان الخاص بالثقب الاسود يجب ان يكون دائما اقل من انطروبى بكنشتاين-هاوكينغ المتناسب مع مساحة افق الحدث الذى ابتدأت عنده عملية التبخر.
ولان جملة الثقب الاسود و اشعاع هاوكينغ موصوفة بحالة نقية فان انطروبى فون نيومان الخاص باشعاع هاوكينغ يساوى انطروبى فون نيومان الخاص بالثقب الاسود.
اذن انطروبى فون نيومان الخاص باشعاع هاوكينغ يجب ان يكون دائما اقل من انطروبى بكنشتاين-هاوكينغ الذى ابتدأت به عملية تبخر الثقب الاسود.
لكن عندما يبدأ الثقب الاسود فى التبخر فان انطروبى اشعاع هاوكينغ يبدأ بالازدياد المضطرد و سيتواصل تزايد هذا الانطروبى بدون حد مع تواصل خروج اشعاع هاوكينغ من الثقب الاسود حتى نصل الى اللحظة التى يصبح فيها انطروبى فون نيومان الخاص باشعاع هاوكينغ اكبر من انطروبى بكنشتاين-هاوكينغ وهذا هو المحال او التناقض او المعضلة.
الفرضية الوحيدة الضمنية هنا هو واحدية unitarity الميكانيك الكمومى اى ان معادلة شرودينغر صالحة دائما.
الحل الوحيد من هذا المأزق هو ان يبدأ انطروبى اشعاع هاوكينغ فى التناقص عندما تبلغ قيمته قيمة انطروبى بكنشتاين-هاوكينغ -هذه اللحظة التى يحدث فيها هذا الامر تسمى زمن بايج Page time-.
المطلوب اذن هو ايجاد نظرية يحدث فيها هذا الامر بشكل صريح. فهذا ليس كلام فقط.
النموذج الوحيد المعروف الذى يحدث فيه هذا الامر و لو فى بعض الحالات الخاصة هو الثنائية الثقالية-المعيارية gauge-gravity duality.
فى الصورة الاولى عملية تشكل formation ثقب اسود من انهيار ثقالى نجمى stellar gravitational collapse ثم عملية تبخره evaporation تماما حتى لا يبقى شيء الا اشعاع هاويكنغ فى فضاء-زمن مينكوفسكى Minkowski spacetime المسطح flat.
فى الصورة الثانية مخطط بنروز Penrose diagram المرفق بهذه الفيزياء. تعلم قراءة مخططات بنروز هو امر مهم جدا فى النسبية العامة يسهل علينا فهم كل ما يحدث فى الفضاء-زمن مهما كان معقدا فى شكل بسيط. و تذكروا فان بنروز هو استاذ هاوكينغ و قد تحصل على جائزة نوبل منذ سنتين و هو ما زال على قيد الحياة.






مدخل الى النسبية العامة من الميكانيك الكلاسيكى

تجربة رائدة فى تدريس النسبية العامة لطلبة مستوى الثالثة او الرابعة ليسانس يقدمها الفيزيائى النظرى جايمس هارتل James Hartle احد اصدقاء هاويكنغ القدماء.

الفكرة الاساسية التى ينطلق منها المؤلف هو محاولة اعادة صياغة نظرية الثقالة النيوتونية Newtonian gravity فى قالب هندسى geometric.
اذن هنا الاساس هو ما يعرفه كل فيزيائى نظرى فى أن قوة الجذب الثقالى هى ليست قوة بالمعنى المتآلف عليه بل هى هندسة geometry و بالضبط انحناء الفضاء-زمن spacetime curvature.
اذن المؤلف ينطلق من فكرة ان النسبية الخاصة special relativity وبالتالى فضاء-زمن مينكوفسكى Minkowski spacetime هى ليست الا ميكانيك كلاسيكى.
ثم يحاول المؤلف بعد ذلك صياغة الميكانيك الكلاسيكى لقوة الجذب العام لنيوتن فى اطار هندسة فضاء-زمن النسبية الخاصة التى سوف تصبح هندسة منحنية curved geometry -ولو بشكل تقريبى- بسبب قوة الجذب الثقالى.
وهذا هو الذى جذبنى الى هذا الكتاب.
اذن المؤلف سوف يفترض ان الفضاء-زمن هو فعلا مسطح flat و ان تأثير قوة الثقالة ينطوى فقط فى تغيير الساعات clocks المستعملة لقياس الزمن.
وهذا ينجح بشكل كافى خاصة ان هذا الموضوع يدرس بدون الدخول على الاطلاق فى نظرية النسبية العامة general relativity و الهندسة التفاضلية differential geometry و لا يستخدم فيه الا مبدأ التكافؤ equivalence principle الذى ينص على تساوى الكتلتين الثقالية gravitational و العطالية inertial و بالتالى على تكافؤ التسارع المنتظم و قوة الجذب الثقالى.
فعلا كان تطبيق مبدأ التكافؤ فى اطار الميكانيك الكلاسيكى كافى جدا من اجل الوصول الى ان تأثير قوة الجذب الثقالى على الفضاء-زمن المسطح flat spacetime للنسبية الخاصة يكمن فقط فى تغيير الساعات المستعملة لقياس الزمن عبر الفضاء-زمن -ما نعرفه نحن فى النسبية العامة تحت مسمى تأثير الانزياح نحو الاحمر الثقالى gravitational redshift-.
بطبيعة الحال نحن نعرف ان الصحيح هو ان الفضاء-زمن هو فعلا منحنى و ليس مسطح و ان هندسة ذلك الانحناء هى بالضبط ما نراها نحن على شكل قوة جذب ثقالى. و من هذا الانحناء نحصل على الانزياح نحو الاحمر الثقالى و غيره من التأثيرات الثقالية النسبية.
اذن التصور الذى تبناه المؤلف حتى يصل عبر الميكانيك الكلاسيكى الى النسبية العامة هو تصور خاطئى يمكن تسميته بالفضاء-زمن-المسطح flat-spacetime.
الفرق بين التصورين ان تصور الفضاء-زمن-المسطح (الخاطئ) لا يسمح لنا من الناحية المبدأية بقياس هندسة الفضاء-زمن اما تصور الفضاء-زمن المنحنى (الصحيح) فهو يقدم فرضية يمكننا التحقق منها عبر الرصد و القياس التجريبيان.
وهذا يشبه تصور الارض-المسطحة flat-earth الذى نعرف انه خاطئ رغم انه يؤدى الى كثير من التوقعات الصحيحة (المسافات تبدو اقصر عندما نذهب نحو الشمال لان المساطر التى تسمح لنا بقياس الطول تتمدد بسبب وجود حقل غير-مرئى مسؤول عن ذلك. هكذا يقول اصحاب هذا الطرح). الجميع يعرف ان الصحيح هو ان الارض منحنية.
انظروا الصفحة 125-126 من اجل المقارنة بين تصورى الفضاء-زمن-المسطح و الارض-المسطحة.
اذن كل هذه الامور موجودة فى القسم الاول من الكتاب و فى الفصل الاول -الفصل 6- من القسم الثانى.
وهذا موضوع محاضرة جديدة فى الميكانيك الكلاسيكى انوى تحضيرها ان شاء الله تعنى بهندسة قوة الجذب الثقالى النيوتونى. و التى سوف تسمح لنا بالقفز نحو فكرة الفضاء-زمن المنحنى الاساسية فى النسبية العامة دون الدخول فى النسبية العامة.
من الواضح ان هذه المحاضرة سوف تحتاج ان تسبق بمحاضرة كاملة فى النسبية الخاصة التى هى بالاساس جزء من الميكانيك الكلاسيكى كما ذكرنا آنفا.
واذا تمكنا من تحضير هذه المحاضرة فاننا ننوى ان شاء الله تحضير محاضرة ثالثة جديدة فى الميكانيك الكلاسيكى بخصوص (كوسمولوجيا الكون المرصود) التى لا تحتاج من النسبية العامة الا الى فكرة هندسة الفضاء-زمن المنحنى و فكرة المبدأ الكوسمولوجى cosmological principle -جميع النقاط و جميع الاتجاهات فى الكون متكافئة-.
وقد نضيف محاضرة رابعة لم لا بنفس المنطق بخصوص (ميكانيك الثقوب السوداء) لم لا.
الكتاب يتكون من ثلاثة اقسام و 24 فصل.
القسم الاول من الفضل 1 الى الفصل بعنوان (الفضاء و الزمن فى الفيزياء النيوتونية و النسبية الخاصة).
القسم الثانى من الفصل 6 الى الفصل 19 بعنوان (الفضاءات-زمن المنحنية فى النسبية العامة). و فى هذا القسم يتناول المؤلف الثقوب السوداء و الكون المرصود بالنقاش التفصيلى و هو لا يعرف من النسبية العامة الا فكرتى مبدأ التكافؤ و الفضاء-زمن المنحنى.
القسم الثالث من الفصل 20 الى الفصل 24 بعنوان (معادلات اينشتاين). و فى هذا الفصل يبدأ المؤلف بالفعل بمناقشة نظرية النسبية العامة و اسسها الهندسة التفاضلية و لو بشكل مقتضب.



الهزاز التوافقى المصفوفى

الهزاز التوافقى harmonic oscillator هو أول مسألة تدرس فى الميكانيك الكمومى Quantum Mechanics بعد ان يتم تقديم المفاهيم الاساسية.

لنتصور الآن هزاز توافقى احداثيته x هى مصفوفة X. نحصل بذلك على ما يسمى الهزاز التوافقى المصفوفى matrix harmonic oscillator.
هذه المسألة تتميز بالآتى:
- تناظر معيارى gauge invariance.
- تناظر كونفورمالى conformal invariance.
-تناظرممتاز supersymmetry.
وهى تصف الاغشية-الممتازة super-membranes فى ثلاثة ابعاد.
وتصف البراينات الصفرية D0 branes فى نظرية الاوتار الممتازة superstring theory فى بعدين.
وهى تصف الثقالة الديلاطونية dilaton gravity فى بعدين.
اى انها تصف الثقوب السوداء black holes الكمومية فى فضاء ديى سيتر الضدى anti-de Sitter space فى بعدين.
و اكثر من هذا هى تصف الهندسة غير-التبديلية non-commutative geomery لفضاء دى سيتر الضدى و الثقب الأسود دى سيتر الضدى.
مسألة من ابسط ما يكون لكنها من اغنى ما يمكن.


اقرب الى الحقيقة

اقرب الى الحقيقة او closer to truth هو برنامج امريكى على القناة الامريكية العامة PBS يعنى بثلاثة مواضيع اساسية هى:
-الفيزياء النظرية او الكون cosmos.
-علوم النفس أو الوعى consciousness.
-فلسفة المعنى أو الله God.
وهذه هى المستوبات الوجودية الثلاثة عند جميع فلاسفة الانظمة systems philosophers المؤمنين منهم و غير المؤمنين.
البرنامج يأتى فى صيغة حوارات مع اكبر المختصين فى الفيزياء النظرية و علوم الوعى و الفلسفة.
هناك ايضا قناة يوتوب تأتى بمقتطفات من هذه الحوارات.
من المواضيع التى تناقش فى محور -الكون- نجد مثلا الميكانيك الكمومى, الكوسمولوجيا, الزمن, فلسفة الكمومى و الكوسمولوجيا و الزمن, عديد-الاكوان و متعدد-العوالم, فيزياء الجسيمات و معضلات التوحيد, نظرية كل شيء, الظواهر الكمومية و تفسيرات الميكانيك الكمومى, الانبعاث, التعقيد, المستقبل البعيد, طبيعة الوجود, الميتافيزيقا.
من المواضيع التى تناقش فى محور -الوعى- نجد مثلا المعضلة عقل/جسم, حرية الارادة, معضلة الهوية, معضلة الوعى , اللاوعى و ما تحت الوعى, المرض العقلى, الموت و الحياة بعد الموت, الحياة الفضائية, علاقة العقل بالدماغ, النشاطات العقلية, الوظائف الدماغية, علم النفس, الذطاء الاصطناعى, ماوراء علم النفس.
من المواضيع التى تناقش فى محور -الله- نجد مثلا فلسفة الدين, العلاقة بين العلم و الايمان, علم الكلام الفلسفى, اسماء و صفات الله, علاقة الله بالعالم, حقيقة النبوة و الوحى, الحياة بعد الموت و نهاية العالم, التفكير النقدى فى الدين.

https://closertotruth.com/


حل معضلة ضياع المعلومات فى الثقب: خلاصة

هذا المنشور فى الرابط كان الاول من سلسلة من المنشورات المخطط لها تخص التفصيلات الحسابية التى تخص حل معضلة اشعاع هاوكينغ Hawking radiation و ضياع المعلومات information loss فى الثقب الأسود black holes.

للأسف لم نستطع ان نقدم الا هذا المنشور بسبب المشاغل الكثيرة لكن سوف نحاول ان شاء الله انهاء مناقشة هذا الموضوع الاساسى الذى يقع فى عمق الفيزياء النظرية.
تحتاجون من اجل تحقيق الفهم السليم الى قراءة:
اولا نظرية المجال الكونفورمال conformal field theory.
ثانيا معضلة ضياع المعلومات فى الثقب الاسود فى اطار نظرية النسبية العامة.
ثالثا التقابل AdS/CFT.
هذا الحل لمعضلة ضياع المعلومات فى الثقب الاسود الذى ابتدأه بولشينسكى Polchinski قبل وفاته يتضمن فى الحقيقة المواضيع الخمسة التالية:
-كيفية حساب انطروبى التشابك entanglement entropy فى نظريات المجال الكونفورمال و علاقة ريو-و-تكاياناغى Ruy-Takayanagi formula و تخمينية الجزيرة island conjecture (هذه ايضا مراجعة فى الحقيقة).
-انطروبى التشابك فى بداية تبخر الثقب و حدسية هايدن-براسكل Hyden-Preskill conjecture فى المعلوماتية الكمومية quantum information. وهذا الحساب هو بالاساس نتيجة هاوكينغ التى تحصل عليها فى السبعينات. الجديد هو ربطها بالمعلوماتية الكمومية. وهذا هو الانجاز الاول.
-انطروبى التشابك بعد زمن بايج Page time و خروج المعلومات من الثقب. وهذا هو الانجاز الاساسى لبولشينسكى و احمد المهيرى و زملاءهما.
حيث سيتضح ان اشعاع هاوكينغ مرتبط عبر ابعاد اضافية extra dimensions بما بداخل الثقب وهذا تتفيذ مباشر لحدسية ال ER=EPR الشهيرة لساسكيند Susskind و مالداسينا Maldacena.
-العلاقة بنموذج ساشداف-و-يى-و-كايتاف Sachdev-Ye-Kitaev model (والحكاية قد بدأت من هنا فى الحقيقة).
-الذهاب الى تكامل الطريق الثقالى gravitational path integral بدون تقابل AdS/CFT و التحول الطورى نحو الثقوب الدودية wormhole عند زمن بايج.
فى المنشور فى الرابط -وهو ليس الا مدخل الى هذا الموضوع الشائك- اقدم بطريقتى الخاصة نموذج المهيرى-و-بولشينسكى Polchinski-Almheiri الذى يعتمد عليه الحل وقد قُدم هذا النموذج عام 2014 من طرف احمد المهيرى و جوزيف بولشينسكى فى البحث رقم 1402.63334.
نموذج المهيرى-و-بولشينسكى هو نموذج ثقالة ديلاطونية dilaton gravity فى بعدين بكمون potential معين يسمح لنا بالحصول على فضاء-زمن دى سيتر-العكسى anti de-Sitter spacetime كخلفية مستقرة stable background و هذا بسبب وجود مجال سلمى scalar field فى النظرية وهو الديلاطون dilaton.
اذن هذا النموذج يقبل ثقوبا سوداء مستقرة لكنها متوازنة حراريا لان اشعاع هاوكينغ ينعكس فى حد boundary الفضاء دى سيتر-العكسى و يرجع نحو الثقب.
فضاء دى سيتر-العكسى الذى يرمز له ب AdS يسمح لنا باستخدام التقابل AdS/CFT اى ان الديناميك الثقالى داخل الفضاء AdS يعطى بنظرية ميكانيك كونفورمال conformal quantum mechanics على الحد boundary الذى هو فى هذه الحالة خط يُعبر عن الزمن.
ايضا ننبه انه فى نموذج المهيرى-و-بولشينسكى فان المادة تعطى بنظرية مجال كونفورمال لا تتفاعل مع الديلاطون الا عبر المترية.
نموذج المهيرى-بولشينسكى هو نموذج مرتبط بشكل عميق بنموذج ساشاداف-و-يى-و-كايتاف (الذى يقوم فى الثقوب السوداء بنفس الدور الذى يقوم به نموذج ايزينغ فى التحولات الطورية من الدرجة الثانية).
فنموذج المهيرى -و-بولشينسكى يوصف بنظرية حدية (اى تعيش على الحد) توصف بالمشتقة الشوارزية Schwarzian derivative مثل نموذج ساشداف-و-يى-و-كايتاف.
فى الفقرة الاولى نقوم بحساب هذه الثقوب السوداء (اى المترية و الديلاطون) ثم نقوم بحساب درجة حرارة هاوكينغ.
هذا الثقب الاسود يسمى ثقب اسود خالد eternal لانه حل رياضى لم ينشأ من انهيارات ثقالية gravitational collapses كما تتشكل الثقوب السوداء فى الواقع.
فى الفقرة الثانية نقوم بحل معادلات الحركة و حساب الثقب الاسود المتشكل من انهيار ثقالى عبارة عن سقوط موجة مادية تنطلق من حد الفضاء لتسقط داخل الثقب.
نحسب مرة اخرى طاقة الثقب و درجة حرارة هاوكينغ و العلاقة بينهما.
فى الفقرة الثالثة نبدأ باستخدام البحث 1905.08762 لاصحابه المهيرى و انجلهارت Engelhardt و مارولف Marolf و ماكسفيلد Maxfield وهو البحث الذى ابتدأ فعليا فيه هذا الحل.
اذن فى الفقرة الثالثة نصف كيف يتم تحويل الثقب الاسود فى فضاء دى سيتر-العكسى الى ثقب اسود حقيقى عن طريق لصق gluing فضاء دى سيتر-العكسى بخزان حرارى heat bath عبارة عن نظرية مجال كونفورمال فى بعدين.
اشعاع هاوكينغ يهرب من الثقب و يخرج عبر الحد الى هذا الخزان الحرارى (هذه هى الفكرة الفيزيائية).
اذن الشروط الحدية يتم تغييرها من شروط عاكسة reflecting عند حد الفضاء الى شروط شفافة transparent (تسمح بمرور الاشعاع).
نقوم ايضا بتحديد الحالة الابتدائية للجملة (ثقب اسود دي سيتر-العكسى+ الخزان الحرارى) التى تعطى بتكامل الطريق الاقليدى Euclidean path integral على نصف الخط المستقيم بحد مشوه deformed boundary.
نقوم بحساب الديفيومورفيزم diffeomorphism الذى نرمز له f الذى يسمح لنا باختزال الجملة لنصف الخط المستقيم (هذه نقطة صعبة تحتاج الى الكثير من التأمل و الحساب معا).
نقوم ايضا بحساب التصحيح الكمومى quantum correction لتنسور الضغط-و-الطاقة-و-الزخم stress-energy-momentum tensor للمادة باستخدام الشذة الكونفورمالية conformal anomaly وهنا تدخل الشحنة المركزية central charge لنظرية المجال الكونفورمال و المشتقة الشوارزية للديفيومورفيزم f بالنسبة للزمن t.
ثم نعين التصحيح الكمومى للديلاطون.
فى الفقرة الرابعة نقوم بحساب النظرية الحدية و نجد كما ذكرنا انها تعطى بدلالة المشتقة الشوارزية ثم نحسب التصحيح الكمومى لطاقة الثقب و درجة حرارة هاوكينغ.

الايونات المحبوسة

 كيوبت عبارة عن ايونات محبوسة trapped ions.

الايون المحبوس اما ان يكون فى الحالة المضيئة -الحالة المثارة و تقابل البت 1- او الحالة المظلمة -الحالة الاساسية و تقابل البت 0-.
و بغض النظر عن الحاسوبية الكمومية فان هذه الايونات المحبوسة -وهى من مواضيع الفيزياء الذرية- تعطى تحقيق تجريبى مباشر للقفزات الكمومية quantum jumps التى تنبأ بها بوهر و هايزنبرغ منذ 100 سنة و اعترض عليهما شرودينغر اعتمادا على تفسيره لمعادلته الشهيرة التى تعطى دالة موجة معبرة عن سعة احتمال probability amplitude طويلته modulus هو بالضبط احتمال الانتقال بين الحالات الكمومية.
لكن تخطئة شرودينغر هى فى قدر صعوبة تخطئة بوهر و هايزنبرغز.
اذن الانتقال بين الحالات الكمومية هو فعلا -حسب شرودينغر- عملية تطور مستمرة continuous evolution فى الفضاء-زمن على سلم الازمان الذرية لكن على سلم الازمان البشرية فان ذلك الانتقال يظهر على شكل قفزات متقطعة discrete jumps بين الحالات الكمومية.
مع هذا فاننا نصر مع بوهر ان الميكانيك الكمومى هو عملية متقطعة اكثر منه عملية مستمرة.
اذن القفزات الكمومية -مثلا هنا بين الحالة المضيئة و الحالة المظلمة للكيوبت- هى حقيقة نراها مباشرة على الاقل على سلم الازمان البشرية.
الايون المضيء يعنى ان الايون فى الحالة المثارة. و يصبح هذا الايون مظلما عندما يقفز الى الحالة الاساسية.


حل بولشينسكى لمعضلة ضياع المعلومات فى الثقب

نظرية الثقالة الكمومية theory of quantum gravity هى هدف كل الفيزيائيين النظريين المشتغلين بالفيزياء النظرية العميقة انطلاقا من العلماء العباقرة المتفوقين الذين يسير علمهم و ارادتهم و قدرتهم هذا المجال وصولا الى الباحثين العاديين من امثالنا.
و الثقالة الكمومية تهدف الى التوحيد بين نظريتى النسبية العامة و الحقول الكمومية.
وتذكروا فان نظرية النسبية العامة هى تعميم للنسبية الخاصة نحو هندسة و انحناء الفضاء-زمن.
اما نظرية الحقول الكمومية فهى تعميم للميكانيك الكمومى نحو الحقول النسبية.
و فى الثقافة الشعبية فان نظرية الثقالة الكمومية تسمى نظرية كل شيء theory of everything لانها سوف تصف كل شيء مادى فى الكون. بل انهم يعتقدون انها سوف تصف الوعى نفسه -وهم بذلك يفترضون ان الوعى و العقل و الفكر و الروح هى كائنات مادية فى النهاية-.
لكن الذى اريد ان اؤكده فى هذا المنشور ان اهم شيء سوف تصفه نظرية الثقالة الكمومية هى مفردة الانفجار الاعظم big bang و مفردة الثقوب السوداء black holes.
و المفردة singularity تعنى النقطة التى تبدأ او تنتهى عندها الاشياء.
ومفردة الانفجار الاعظم هى مفردة عارية naked singularity نستطيع ان نراها -مثلا عبر اشعاع الخلفية الميكروى microwave background radiation اى اشعاع ال CMB الذى يتخلل الكون و عبر هذا التوسع المتسارع accelerating expansion الذى نرصده للكون-. و هذه المفردة هى بالضبط المفردة التى ابتدأ عندها الزمن و الفضاء نفسهما اللذان نعيش فيهما.
اما مفردة الثقوب السوداء فهى مفردة مخفية hidden singularity ينتهى عندها الفضاء-زمن كبنية هندسية اى ينتهى وجوده و كينونته لكن سوف ينتهى ايضا عندها الزمن نفسه الذى نعرفه لان الثقوب السوداء هى آخر شيء سوف يموت فى الكون.
ومفردة الانفجار الاعظم هى اهم بالنسبة للكسومولوجيا.
اما مفردة الثقوب السوداء فهى أهم بالنسبة للميكانيك الكمومى و تفسيره و اسسه.
بالخصوص بالنسبة لنا هنا فان معضلة ضياع المعلومات فى الثقوب السوداء black holes information loss paradox اى مسألة امكانية ضياع المعلومات فى مفردة الثقب الاسود هى اهم مسألة تخص الثقوب السوداء و هى ذات علاقة وثيقة بمعضلة الرصد الكمومى quantum measurement paradox اى دور الوعى فى عملية الرصد و القياس بصفة خاصة و دوره فى الفيزياء بصفة عامة و فى الكون بصفة اعم.
معضلة ضياع المعلومات فى الثقوب السوداء و معضلة الرصد الكمومى هما اعظم معضلتين تواجهان الفيزياء النظرية التى تبقى أهم اهدافها على الاطلاق هو ايجاد نظرية الثقالة الكمومية.
و لذا فانك تجد من يتناول نظرية الثقالة الكمومية بالدراسة يدرس و يهتم بالثقوب السوداء اكثر من اهتمامه بأى شيء آخر.
ولهذا فاننى على هذه الصفحة و على مدار سنوات طويلة ناقشنا هذه المعضلة من مختلف الزوايا و تناولنا بالنقاش مختلف الاكتشافات التى تمت فى هذا المضمار و كثير من هذه الاكتشافات وقعت خلال السنوات الاخيرة فقط منذ افتتاح هذه الصفحة عام 2016.
اولا كنا قدمنا فى عام 2016 على مستوى الماستر 2 نظرى بجامعة عنابة محاضرات بيداغوجية مستفيضة حول معضلة ضياع المعلومات فى الثقوب السوداء و قد ظهرت هذه المحاضرات فيما بعد كأهم و اكبر فصل فى كتاب النسبية العامة الذى نشره المعهد البريطانى للفيزياء IOP.
ملخص هذه المحاضرات بالعربية موجود هنا
ثم ناقشنا خلال السنوات الموالية اكتشافات الجدار-النارى firewall و ال ER=EPR و علاقة كل ذلك بتفسير و اسس الميكانيك الكمومى فى كتابات و ندوات مختلفة.
ثم قدمت فى عام 2018 محاضرات اخرى على مستوى الماستر 2 فيزياء نظرية فى جامعة عنابة تناولنا فيها التشابك الكمومى quantum entanglement و الثنائية الثقالية-المعيارية gauge-gravity duality فى مثالها الاشهر المعروف باسم التقابل AdS/CFT و دورهما فى انبعاث emergence الفضاء-زمن هذا الانبعاث الذى يلعب دورا محوريا فى حد ذاته فى معضلة ضياع المعلومات فى الثقوب السوداء.
هذه المحاضرات نشرت هى الاخرى مع المعهد البريطانى IOP كفصل فى كتاب نظرية الحقول الكمومية. تجدون الفصل حرا و مفتوحا هنا
ملخص هذه المحاضرات بالعربية هنا
ثم ابتدأنا عامى 2019 و 2020 النقاش حول حل معضلة ضياع المعلومات فى الثقب الاسود الذى كان قد ابتدأه بولشينسكى Polchinski مع تلاميذه قبل وفاته بقليل ثم واصل الحل من بعده تلاميذه و على رأسهم الفيزيائى النظرى الاماراتى احمد المهيرى و غيره.
هذا الحل الذى قدمه بولشينسكى و تلاميذه هو حل نهائى فى اطار نماذج معينة تعيش فى بعدين و هو حل لا يحتاج بالضرورة الى نظرية الاوتار الممتازة رغم انه ابتدأ منها.
خلاصة الحل انه ليس هناك ضياع للمعلومات فى الثقب الاسود بحيث ان اشعاع هاوكينغ Hawking radiation خارج الثقب مرتبط correlated عبر التشابك الكمومى مع المعلومات التى دخلت وراء افق الحدث event horizon.
فى هذا الحل فان افق الحدث يلعب دورا اكثر اهمية من مفردة الثقب الاسود.
هذا الحل يعتمد على كثير من الفيزياء الاساسية من اهمها: انبعاث الفضاء-زمن spacetime emergence, المبدأ الهولوغرافى holographic principle, التشابك الكمومى quantum entanglement, الثقوب-الدودية worm-holes, و الحاسوبية الكمومية quantum computation.
هذا الحل الذى ابتدأنا نقاشه هنا منذ سنوات سوف نرجع باذن الله الى تفصيله على هذه الصفحة فى منشورات قادمة ان شاء الله فترقبوه.

بى دى اف المحاضرات



Atomic Physics 1: Angular Momentum and Spin


فى هذه المحاضرة نتكلم عن العزم الحركى angular momentum و السبين spin.
فى الفقرة الاولى نقوم بالبرهان الرياضى على تكميم quantization العزم الزاوى المدارى orbital angular momentum فى الميكانيك الكمومى. و نقوم بحل معادلة جسيم على كرة حيث ان الحلول تعطى بدوال على الكرة تسمى التوافقيات الكروية spherical harmonics. هذه المسألة هى الجزء الزاوى فى ذرة الهيدروجين و اى مسألة مركزية central problem اخرى.
فى الفقرة الثانية نقدم تجربة ستارن-غالاش Stern-Gerlach experiment التاريخية (1922) فى الفيزياء التى تم بها اكتشاف العزم الزاوى للسبين spin angular momentum (او السبين اختصارا) و التى اكدت بما لا يدع اى مجال للشك صحة نظرية الميكانيك الكمومى الجديدة التى كان يتزعمها بوهر Bohr. هذه النظرية التى كانت تسمى فى ذلك الوقت الفيزياء الذرية.
فى الفقرة الثالثة نقدم نموذج الكيوبت qubit model للسبين الذى يسمح لنا بفهم السبين على انه وحدة المعلومات فى الحاسوبية الكمومية quantum computation. السبين مفهوم معقد جدا نظريا ليس له اى علاقة بالحركة فى الفضاء الفيزيائى و يبقى ابسط فهم له هو مفهوم البت الكمومى quantum bit او الكيوبت -الذى يرجع الى ويلر Wheeler- و الذى جاء بعد مفهوم السبين بحوالى خمسين سنة.
فى الفقرة الرابعة نقدم بسرعة نظرية جمع العزوم الزاوية theory of addition of angular momenta. بالخصوص نقدم بالتفصيل فضاء هيلبرت Hilbert space الخاص بمجموع العزم الزاوى المدارى و السبين الذى سيلعب دورا اساسيا فى ذرة الهيدروجين.
بالنسبة للطلبة الحضوريين (الفقرة الثانية و الفقرة الثالثة غير داخلتين فى الامتحان النهائى).

Atomic Physics 2: Bohr's atom and its harmonic oscillator formulation


بعد ان قمنا بحل الجزء الزاوى angular part لذرة الهيدروجين فى المحاضرة الماضية.
نقوم فى الفقرة الاولى من هذه المحاضرة بحل الجزء المدارى radial part لذرة الهيدروجين.
بالخصوص نقوم باشتقاق المستويات الطاقوية لذرة الهيدروجين (الشهيرة باسم مستويات طاقة بوهر Bohr's energy levels) و نقوم بالبرهان التفصيلى على تكميم الطاقة energy quantization التى تعد واحدة من اعظم نتائج الفيزياء و الفيزياء النظرية و الميكانيك الكمومى و الفيزياء الذرية موضوعيا و تاريخيا و تعليميا.
الاساس فى كل شيء هو الميكانيك الكمومى فكل شيء تابع و خاضع الى قواعده.
نقوم ايضا باشتقاق الدوال الموجية المدارية orbital wave functions و نبرهن ان الذى يحكمها هى كثيرات حدود لاغار Laguerre polynomials (تذكروا فان الدوال الموجية الخاصة بالجزء الزاوى المعطاة بالتوافقيات الكروية spherical harmonics وجدنا فى المحاضرة الماضية ان الذى يحكمها هى كثيرات حدود لوجوندر Legendre polynomials).
فى الفقرة الثانية من المحاضرة اردنا سبر السبب الفيزيائى العميق وراء تكميم الطاقة فى ذرة الهيدروجين -التى افضل شخصيا تسميتها باسم ذرة بوهر Bohr's atom-.
اولا نحن نعلم ان احتمال probability وجود الالكترون فى اى نقطة من الفضاء يجب ان يكون عدد منتهى -بل ان هذا هو السبب الرياضي وراء التكميم الذى وجدناه فى الفقرة الاولى-.
لكن هذا غير كافى لان هذا السبب هو راجع فقط الى وجوب ضرورة ان يكون الاحتمال الكلى لوجود الالكترون فى مكان ما من الفضاء هو واحد (هذه هى قاعدة بورن Born rule الاحصائية وهو سبب غير هين بالمرة لكنه غير كافى كتفسير فيزيائى وراء التكميم).
السبب الفيزيائى الحقيقى وراء تكميم الطاقة فى ذرة الهيدروجين هو الثنائية موجة-جسيم wave-particle duality التى يبقى مبدأ الارتياب لهايزنبرغ Heisenberg uncertainty principle هو ادق تعبير فيزيائى و رياضى لها.
الثنائية موجة-جسيم تعنى ان الالكترون داخل ذرة الهيدروجين يتصرف بالفعل كموجة.
اذن فى هذه الفقرة الثانية نقدم نتيجة غير معروفة بالمرة فى كتب الميكانيك الكمومى و الفيزياء الذرية حيث نعيد صياغة ذرة الهيدروجين على انها مسألة هزاز توافقى harmonic oscillator problem.
والهزاز التوافقى هو اساس الطبيعة و اساس الفيزياء.
بشكل ادق نبين ان ذرة بوهر هى عبارة عن هزازين توافقين فى بعدين مقترنين coupled عبر شرط ان العزم الزاوى angular momentum الكلى الخاص بهما يساوى صفر.
فى صياغة الهزاز التوافقى لذرة الهيدروجين فان علاقات التبادل الاساسية fundamental commutation relations التى تحكم تكميم الفضاء الطورى phase space الخاص بالجملة يمكن كتابتها بشكل مباشر.
علاقات التبادل هذه تؤدى مباشرة الى مبدأ الارتياب لهايزنبرغ بين متغيرات الجملة و منه فان الثنائية موجة-جسيم التى تؤسس لتكميم الطاقة لذرة الهيدروجين تصيح واضحة جدا.
نقدم بالخصوص كيف نحصل على تكميم الطاقة فى ذرة الهيدروجين انطلاقا من صياغة الهزاز التوافقى لذرة الهيدروجين.
بالنسبة للطلبة الحضوريين فان الفقرة الثانية غير داخلة فى الامتحان.

Atomic Physics 3 : The spin-orbit coupling and the fine structure of hydrogen


بعدما قمنا فى المحاضرتين السابقتين بحساب مستويات طاقة بوهر Bohr's energy levels لذرة الهيدروجين و وجدنا انها تتعلق على عدد كمومى واحد يسمى العدد الكمومى الرئيسى principal quantum number و يرمز له ب n فاننا نقوم فى هذه المحاضرة بحساب اهم تصحيح اضطرابى perturbative correction لهذه المستويات الطاقوية.
سنحتاج بشكل مكثف الى نظرية الاضطرابات perturbation theory اذن نقدم فى الفقرة الاولى نبذة سريعة عن اهم ثلاثة نتائج فى نظرية الاضطرابات من الميكانيك الكمومى التى سوف نحتاجها:
-تصحيح نظرية الاضطرابات غير-المنحلة non-degenerate من الرتبة-الاولى first-order. وهذه سهلة جدا عموما.
-تصحيح نظرية الاضطرابات المنحلة degenerate من الرتبة-الاولى first-order. وهذه معقدة جدا عموما.
-تصحيح نظرية الاضطرابات المنحلة degenerate من الرتبة-الاولى first-order فى حالة وجود تناظر symmetry و صمود invariance. فى هذه الحالة نبين انه يمكننا استخدام نظرية الاضطرابات غير-المنحلة من الرتبة الاولى مع اجراء الحساب فى ما يسمى الحالات الكمومية الجيدة good quantum states.
تذكروا فان هذه المادة هى مادة فيزياء ذرية و ليست ميكانيك كمومى اذن هذه النتائج نقدمها بدون برهان.
فى الفقرة الثانية من المحاضرة نبدأ فى حساب اهم تصحيح على الاطلاق لذرة الهيدروجين وهو يسمى البنية الدقيقة لذرة الهيدروجين fine structure of hydrogen atom.
هذا التصحيح هو اهم تصحيح لانه هو اكبر تصحيح من ناحية القدر.
البنية الدقيقة fine structure لذرة الهيدروجين تتشكل من تأثيرين فيزيائيين مختلفين جدا فى اطار نظرية الميكانيك الكمومى.
التأثير الاول هو التصحيح النسبى relativistic correction الناجم عن سرعة الالكترون الكبيرة داخل ذرة الهيدروجين. وهذا تصحيح سهل الحساب عكس ما قد يتبادر الى الذهن من ان التصحيحات النسبية صعبة.
التأثير الثانى هو التصحيح الناجم عن الاقتران coupling بين العزم الزاوى لسبين spin angular momentum الالكترون و العزم الزاوى المدارى orbital angular momentum للالكترون. لهذا فان هذا التأثير يسمى الاقتران سبين-مدار spin-orbit coupling.
حساب هذا التصحيح معقد جدا.
اولا علينا ان نفهم ان هذا التصحيح ناجم (فى معلم الاكترون electron frame) عن تفاعل العزم الثنائى المغناطيسى magnetic dipole moment لسبين الالكترون مع الحقل المغناطيسى magnetic field للبروتون. معلم الالكترون هو معلم غير-عطالى non-inertial و هذا يزيد فى تعقيد عملية الحساب.
فى معلم البروتون proton frame (الذى هو معلم المختبر laboratory frame وهو معلم عطالى inertial) فان هذا التصحيح ناجم عن تفاعل العزم الكهربائى electric dipole moment للاكترون مع الحقل الكهربائى electric field للبروتون.
الحساب فى معلم الالكترون رعم انه غير-عطالى الا انه اسهل - لأن التعامل مع الحقول المغناطيسية هو اسهل من التعامل مع الحقول الكهربائية عكس ما قد يتبادر الى الذهن مرة اخرى-.
نقوم اذن بحساب طاقة تفاعل العزم الثنائى المغناطيسى لسبين الالكترون مع الحقل المغناطيسى للبروتون (هذا الحقل المغناطيسى نجده متناسب مع العزم الزاوى المدارى للاكترون).
بعد ذلك علينا تطبيق نظرية الاضطرابات المنحلة من اجل حساب التصحيح الناجم عن هذا التأثير. تذكروا فان مستويات طاقة بوهر هى مستويات منحلة degenerate levels اذن نحن مضطرون الى استخدام نظرية الاضطرابات المنحلة و هى معقدة جدا.
لحسن الحظ فان هناك تناظر و صمود دورانى للجملة مما يسمح لنا بتعريف ما يسمى الاعداد الكمومية الجيدة good quantum numbers للجملة و استخدام نظرية الاضطرابات غير-المنحلة الاسهل مع اجراء الحساب فى الحالات الكمومية الجيدة التى هى اشعة الحالة فى فضاء هيلبرت Hilbert space التى تقابل الاعداد الكمومية الجيدة.
الاعداد الكمومية الجيدة هى بالضبط القيم الذاتية eigenvalues للمؤثرات operators المرفقة بالتناظر و الصمود الدورانى.
هذا حساب اسهل نوعا ما لكنه حساب ليس سهلا ابدا.
بعد اجراء جميع الحساب و اخذ مجموع التصحيح النسبى و التصحيح سبين-مدار نحصل على البنية الدقيقة التى هى من اعظم و ادق تنبئات الميكانيك الكمومى التى اكدتها التجربة منذ اكثر من 100 سنة.
مثلا نجد ان مستويات طاقة بوهر تصبح متعلقة ليس فقط على العدد الكمومى الرئيسى بل تتعلق ايضا على القيم الذاتية للعزم الزاوى الكلى total angular momentum للجملة وهو العزم المدارى + السبين.
بالنسبة للطلبة الحضوريين اؤكد ان كل هذه المحاضرة داخلة فى الامتحان النهائى.

Atomic Physics 4: Hyperfine structure, Stark effect and Lamb shift


فى هذه المحاضرة نقوم بحساب ثلاثة تصحيحات اضطرابية perturbative corrections اضافية لذرة الهيدروجين.
اولا ما يسمى تصحيح البنية الادق hyperfine structure لذرة الهيدروجين. وهذا ناجم عن تفاعل العزوم الزاوية لسبينات spin angular momenta الالكترون و البروتون.
فى هذا التصحيح فان دالة موجة السبين الكلى -سبين الالكترون+ سبين البروتون- تنقسم الى مستويين طاقويين: الاول يسمى الحالة المفردة singlet state و هو غير منحل non-degenerate و الثاني يسمى الحالة الثلاثية triplet state و كما يدل اسمها فهى منحلة degenerate الى ثلاثة حالات لها نفس الطاقة.
تصحيح البنية الادق hyperfine structure correction هو اصغر من ناحية القيمة من تصحيح البنية الدقيقة fine structure correction الذى قدمناه فى المحاضرة السابقة.
لكن اهميته تكمن فى الكوسمولوجيا.
حيث ان القفزة transition (قفزة الالكترون) بين الحالة المفردة و الحالة الثلاثية تقابل موجة كهرومغناطيسية شهيرة جدا تسمى 21 سم (لان طول الموجة هو فعلا 21 سم) و هذه الموجة هى التى تطغى على اشعاع الخلفية الميكروى microwave background radiation للكون منذ عهد تشكل الهيدروجين -ما يسمى عهد التركيب combination epoch- حتى عهد تشكل النجوم -ما يسمى عهد اعادة التأين re-ionization epoch-
خلال كل هذه الاحقاب من 400 مليون سنة بعد الانفجار الاعظم حتى 1 مليار سنة بعد الانفجار الاعظم فان كل اشعاع الخلفية الميكروى ال CMB كان متشكل من هذه ال 21 سم.
فى الفقرة الثانية نناقش تأثير ستارك Stark effect و هو كيف تتصرف ذرة الهيدروجين تحت تأثير حقل كهربائى منتظم. هذا التأثير يلعب دورا اساسيا فى الفيزياء التجريبية مثلما ان التصحيح الادق يلعب دورا كبيرا فى الكوسمولوجيا.
فى الفقرة الثالثة نقدم تصحيح شهير جدا يسمى ازاحة لامب Lamb shift و لامب Lamb هو الفيزيائى الذى اكتشف تجريبيا هذا التأثير و حصل على نوبل من اجل ذلك.
فى هذا التأثير فان التصحيح الداخل على مستويات طاقة بوهر يرجع الى تكميم الحقل الكهرومغناطيسى quantization of the electromagnetic field.
بل هو يرجع بالضبط الى مخططات فايمان Feynman diagrams التى تعطى تصحيح
- لدالة موجة الفوتون (استقطاب الفراغ vacuum polarization)
-و لطاقة الالكترون (اعادة استنظام الكتلة mass renormalization)
-و للعزم المغناطيسى للالكترون (تصحيح العقدة vertex correction).
اكتشاف ازاحة لامب عام 1947 كان الدافع الاساسى وراء التطوير الهائل و السريع و الدراماتيكى لنظرية الحقول الكمومية quantum field theory و الصعود التاريخى لفيزياء الجسيمات على زعامة الفيزياء النظرية.
بالنسبة للطلبة الحضوريين فقط الفقرة الاولى داخلة فى الامتحان.

Atomic Physics 5: The Zeeman effect


نقدم فى هذه المحاضرة تصحيح اضطرابى perturbative correction اخير لمستويات طاقة بوهر Bohr's energy levels هو تأثير زيمان Zeeman effect.
تأثير زيمان هو كيف تتصرف ذرة الهيدروجين تحت تأثير حقل مغناطيسى.
وكما ذكرت فان الحقول المغناطيسية اهم بكثير من الحقول الكهربائية و لهذا فان تأثير زيمان اهم من تأثير ستارك Stark effect الذى قدمناه فى الحصة السابقة.
حيث ان اهمية تأثير زيمان تكمن فى التجربة كما تكمن فى الطبيعة لان تأثير زيمان قد يكون فعل طبيعى محض.
مثلا فان الحقول المغناطيسية التى تؤثر فى الفضاء النجمى داخل المجرات و بين المجرات تؤدى الى ازاحة المستويات الطاقوية للذرات بشكل معين يتنبأ به الميكانيك الكمومى.
اذن تأثير زيمان يأتى مع حقل مغناطيسى خارجى.
وتذكروا فان تصحيح البنية الدقيقة fine structure correction و بالضبط الاقتران سبين-مدار spin-orbit coupling يأتى مع حقل مغناطيسى داخلى و هو الحقل الذى يولده البروتون فى معلم الالكترون.
لدينا هنا اذن حالات.
الحالة الاولى ان يكون الحقل المغناطيسى الخارجى اصغر بكثير من الحقل المغناطيسى الداخلى. فى هذه الحالة فان الاقتران سبين-مدار اقوى من الحقل المغناطيسى الخارجى الذى يمكننا ان نتعامل معه على انه اضطراب perturbation.
فى هذه الحالة فان الاعداد الكمومية الجيدة good quantum numbers هى نفسها الاعداد الكمومية التى تظهر فى تصحيح البنية الدقيقة. اذن يمكننا استخدام نظرية الاضطرابات غير-المنحلة non-degenerate perturbation theory مع اجراء الحساب فى الحالات states المرفقة بهذه الاعداد الكمومية.
نقوم باجراء الحساب باستخدام مبرهنة فيغنر-ايكارت Wigner-Eckart theorem و نجد ان التصحيح متناسب مع ما يسمى معامل لاندى Lande factor. اى انه رغم ان العزم المغناطيسى total magnetic moment الكلى للالكترون غير متناسب مع العزم الزاوى الكلى total angular momentum للالكترون فان متوسط العزم المغناطيسى الكلى للالكترون متناسب مع متوسط العزم الزاوى الكلى للالكترون.
الحالة الثانية ان يكون الحقل المغناطيسى الخارجى اكبر بكثير من الحقل المغناطيسى الداخلى للبروتون. فى هذه الحالة فان تأثير زيمان هو اقوى من الاقتران سبين-مدار. بل انه فى هذه الحالة فان الحقل المغناطيسى يجب عدم احتسابه على انه اضطراب بل يجب ضمه الى طاقة بوهر غير-الاضطرابية التى ابتدأت بها هذه النظرية.
اذن الاعداد الكمومية الجيدة فى هذه الحالة هى الاعداد الكمومية المرفقة بذرة بوهر غير المضطربة. و يمكننا استخدام نظرية الاضطرابات غير-المنحلة مع اجراء الحساب فى الحالات المرفقة بهذه الاعداد الكمومية.
فى هذه الحالة يجب ايضا اعادة حساب تصحيح البنية الدقيقة لانه تغير بسبب تغير طبيعة الحالات الكمومية الجيدة.
بصفة عامة فان الحقل المغناطيسى الخارجى قد يكون لا هو صغير جدا و لا هو كبير جدا. فى هذه الحالة فان الحساب اعقد بكثير لانه يجب استخدام نظرية الاضطرابات المنحلة degenerate بكل قوتها.
بالنسبة للطلبة الحضوريين هذه المحاضرة داخلة فى الامتحان.

Atomic Physics 6: Time-dependent harmonic perturbation and Fermi's golden rule

Atomic Physics 7: Stimulated emission and absorption of radiation

Atomic Physics 8: Hydrogen atom quantum computation

لحد الآن لم نتعامل مع ذرة الهيدروجين الا كجملة ساكنة اى اننا لم نناقش الا طيف المستويات الطاقوية لذرة الهيدروجين و ذرة بوهر و لم نتكلم اطلاقا عن الجزء الديناميكى لذرة بوهر الذى يتشكل من طيف القفزات الكمومية بين هذه المستويات الطاقوية.
هذه القفزات الكمومية هى اهم شيء يميز الذرات و هو اهم انجاز جاء به بوهر و من كان معه بل كان فهم هذه القفزات هو المحفز الاساسى وراء تطوير نظرية الميكانيك الكمومى فى العشرينات.
اولا اقول ان القفزات الكمومية quantum jumps التى تسمى ايضا الانتقالات الكمومية quantum transitions بين المستويات الطاقوية لبوهر تحتاج الى اضطراب متعلق-بالزمن time-dependent perturbation.
اى نحتاج الى تفاعل ما بين الذرات و الوسط الخارجى يسمح للذرات بالقفز بين مختلف المستويات الطاقوية لبوهر Bohr's energy levels.
هذا التفاعل كما سنرى هو بالضبط الاشعاع الكهرومغناطيسى و لهذا فان هذا الفصل هو فى الحقيقة يعنى بالتفاعل بين الذرات و الاشعاع الكهرومغناطيسى.
اذن لو تصورنا ان ذرة الهيدروجين معزولة بالكامل بحيث لا تتعرض الى اى اضطراب او اشعاع كهرومغناطيسى فان الالكترون فى مستوى طاقوى معين لن يقفز ابدا الى اى مستوى طاقوى آخر.
اذن فى المحاضرة الاولى نقوم بالتذكير بأهم النتائج من نظرية الاضطرابات المتعلقة-بالزمن.
بالخصوص سنذكر بما يسمى قاعدة فرمى الذهبية Fermi's golden rule التى تسمح لنا بحساب ما يسمى نسبة الانتقال transition rate اى احتمال الانتقال فى وحدة الزمن الذى هو بعبارة اخرى عدد الالكترونات فى وحدة الزمن التى تقوم بالقفز من مستوى ابتدائى معين الى مجموعة محددة من المستويات النهائية.
نقوم مباشرة بالتطبيق على ما يسمى الاضطرابات التوافقية hramonic perturbation التى هى اضطرابية جيبية sinusoidal وهى اهم نوع من الاضطرابات لان الاشعاع المغناطيسى نفسه سنتعامل معه على انه اضطراب توافقى.
سنكتشف فى حالة الاضطراب التوافقى و مباشرة ان هناك فى الحقيقة نوعان من التفاعل بين الذرات و الاضطراب التوافقى:
-الامتصاص absorption اى ان الذرة تمتص طاقة من الاضطراب يسمح للالكترون بالانتقال الى مستوى اعلى.
-الانبعاث المستحث stimulated emission اى ان الذرة تعطى طاقة للاضطراب مما يجعل الالكترون يقفز الى مستوى ادنى.
حسب الميكانيك الكلاسيكى لا يوجد هناك انبعاث مستحث نهائيا لكن يوجد ما يسمى الانبعاث التلقائى spontaneous emission اى يمكن للالكترون ان يقفز لوحده الى مستوى ادنى مع ارسال اشعاع دون الحاجة الى اى اضطراب.
الوضعية فى الميكانيك الكمومى عكسية. اى لا يوجد انبعاث تلقائى مطلقا بل كل شيء هو انبعاث مستحث.
بالفعل فانه حسب الميكانيك الكمومى فان كل شيء هو فى الحقيقة انبعاث مستحث فحتى الانبعاث التلقائى هو راجع الى تفاعل الذرة مع الفراغ الكمومى quantum vacuum.
الانبعاث المستحث كان مفجأة شديدة للفيزيائيين عندما اكتشفه اينشتاين لاول مرة عام 1917.
نؤكد ايضا ان الامتصاص و الانبعاث المستحث هما فى الحقيقة ظواهر رنين resonances بين التواترات او الترددات الطبيعية natural frequencies للذرات و التواترات او الترددات السائقة driving frequencies للاضطراب التوافقى او الاشعاع الكهرومغناطيسى.
نبين باستخدام ما يسمى شرط التوازن التفصيلى condition of detailed balance على ان نسبة الانتقال بالامتصاص absorption transition rate تساوى بالضبط نسبة الانتقال بالانبعاث المستحث stimulated emission transition rate.
هذه نتيجة مهمة جدا. اذن نعيد فى المحاضرة الثانية حساب تفاعل الذرات مع موجة كهرومغناطيسية بالتفصيل.
هذا حساب طويل ومعقد لكنه حساب اساسى فى موضوع تفاعل الذرات مع الحقل الكهرومغناطيسى الكلاسيكى.
نبين مرة اخرى ان هناك توازن تفصيلى بين الامتصاص و الانبعاث المستحث.
الانبعاث المستحث هو اساس تطبيق الليزر LASER اى ظاهرة تضخيم الضوء عبر الانبعاث المستحث للاشعاع light amplification by stimulated emission of radiation.
حيث ان فوتون واحد يمكنه ان يستحث الالكترون الى القفز الى حالة دنيا مما يؤدى الى خروج فوتون اضافى بنفس التواتر. هذان الفوتونان يتسببان بنفس الطريقة فى خروج 4 فوتونات و هذه ال 4 تسبب بعد ذلك فى خروج 8 وهكذا نحصل على عدد ضخم جدا من الفوتونات تخرج فى نفس الوقت و كلها بنفس التواتر.
فى الفقرة الثانية من المحاضرة الثانية نرجع الى الحساب الذى قام به اينشتاين عام 1917 و نقوم باشتقاق اشعاع الجسم الاسود black body radiation و نسبة الانتقال بالانبعاث التلقائى spontaneous emission transition rate.
بالفعل فان الذرات عندما تكون فى حالة توازن حرارى thermal equilibrium مع الاشعاع الكهرومغناطيسى فان القانون الحاكم لتوزيع الطاقة هو توزيع بولتزمان Boltzmann distribution و كنتيجة فان الجملة ذرات+اشعاع تتصرف كجسم اسود black body و نجد ان الامتصاص يساوى الانبعاث المستحث اى شرط التوازن التفصيلى مع شرط اضافى آخر يسمح لنا بحساب الانبعاث التلقائى.
هذا البرهان الذى قدمه اينشتاين هنا هو من اجمل نتائج الفيزياء الاساسية و الفيزياء النظرية فى التاريخ.
فى المحاضرة الثالثة نركز على الجمل ذات-الحالتين two-level systems و الجمل ذات-الجسيمين two-particle systems اللذان يلعبان دورا مهما فى الحاسوبية الكمومية quantum computation.
الجمل ذات-الحالتين تلعب ايضا دورا مهما فى تطبيق الميزر MASER و الرنين النووى المغناطيسى nuclear magnetic resonance اى ال NMR.
نقوم اولا بحل معادلة شرودينغر بشكل مضبوط من اجل الجمل ذات-الحالتين مما يسمح لنا برؤية ظاهرة الرنين بشكل بعيد عن نظرية الاضطرابات.
فى الفقرة الثانية نقوم بتفصيل شديد ببناء البوابة المنطقية CNOT التى هى بوابة كونية universal gate فى الحاسوبية الكمومية و هذا باستخدام ذرة الهيدروجين او البوزيترونيوم positronium او ال NMR.







نموذج الابعاد الاضافية الكبيرة

نموذج ال ADD نسبة الى اصحابه و يسمى ايضا نموذج الابعاد الاضافية الكبيرة large extra dimensions او نموذج ال LED اختصارا هو نموذج كان مشهورا جدا فى آواخر التسعينات فقد كان احد موضات ذلك العصر.

فهو النموذج الذى ابتدأت به فكرة (سيناريوهات عوالم-البراينات braneworld scenarios) او (كوسمولوجيا البراينات brane cosmology) فى فيزياء الجسيمات الاولية و فى الكوسمولوجيا.
هذا النموذج جعل اصحابه وهم ال ADD وهم نيما اركانى-حامد Nima Arkani-Hamed و سافاس ديموبولوس Savas Dimopoulos و جيا دفالى Gia Dvali من مشاهير الفيزياء النظرية و جعلهم اساتذة فى اعرق الجامعات و هم مازالوا هناك رغم ان النموذج يتميز بشيئين اساسيين:
- اولا هو نموذج بسيط جدا يمكن لاى طالب مبتدئ ان يفهمه كما سنبين ذلك.
-و ثانيا هو نموذج خاطئ تجريبيا لكنه فتح المجال امام حل معضلة الهايراركى Hierarchy problem او معضلة التراتب (اى لماذا قوة الثقالة هى اضعف بكثير من بقية القوى فى مجالت الجسيمات الاولية).
اذن يكفى ان تكون الفكرة بسيطة جدا و يكفى ان تعرضها بشكل جيد ثم يكفى ان تكون محظوظا فى المكان و الوقت المناسبين و تُقبل من المجتمع العلمى الدولى حتى تنجح.
نموذج ال ADD يستمد من نظرية الوتر string theory التى تاتى من بين ما تاتى به بفكرتين اساسيتين:
-اولا ان ابعاد الفضاء-زمن هى اكثر من اربعة.
-و ثانيا ان قوة الثقالة عكس بقية القوى يمكنها ان تنتشر فى جميع ابعاد الفضاء-زمن.
تذكروا ايضا ان نظرية الوتر هى التى جاءت بمفهوم البراينات branes (مفردها براين brane) الذى يلعب دورا محوريا فى نموذج ال ADD و غيرها من سيناريوهات عوالم-البراينات.
كلمة البراين brane تُنطق مثل ال brain اى الدماغ لكن الكلمة شيء آخر تماما فهى أتت من الجزء الاخير من كلمة membrane اى mem(brane) وهو الغشاء و هو السطح الثنائى و لان هذه البراينات قد تكون بأى بعد فانه تم اخذ كلمة brane كتسمية لهذه الاغشية فى بعدين و لغيرها من البراينات فى الابعاد العليا.
بكل بساطة البراين هو تعميم للشحنة النقطية الكهربائية وهو سطح surface او حجم volume او حجم-فرطى hyper-volume موجود فى فضاء-زمن ببعد أعلى.
بعبارة اخرى فان مسار البراين فى الفضاء-زمن هو بكل بساطة فضاء-زمن بأكمله لكن ببعد ادنى وهذا هو الفضاء-زمن الذى نعيش فيه وهو مغموس embedded فى فضاء-زمن اكبر ببعد اعلى وهو الفضاء-زمن الاساسى و يسمى فى هذه الحالات الجسد او البلك bulk.
اذن نحن نعيش على البراين و ليس فى الجسد او البلك. وهذا البراين الذى نعيش عليه ينتشر فى الجسد أو البلك و كأنه جسيم وهو فعلا كذلك بأتم معنى للكلمة.
اى اننا نفترض فى نموذج ال ADD ان جميع القوى غير-الثقالية (الكهرومغناطيسية و النووية اللونية القوية strong color force و النووية الذوقية الضعيفة weak flavor force) تعيش فوق براين هو عبارة عن فضاء-زمن بأربعة ابعاد اما قوة الثقالة فهى تؤثر على البراين لكن يمكنها ايضا ان تنتشر عبر الفضاء-زمن الجسد او البلك الذى نفترض ان عدد ابعاده هو خمسة او بصفة عامة D. هذا موضح فى الصورة الاولى.
النتيجة الاساسية ان طول بلانك فى البلك lP(D) مرتبط بطول بلانك فى البراين lP بالعلاقة فى الصورة الثانية.
الثابت G هو ثابت نيوتن على البراين اما G(D) فهو ثابت نيوتن فى الجسد او البلك.
اذن رغم ان ثابت نيوتن هو مكافئ لطول بلانك (كما بينا فى منشور سابق) الا ان طول بلانك هو اكثر اساسية لانه يحمل نفس الوحدات فى اى بعد و بالتالى يمكننا مقارنة طول بلانك فى اربعة ابعاد (البراين) و طول بلانك فى D بعد (الجسد او البلك). وهذه العلاقة فى الصورة الثانية تعطينا بالضبط هذه المقارنة.
الآن نقوم بالتضميم compactification على الابعاد الاضافية.
اى اننا نفترض ان الابعاد خارج البراين هى متضامة compact عبارة عن دوائر بنصف قطر يساوى R اى بمحيط نرمز له اما ب yc او lC اى ان lC=yc=2*π*R.
نضع الآن D=d+4 و نأخذ فعل هيلبرت-اينشتاين Hilbert-Einstein action للنسبية العامة فى الجسد اى فى بعد D. هذا الفعل موجود فى الصورة الثالثة.
ثم نفترض ان المترية لا تتعلق بالابعاد الاضافية اى انه يمكننا اجراء التكامل على الابعاد الاضافية لنحصل على المعادلة فى الصورة الرابعة التى هى فعل هيلبرت-اينشتاين للنسبية العامة فى البراين اى فى بعد اربعة.
من هذه النتيجة نستنتج ان كتلة بلانك Planck mass فى بعد D (فى البلك او الجسد) وهى M(d+2) متناسبة مع كتلة بلانك فى بعد اربعة (البراين) وهى MPl حيث ان ثابت التناسب هو بالضبط حجم الابعاد الاضافية yc**d=yc**(D-4). انظر الصورة الخامسة.
تذكروا الآن العلاقة التى تربط كتلة بلانك مع ثابت نيوتن من المنشور السابق.
نجد اذن النتيجة فى الصورة السادسة التى يمكن التعبير عنها بأن نسبة ratio ثابت نيوتن فى بعد D اى فى البلك وهو G(D) و ثابت نيوتن فى اربعة ابعاد وهو G يساوى بالضبط حجم الابعاد الاضافية اى lC**(D-4).
باستخدام النتيجة فى الصورة الثانية و النتيجة فى الصورة السادسة يمكننا التعبير عن محيط الابعاد الاضافية lC بدلالة طول بلانك فى البعد D اى فى البلك lP(D) و طول بلانك فى اربعة ابعاد اى فى البراين lP بالعلاقة فى الصورة السابعة.
نختار الآن عدد الابعاد الاضافية d=D-4 و محيط الابعاد الاضافية lC بحيث ان طول بلانك lP فى اربعة ابعاد اى على البراين اين نعيش هو القيمة المعروفة 10 للاس -33 سنتيمتر اما طول بلانك lP(D) فى بعد D اى فى البلك او الجسد فهو يساوى الى اقصى طول تم قياسه فى التجارب المعملية وهو حوالى 10 للاس -18 سنتيمتر.
اى اننا نختار كتلة بلانك الاساسية التى هى كتلة بلانك فى البلك تساوى الى 1 TeV اى 1 تيراالكترون فولط وهو سلم scنموذج ال ADD نسبة الى اصحابه و يسمى ايضا نموذج الابعاد الاضافية الكبيرة large extra dimensions او نموذج ال LED اختصارا هو نموذج كان مشهورا جدا فى آواخر التسعينات فقد كان احد موضات ذلك العصر. فهو النموذج الذى ابتدأت به فكرة (سيناريوهات عوالم-البراينات braneworld scenarios) او (كوسمولوجيا البراينات brane cosmology) فى فيزياء الجسيمات الاولية و فى الكوسمولوجيا. هذا النموذج جعل اصحابه وهم ال ADD وهم نيما اركانى-حامد Nima Arkani-Hamed و سافاس ديموبولوس Savas Dimopoulos و جيا دفالى Gia Dvali من مشاهير الفيزياء النظرية و جعلهم اساتذة فى اعرق الجامعات و هم مازالوا هناك رغم ان النموذج يتميز بشيئين اساسيين: - اولا هو نموذج بسيط جدا يمكن لاى طالب مبتدئ ان يفهمه كما سنبين ذلك. -و ثانيا هو نموذج خاطئ تجريبيا لكنه فتح المجال امام حل معضلة الهايراركى Hierarchy problem او معضلة التراتب (اى لماذا قوة الثقالة هى اضعف بكثير من بقية القوى فى مجالت الجسيمات الاولية). اذن يكفى ان تكون الفكرة بسيطة جدا و يكفى ان تعرضها بشكل جيد ثم يكفى ان تكون محظوظا فى المكان و الوقت المناسبين و تُقبل من المجتمع العلمى الدولى حتى تنجح. نموذج ال ADD يستمد من نظرية الوتر string theory التى تاتى من بين ما تاتى به بفكرتين اساسيتين: -اولا ان ابعاد الفضاء-زمن هى اكثر من اربعة. -و ثانيا ان قوة الثقالة عكس بقية القوى يمكنها ان تنتشر فى جميع ابعاد الفضاء-زمن. تذكروا ايضا ان نظرية الوتر هى التى جاءت بمفهوم البراينات branes (مفردها براين brane) الذى يلعب دورا محوريا فى نموذج ال ADD و غيرها من سيناريوهات عوالم-البراينات. كلمة البراين brane تُنطق مثل ال brain اى الدماغ لكن الكلمة شيء آخر تماما فهى أتت من الجزء الاخير من كلمة membrane اى mem(brane) وهو الغشاء و هو السطح الثنائى و لان هذه البراينات قد تكون بأى بعد فانه تم اخذ كلمة brane كتسمية لهذه الاغشية فى بعدين و لغيرها من البراينات فى الابعاد العليا. بكل بساطة البراين هو تعميم للشحنة النقطية الكهربائية وهو سطح surface او حجم volume او حجم-فرطى hyper-volume موجود فى فضاء-زمن ببعد أعلى. بعبارة اخرى فان مسار البراين فى الفضاء-زمن هو بكل بساطة فضاء-زمن بأكمله لكن ببعد ادنى وهذا هو الفضاء-زمن الذى نعيش فيه وهو مغموس embedded فى فضاء-زمن اكبر ببعد اعلى وهو الفضاء-زمن الاساسى و يسمى فى هذه الحالات الجسد او البلك bulk. اذن نحن نعيش على البراين و ليس فى الجسد او البلك. وهذا البراين الذى نعيش عليه ينتشر فى الجسد أو البلك و كأنه جسيم وهو فعلا كذلك بأتم معنى للكلمة. اى اننا نفترض فى نموذج ال ADD ان جميع القوى غير-الثقالية (الكهرومغناطيسية و النووية اللونية القوية strong color force و النووية الذوقية الضعيفة weak flavor force) تعيش فوق براين هو عبارة عن فضاء-زمن بأربعة ابعاد اما قوة الثقالة فهى تؤثر على البراين لكن يمكنها ايضا ان تنتشر عبر الفضاء-زمن الجسد او البلك الذى نفترض ان عدد ابعاده هو خمسة او بصفة عامة D. هذا موضح فى الصورة الاولى. النتيجة الاساسية ان طول بلانك فى البلك lP(D) مرتبط بطول بلانك فى البراين lP بالعلاقة فى الصورة الثانية. الثابت G هو ثابت نيوتن على البراين اما G(D) فهو ثابت نيوتن فى الجسد او البلك. اذن رغم ان ثابت نيوتن هو مكافئ لطول بلانك (كما بينا فى منشور سابق) الا ان طول بلانك هو اكثر اساسية لانه يحمل نفس الوحدات فى اى بعد و بالتالى يمكننا مقارنة طول بلانك فى اربعة ابعاد (البراين) و طول بلانك فى D بعد (الجسد او البلك). وهذه العلاقة فى الصورة الثانية تعطينا بالضبط هذه المقارنة. الآن نقوم بالتضميم compactification على الابعاد الاضافية. اى اننا نفترض ان الابعاد خارج البراين هى متضامة compact عبارة عن دوائر بنصف قطر يساوى R اى بمحيط نرمز له اما ب yc او lC اى ان lC=yc=2*π*R. نضع الآن D=d+4 و نأخذ فعل هيلبرت-اينشتاين Hilbert-Einstein action للنسبية العامة فى الجسد اى فى بعد D. هذا الفعل موجود فى الصورة الثالثة. ثم نفترض ان المترية لا تتعلق بالابعاد الاضافية اى انه يمكننا اجراء التكامل على الابعاد الاضافية لنحصل على المعادلة فى الصورة الرابعة التى هى فعل هيلبرت-اينشتاين للنسبية العامة فى البراين اى فى بعد اربعة. من هذه النتيجة نستنتج ان كتلة بلانك Planck mass فى بعد D (فى البلك او الجسد) وهى M(d+2) متناسبة مع كتلة بلانك فى بعد اربعة (البراين) وهى MPl حيث ان ثابت التناسب هو بالضبط حجم الابعاد الاضافية yc**d=yc**(D-4). انظر الصورة الخامسة. تذكروا الآن العلاقة التى تربط كتلة بلانك مع ثابت نيوتن من المنشور السابق. نجد اذن النتيجة فى الصورة السادسة التى يمكن التعبير عنها بأن نسبة ratio ثابت نيوتن فى بعد D اى فى البلك وهو G(D) و ثابت نيوتن فى اربعة ابعاد وهو G يساوى بالضبط حجم الابعاد الاضافية اى lC**(D-4). باستخدام النتيجة فى الصورة الثانية و النتيجة فى الصورة السادسة يمكننا التعبير عن محيط الابعاد الاضافية lC بدلالة طول بلانك فى البعد D اى فى البلك lP(D) و طول بلانك فى اربعة ابعاد اى فى البراين lP بالعلاقة فى الصورة السابعة. نختار الآن عدد الابعاد الاضافية d=D-4 و محيط الابعاد الاضافية lC بحيث ان طول بلانك lP فى اربعة ابعاد اى على البراين اين نعيش هو القيمة المعروفة 10 للاس -33 سنتيمتر اما طول بلانك lP(D) فى بعد D اى فى البلك او الجسد فهو يساوى الى اقصى طول تم قياسه فى التجارب المعملية وهو حوالى 10 للاس -18 سنتيمتر. اى اننا نختار كتلة بلانك الاساسية التى هى كتلة بلانك فى البلك تساوى الى 1 TeV اى 1 تيراالكترون فولط وهو سلم scale طاقات فيزياء الجسيمات الاولية وهذا يحل معضلة الهايراركى. نجد مثلا من اجل بعدين اضافيين ان محيط الابعاد الاضافية هو 0,001 سنتيمتر و هذا بعد كبير يمكن قياسه فى التجارب المعملية ولهذا يسمى هذا النموذج بنموذج الابعاد الاضافية الكبيرة. من الواضح ان قوة نيوتن على المسافات الكبيرة جدا بالمقارنة مع محيط الابعاد الاضافية yc تبقى معطاة بنفس القانون الذى نعرفه اى ان الطاقة الكامنة المرفقة بها متناسبة عكسا مع المسافة. اما من اجل المسافات الصغيرة جدا بالمقارنة مع محيط الابعاد الاضافية فان قوة نيوتن سوف ترى الفضاء-زمن الكلى (فضاء-زمن البلك) وتصبح متناسبة عكسا مع المسافة مرفوعة للأس D-3 كما هو مبين فى الصورة الاخيرة.ale طاقات فيزياء الجسيمات الاولية وهذا يحل معضلة الهايراركى.
نجد مثلا من اجل بعدين اضافيين ان محيط الابعاد الاضافية هو 0,001 سنتيمتر و هذا بعد كبير يمكن قياسه فى التجارب المعملية ولهذا يسمى هذا النموذج بنموذج الابعاد الاضافية الكبيرة.
من الواضح ان قوة نيوتن على المسافات الكبيرة جدا بالمقارنة مع محيط الابعاد الاضافية yc تبقى معطاة بنفس القانون الذى نعرفه اى ان الطاقة الكامنة المرفقة بها متناسبة عكسا مع المسافة.
اما من اجل المسافات الصغيرة جدا بالمقارنة مع محيط الابعاد الاضافية فان قوة نيوتن سوف ترى الفضاء-زمن الكلى (فضاء-زمن البلك) وتصبح متناسبة عكسا مع المسافة مرفوعة للأس D-3 كما هو مبين فى الصورة الاخيرة.









الغرافيتون الثقيل كحل للمادة المظلمة


 المادة فى الكون تأتى على شكل مادة مضيئة luminous matter و على شكل مادة مظلمة dark matter.

المادة المضيئة هى الذرات التى تتشكل منها اجسامنا و الارض و النجوم و ما نشاهده من المجرات وهى حوالى 16 بالمائة فقط من المادة الموجودة فى الكون.
اما المادة المظلمة فتشكل ما يقارب ال 84 بالمائة من مادة الكون.
لكننا لا نراها فهو لا يصدر عنها اى اشعاع كهرومغناطيسى ولهذا تسمى مظلمة.
هذه المادة المظلمة لا نرى تأثيرها الا عبر تأثيرها على دوران المجرات اى عبر التفاعلات الثقالية لهذه المادة المظلمة مع المادة المضيئة.
المادة المضيئة توصف فيزيائيا بشكل دقيق الى حد كبير جدا-بل دقيق جدا-باستخدام النموذج القياسى للجسيمات الاولية standard model of particle physics.
أما المادة المظلمة التى هى الاغلبية الساحقة من المادة فى الكون فهى مازالت مجهولة و هناك عدة مقترحات فى فيزياء الجسيمات حول اصلها و كل هذه المقترحات واقعة اليوم فى ازمة.
من هذه المقترحات جسيمات سلمية تسمى الاكسيونات (مفرد اكسيون axion) و الجسيمات الشبحية الشهيرة المسماة النيوترينوات (مفرد نيوترينو neutrino و هى جسيمات العبقرى باولى Pauli).
اما اشهر المقترحات فهى جسيمات تعرف باسم الويمب WIMP اى الجسيمات الثقيلة المتفاعلة بضعف weakly interacting massive particles التى يتنبأ بها النموذج القياسى الممتاز اصغريا minimally supersymmetric standard model للجسيمات الاولية.
كل هذه المقترحات التى سيطرت على المجال خلال ال 25 سنة الاخيرة واقعة اليوم فى ازمة لان المشاكل التى تواجهها اكبر مما حققتها من انجازات و ايضا لانه لم يتم اكتشاف لا التناظر الممتاز و لا اى ويمب فى اى من التجارب الاخيرة للمسرعات.
من المقترحات التى لفتت نظرى بشدة مؤخرا هو مقترح ان المادة المظلمة تتشكل من جسيمات هى غرافيتونات (مفرد غرافيتون graviton) ثقيلة.
هذه الغرافيتونات الثقيلة نحصل عليها من الاختزال البعدى كالوزا-كلاين Kaluza-Klein dimensional reduction للابعاد الاضافية extra dimensions التى سوف نفترض ان الفضاء-زمن ينطوى عليها.
اذن الفضاء-زمن هو ليس اربعة ابعاد بل نفترض ان عدد ابعاده هو اكثر قد تكون خمسة ابعاد وقد تكون d بعد بصفة عامة.
و كتلة الغرافيتون فى الفضاء-زمن الاعلى (اى الفضاء-زمن ببعد خمسة او ببعد d) يجب ان تكون منعدمة وهذا هو شرط الصمود الديفيومورفيزمى diffeomorphism invariance للنسبية العامة فى d بعد.
اذن هناك d-4 بعد اضافى قد تكون كبيرة large او كونية universal او مغلفة wrapped وهذا هو النوع الاهم.
سوف نفترض عموما ان هذه الابعاد الاضافية هى متضامة compact اى متناهية فى الطول مثلا هى دوائر بنصف قطر R.
و عندما نقوم باختزال هذه الابعاد الاضافية باستخدام ميكانيزم كالوزا-كلاين Kaluza-Klein mechanism فان الغرافيتون سوف يظهر من وجهة نظر الفضاء-زمن باربعة ابعاد الذى نعيش فيه على انه برج tower اى سلسة لانهائية من الانماط الحقلية field modes اى جسيمات.
هذه الجسيمات او الانماط الحقلية تسمى غرافيتونات كالوزا-كلاين Kaluza-Klein gravitons وهى كلها ثقيلة اى بكتلة غير معدومة متناسبة عكسا مع نصف القطر R.
هذه العملية تسمى التضميم compactification كما شرحنا ذلك فى منشور سابق و عندما يذهب نصف القطر الى صفر فان التضميم يصبح اختزال بعدى dimensional reduction.
وكلتا العمليتين (التضميم و الاختزال البعدى) نسميهما معا ميكانيزم كالوزا-كلاين او اختزال كالوزا-كلاين Kaluza-Klein reduction.
هذه فى الحقيقة عملية بسيطة.
يمكننا أن نبين ميكانيزم او اختزال كالوزا-كلاين بالنسبة الى حقل سلمى Φ يعيش فى d=D+1 بعد اى لدينا بعد اضافى واحد كما يلى.
نفترض ان البعد الاضافى هو دائرة نصف قطرها R و احداثية البعد الاضافى هى z.
الحقل السلمى Φ يجب ان يكون دورى periodic فى اتجاه البعد الاضافى لانها دائرة. اذن يجب ان تتحقق المعادلة الاولى فى الصورة.
نقوم الآن بتحويل فورييه Fourier transform فى اتجاه البعد الاضافى و هذه هى المعادلة الثانية فى الصورة حيث ان Φn هى الانماط الحقلية field modes وهى حقول سلمية فى البعد الادنى d.
الحقل السلمى Φ فى البعد الاعلى D=d+1 يظهر على شكل حقول سلمية Φn بالنسبة لراصد يعيش فى البعد الادنى d.
الحقل السلمى Φ يجب ايضا ان يحقق معادلة كلاين-غوردن Klein-Gordon بكتلة معدومة فى البعد الاعلى D=d+1.
هذا يؤدى مباشرة الى كون الحقول السلمية Φn يجب هى الاخرى ان تحقق معادلة كلاين-غوردن لكن نجد انها تحقق معادلة كلاين-غوردن بكتل غير معدومة فى البعد الادنى d.
هذه هى المعادلة الثالثة فى الصورة. و كما ترون فان الكتل هى بالضبط متناسبة عكسا مع نصف القطر R.
الحقول Φn تسمى انماط كالوزا-كلاين Kaluz-Klein modes او الحقول السلمية لكالوزا-كلاين.
لو اخذنا عوض الحقل السلمى Φ مترية الفضاء-زمن spacetime metric (التى هى بالضبط مترية الغرافيتون فى الفضاء-زمن الاعلى) فان انماط كالوزا-كلاين التى نحصل عليها فى هذه الحالة هى بالضبط الغرافيتونات الثقيلة كالوزا-كلاين فى الفضاء-زمن الادنى.
المهم ان الغرافيتون المعدوم الكتلة فى بعد اعلى يظهر لنا نحن فى بعد ادنى على شكل عدد لانهائى من الغرافيتونات الثقيلة.
هذه الغرافيتونات الثقيلة هى بالضبط جسيمات المادة المظلمة.
هذه الغرافيتونات يتم انتجاها مباشرة بعد الانفجار الاعظم من تصادم scattering الجسيمات الاولية المضيئة و حسب ما يبدو فانه يتم انتاجها بأعداد كافية تغطى كمية المادة المظلمة التى نراها اليوم فى الكون.
هذه النتيجة لم تكن متوقعة لان هذه التصادمات معروف عنها انها نادرة جدا لكن الذى وجده الباحثون ان هناك تعزيز enhancement فى انتاج هذه الجسيمات لم يكن معروفا من قبل وهذا ما يجعل اعدادها المنتجة بعد الانفجار الاعظم (حوال 1 بيكوثانية picosecond اى واحد من 10 للاس 12 من الثانية بعد الانفجار الاكبر) كافى جدا من اجل تغطية كمية المادة المظلمة التى نشاهدها اليوم فى الكون.
هذه الغرافيتونات الثقيلة يتم انتجاها تحت سلم كتلة جسيم الهيغز Higgs particle (المسؤول عن كتلة الجسيمات المضيئة) فهى تأتى بكتلة اقل من 1 ميغاالكترون فولط.و اذن فان كتلة هذه الغرافيتونات الثقيلة بعيدة عن الطاقات العليا و بالتالى فهى محمية من مشاكل اعادة-التنظيم او الاستنظام renormalization.
اكثر من هذا فان تفاعل هذه الغرافيتونات الثقيلة مع الجسيمات المضيئة هو تفاعل ضعيف جدا وهو تفاعل ثقالى محض وهذا عكس الويمب مثلا الذى رغم ان تفاعله مع الذرات هو ايضا ضعيف الا انه يتفاعل معها عبر القوى الاخرى ايضا.
هذا التفاعل الضعيف بين الغرافيتونات الثقيلة و الجسيمات المضيئة يؤدى ايضا الى استقرار الغرافيتونات الثقيلة اى الى تهافتها البطئ جدا بالمقارنة مع عمر الكون.
للاستزادة المرجع هنا:



معضلة الهايراركى او التراتب فى فيزياء الجسيمات الاولية

 فى اربعة ابعاد (3 فضائية و واحدة زمنية) فان قوة نيوتن بين كتلتين m1 و m2 تكون متناسبة مع جداء الكتلتين m1*m2 و متناسبة عكسا مع مربع المسافة r بين الكتلتين.

القانون موجود فى الصورة الاولى و المعامل G هو ما يسمى ثابت نيوتن للجاذبية الثقالية Newton's constant of gravitational attraction.
هذا الثابت هو ثابت كونى universal constant اى انه لا يتعلق بالكتلتين و لا بالمسافة بينهما فهو نفس الثابت بالنسبة لجميع الكتل و المسافات فى الكون.
جميع التفاعلات الثقالية فى الكون يتحكم فيها هذا الثابت (ثابت نيوتن للجاذبية الثقالية).
القيمة العددية لهذا الثابت فى نظام الوحدات الدولى international system of units او SI اختصارا نضعها فى الصورة الثانية.
فى الصورة الثانية نضع ايضا القيم العددية لسرعة الضوء c (التى تتحكم فى جميع الظواهر النسبية فى الطبيعة) و لثابت بلانك المختزل reduced black constant الذى يرمز له ب hbar\ (الذى يتحكم فى جميع الظواهر الكمومية فى الطبيعة).
اذن لدينا ثلاثة ثوابت اساسية هى ثابت نيوتن G و سرعة الضوء c و ثابت بلانك hbar\.
و هى تقابل ثلاثة وحدات اساسية فى الطبيعة هى الطول L و الزمن T و الكتلة M. فهذه الوحدات هى بالضبط التى تظهر مع القيم العددية فى الصورة الثانية.
فى ما يسمى نظام الوحدات البلانكى Planckian system الذى يتوائم اكثر مع الظواهر الثقالية الكمومية النسبية (وهذا هو مجال الثقالة الكمومية quantum gravity اى لب الطبيعة) فان ثابت نيوتن G و سرعة الضوء c و ثابت بلانك hbar\ يأخذ جميعها بالضبط القيمة 1 كما هو مبين فى الصورة الثالثة.
الطول lp هو طول بلانك Planck length و الزمن tp هو زمن بلانك Planck time و الكتلة mp هى كتلة بلانك Planck mass و يمكن ايجاد القيم العددية للجميع من المعادلتين فى الصورتين الثانية و الثالثة و النتيجة موضوعة فى الصورة الرابعة.
طول بلانك lp هو اصغر طول ممكن فى الكون. وهذا الطول هو الطول الوحيد الذى يمكن بنائه من اخذ جداء الثوابت الاساسية G و c و hbar\ مرفوعة للأسس 1/2 و 3/2- و 1/2 على التوالى (تأكدوا من هذا الامر البسيط).
وزمن بلانك tp هو اصغر زمن ممكن فى الكون. وهو الزمن الذى يستغرقه الضوء لقطع طول بلانك.
و كتلة بلانك mp هو اكبر كتلة ممكنة من وجهة نظر الجسيمات الاولية. الطاقة المقابلة لهذه الكتلة (اى Ep=mp*c**2) هى بالضبط الطاقة الشهيرة باسم طاقة بلانك Planck energy التى تظهر عندها تأثيرات الثقالة الكمومية وهى تساوى تقريبا 10 للأس 19 جيغا-الكترون-فولط.
كتلة بلانك هى اكبر من كتلة البروتون مثلا ب 10 للأس 19 (وهو نفس العدد اعلاه) و هذا عدد هائل.
هذه الملاحظة (لماذا كتلة البروتون هى اكبر من كتلة بلانك بهذا القدرالعظيم يسمى معضلة التراتب hierarchy problem).
وحتى نفهم معضلة التراتب (فهى معضلة عظيمة جدا يعانى منها النموذج القياسى للجسيمات الأولية standard model of elementary particles) علينا طرح السؤال الآتى: ماذا يجب ان تكون قيمة كتلة البروتون حتى لا يكون هناك معضلة تراتب?
بكل بساطة لو كانت كتلة البروتون هى M بحيث ان قوة نيوتن التجاذبية الثقالية بين بروتون و بروتون آخر يبعد عنه بمسافة r تعدم تماما قوة كولوموب Coulomb force التنافرية الكهربائية بين هذين البروتونين فانه لن يكون هناك معضلة تراتب.
اذن يجب ان يكون لدينا تساوى بين طويلتى القوتين الثقالية و الكهربائية كما فى الصورة الخامسة (فى هذه الصورة e هو الشحنة الكهربائية).
هذه المعادلة تؤدى مباشرة الى الشرط على كتلة البروتون M الموجود فى المعادلة السادسة.
اى ان كتلة البروتون M كان يجب ان تكون تساوى تقريبا عشر كتلة بلانك اذا اردنا ان لا يكون هناك معضلة تراتب.
لكن ليس هذا هو الواقع. اذن لماذا. هذا هو السؤال. وهذه هى معضلة التراتب او معضلة الهايراركى.








ثلاثة كتب فى نظرية الحقل الكمومى على الشبكة

 ماهو التخصص الذى يجب ان يختاره الطالب.

هذه قضية وجودية فهى ستحدد مصير الانسان الى سنوات عديدة فى المستقبل و تغيير هذا الاختيار الاول قد يكون صعبا جدا ان لم يكن مستحيلا (بل هو مستحيل).
اولا علينا ان ندرك أن الفيزياء النظرية تنقسم الى قسمين عظيمين.
القسم الاول هو تحليلى-رياضى.
القسم الثانى هو عددى-حاسوبى.
من البداية انصح بعدم اختيار القسم الاول من الاغلبية الساحقة من الطلبة و الطلبة الاساتذة.
بكل بساطة الجزء التحليلى-الرياضى يحتاج الى دراسة مستمرة للفيزياء كتخصص-أول و دراسة مستمرة للرياضيات كتخصص-ثان و قراءة مكثفة للفيزياء و الرياضيات كثقافة موسوعية.
وهذا صعب جدا لن يستطيعه الا الطالب الموهوب الذى يجد نفسه مع استاذ متمكن و فى مجموعة بحثية متكاملة.
القسم الثانى هو العددى-الحاسوبى وهذا هو الذى انصح به.
هذا القسم هو فيزياء نظرية بنكهة تجريبية و صعوبته الوحيدة هى التمكن من الرياضيات التطبيقية و الفيزياء العددية و الحاسوبيات و هذا اسهل نسبيا.
لكن الحقيقة ان هذا الاتجاه هو ليس سهل كما قد يتصور البعض بل هو من نوع السهل الممتنع فهو أمر ليس هين ايضا لكنه اقرب الى مقدرة اغلبية الطلبة و امكانياتهم.
ايضا هذا توجه لا يتطلب قراءة فيزيائية و رياضية مستمرة و مكثفة بل يحتاج الى عمل عددى و حاسوبى مستمر و هذا اسهل نسبيا.
القسم العددى-الحاسوبى يستعمل بصورة اساسية طرق مونتى كارلو Monte Carlo methods وهو يتلخص فى اتجاهين ضخمين:
-نظرية الحقل على الشبكة lattice field theory وهذه النظرية هى احدى اكبر انجازات الفيزياء النظرية فى ال 50 سنة الاخيرة فقد كان هذا الاتجاه هو احد الابواب الكبيرة التى سمحت بتحقيق تقدم كبير فى الحاسوبيات و الخوارزميات و لغات البرمجة وهذا بغض النظر عن الفيزياء (بالخصوص فهم ديناميك القوة النووية اللونية و فهم ديناميك التحولات الطورية من الرتبة الثانية فى الطبيعة).
-نظرية الوتر على الشبكة lattice string theory و هذا اتجاه بدأ يتبلور فى ال 20 سنة الاخيرة فقط و هو يعد يتحقيق انجازات هائلة فى نظرية الثقالة الكمومية وهو المجال الاكبر و الاكثر اساسية فى الفيزياء النظرية و الذى يبقى مجهولا الى حد كبير رغم عمل و اجتهاد كل هؤلاء العباقرة الذين يشتغلون عليه فى الاتجاه التحليلى-الرياضى.
الاتجاه الاول (الحقل على الشبكة) صعب لكنه راسخ اما الاتجاه الثانى (الوتر على الشبكة) فصعب جدا و مجهول و لا يمكنكم العبور الى الاتجاه الثانى الا بعد دراسة وافية للاتجاه الاول.
اضع فى الرابط الكتاب المعيارى فى نظرية الحقل على الشبكة وهو ليس الكتاب الاسهل فى هذا المجال لكنه الكتاب الاكثر دقة.
هناك كتب اخرى مثلا كتاب مايكل كرويتز Michael Creutz المعنون ب (الكواركات, الغليونات و الشبكات Quarks, Gluons and Lattices) و هو كتاب تاريخى و جميل جدا من ناحية فيزياء الجسيمات الاولية -وبخاصة قوة الكروموديناميك او الديناميك اللونى الكمومى او ال QCD.
نؤكد هنا ان المؤلف كرويتز هو فيزيائى نظرى كبير جدا كان و مازال رائد نظرية الحقل على الشبكة حيث ابتدأها لوحده فى السبعينات. رابط كتابه هنا:
هناك ايضا كتاب جميل جدا هو كتاب يان سميت Jan Smith المعنون (مقدمة فى الحقول الكمومية على الشبكة Introduction to Quantum Fields on a Lattice) و هو افضل الكتب عندى شخصيا. فهو رائع بأتم معنى الكلمة من جميع النواحى الفيزيائية و الرياضية و العددية. الرابط هنا:
نصيحة اخيرة.
اذا اردتم تعلم هذه الامور فأول شي عليكم القيام به هو نسيان كل شيء قيل لكم فى مواد الحاسوبيات فى جامعاتنا ثم التعلم من جديد.
فطريقة تعامل الجميع عندنا -على الاقل فى الجزائر- مع هذا الموضوع هى طريقة البريكولاج اى تحقيق نتائج عددية شكلية بأسرع الطرق دون الاكتراث بالجانب الفيزيائى و النظرى و الرياضى وهذا ينم عن عدم فهم حقيقى و تام و عميق للطريقة الحاسوبية فى الفيزياء النظرية.
وقد حاولت شخصيا تصحيح هذا الوضع فى جامعتنا لكننى فشلت تماما فى تحقيق اى تأثير.
رابط كتاب مونتفاى Montvay و مونستر Munster وهو المرجع المعيارى فى نظرية الحقل على الشبكة.

التضميم

 التضميم compactification هى عملية مشهورة فى الفيزياء النظرية (بالخصوص فى فيزياء الاوتار-الممتازة super-strings) تتلخص فى افتراض ان بعض الاتجاهات فى الفضاء-زمن تأتى باطوال صغيرة تقارب عموما طول بلانك Planck length الذى هو اصغر طول فى الكون.

النظرية على الفضاء الكلى هى نظرية اساسية اما النظرية التى نحصل عليها بعد التضميم فهى نظرية فعالة.
بشكل ادق التضميم يعنى ما يلى:
-اولا نأخذ نظرية تعيش على فضاء ما MxC.
-ثانيا نقوم بافتراض ان الفضاء C هو متضام compact اى حجمه متناهى مثلا كرة sphere او طارة torus.
-ثالثا ثم نقوم بالاختزال reduction على هذا الفضاء المتضام C اى نقوم بتفكيك الحقول على MxC (اى نأخذ تحويل فورييه Fourier transform على C من اجل الحصول على الانماط الحقلية field modes التى تبقى تتعلق فقط على الفضاء المتبقى M).
-ثم نقوم بالمكاملة صراحة على اتجاهات الفضاء المتضام C (الانماط الحقلية فى اتجاهات الفضاء المتضام C تصبح حقول بأتم معنى الكلمة على الفضاء المتبقى M).
هذه العملية الاختزالية من MXC الى M تسمى اختزال كالوزا-كلاين Kaluza-Klein reduction.
لأن كالوزا Kaluza و كلاين Klein كانا اول من استعملها من اجل توحيد النسبية العامة و الكهرومغناطيسية فى نظرية واحدة تعيش فى خمسة ابعاد (البعد الخامس هو دائرة بنصف قطر صغير).
عندما يذهب حجم الفضاء المتضام C الى الصفر فان عملية التضميم تتحول الى ما يسمى عملية الاختزال البعدى dimensional reduction.
فى الاختزال البعدى فان الحقول لا تتعلق تماما بالفضاء المتضام C.
عملية التضميم استخدمت ايضا فى فيزياء الجسيمات الاولية حيث ان الفضاء المتضام يسمى فى تلك الحالة بالابعاد الاضافية extra dimensions.
وبافتراض ان هذه الابعاد الاضافية هى ابعاد اضافية كبيرة large extra dimensions (وليس صغيرة) فاننا نجد ان قوة الثقالة تصبح اضعف بكثير من القوى الاخرى اذا كانت هذه الاخيرة محبوسة confined فى الفضاء M اما قوة الثقالة فنسمح لها ان تنتشر فى الفضاء الكلى MxC.
هذه الفكرة الاخيرة جعلت اصحابها (لن اذكر اسمائهم) مشاهير منذ 20 سنة اكثر مما تسبب التضميم فى شهرة كالوزا و كلاين منذ 100 سنة.
المهم ان النظرية التى تعيش فى الفضاء الكلى MXC هى نظرية اساسية fundamental theory اما النظرية التى نحصل عليها بعد التضميم و التى تعيش فى الفضاء M فهى نظرية فعالة effective theory بالمقارنة.


الاشخاص الذين تحصلوا على نوبل مرتين

خمسة اشخاص تحصلوا على نوبل مرتين.

لكن فى الحقيقة علينا احتساب اربعة اشخاص فقط تحصلوا على نوبل مرتين لان الشخص الرابع تحصل على نوبل الثانية فى السلام و ليس فى الفيزياء او الكيمياء.
تذكروا نوبل تأتى فى 5+1 مجالات هى الفيزياء و الكيمياء و الادب و الطب و الاقتصاد (و السلام).
حسب الاهمية برأيى.
اولا جون باردين John Bardeen تحصل مع زملائه على نوبل الاولى بسبب اختراعه الترانزستور (الذى هيمن على ماضى الالكترونيات) و نوبل الثانية بسبب نظريته فى النواقل-الممتازة (التى سوف تهيمن على مستقبل الالكترونيات).
اذن باردين تحصل على نوبل فى الفيزياء مرتين (وهو الوحيد) على عمل تجريبى-تطبيقى و آخر نظرى و هذا من اقوى الاشياء.
هذه الاعمال التى قام بها باردين كانت نتائجها بالنسبة للتكنولوجيا اقوى بكثير من نتائجها بالنسبة للفيزياء.
ثانيا فريديريك سانجغر Frederick Sanger الذى تحصل على نوبل الاولى على اكتشافه بنية الانسولين و تحصل على نوبل الثانية على اكتشافه ال DNA الذى هو أساس كل الفهم البيولوجيى اليوم.
لكن سنجغر الذى كان عمله حاسما بالنسبة للبيولوجيا و الطب فان جائزتيه كانت فى الكيمياء -وهو ثان اثنين حقق ذلك فى الكيمياء على عكس الفيزياء التى لم يحقق فيها ذلك الا باردين-.
ثالثا مارى كورى Marie Curie وهى قد تحصلت على نوبل الاولى فى الفيزياء (1903) و الثانية فى الكيمياء (1911). و هى قد تحصلت على الجائزتين فى الفيزياء و الكيمياء على نفس العمل وهو اكتشاف المواد المشعة التى تلعب دورا هائلا فى المجالين.
و قد كادت مارى كورى الا تحصل على الجائزة الاولى فى الفيزياء -من باب التمييز ضد المرأة- لولا تدخل بعض العلماء المخلصين و من بينهم زملائها فى الاكتشاف.
رابعا كارل بارى شاربلس Karl Barry Sharpless الذى تحصل على نوبل فى الكيمياء مرتين فى ال 20 سنة الاخيرة وهو الوحيد من الخمسة الذي مازال على قيد الحياة.




تدوير وييك

 من اعظم نتائج الفيزياء النظرية أن نظرية الحقول الكمومية النسبية relativistic quantum fields theory ترتبط بالميكانيك الاحصائى (الذى يسمى ايضا نظرية الحقول الاقليدية Euclidean fields theory) بما يسمى تدوير وييك Wick rotation.

و وييك هو نسبة الى الفيزيائى النظرى الايطالى جيان كارلو وييك Gian Carlo Wick الذى استعملها من اجل تحويلات تكاملات فضاء-زمن مينكوسفكى Minkowski spacetime فى نظرية الالكتروديناميك الكمومى quantum electrodynamics او ال QED الى تكاملات فى الفضاء الاقليدى Euclidean space.
الميكانيك الاحصائى او نظرية الحقول الاقليدية هى نظرية تتصرف من الناحية الرياضية بشكل افضل بكثير من نظرية الحقول الكمومية النسبية كما ان الميكانيك الاحصائى مفهوم تماما من الناحية الحاسوبية اى اننا يمكننا تطبيق الخوارزميات و العدديات عليه بشكل مباشر اما نظرية الحقول الكمومية النسبية فلا.
والسر الكبير فى الاختلاف بينهما يكمن فى الزمن و فى درجة الحرارة.
الزمن فعلا نسبى و نظرية الحقول تفهم هذا بالضبط.
اما درجة حرارة نظرية الحقل الكمومى النسبى فهى صفر تماما فكيف اذن يمكن اذن ان نحصل على نظرية الحقل الكمومى النسبى عند درجة حرارة تختلف عن الصفر.
حل هذه المعضلة يكمن فى تدوير وييك.
من الناحية الرياضية تدوير وييك هو تدوير للزمن نحو المحور التخيلى imaginary axis فى المستوى المركب complex plane كما هو مبين فى الصورة الاولى.
اذن المترية metric تنتقل من الشكل الاول الى الشكل الثانى فى الصورة الثانية. الزمن الحقيقى هو t اما τ فهو يسمى الزمن التخيلى او الزمن الاقليدى.
عندما نقوم بهذا الامر (بهذا التدوير) فان الفعل action فى نظرية الحقول الكمومية النسبية (الصورة الثالثة) يتحول الى الطاقة فى الميكانيك الاحصائى (الصورة الرابعة).
واكثر من هذا فان تكامل طريق فايمان Feynman path integral فى نظرية الحقول تتحول الى دالة تقسيم بولتزمان Boltzmann partition function فى الميكانيك الاحصائى.
الشيء الوحيد الذى يجب التنبه اليه هو انه يجب المكاملة الآن فى تكامل طريق فايمان ليس على طرق paths كيفية بل على طرق فى فضاء التشكيل configuration space هى دورية periodic فى الزمن الاقليدى τ و دورها يساوى بالضبط مقلوب درجة الحرارة β التى تظهر فى دالة تقسيم بولتزمان.
انظر المعادلة الاخيرة فى الصورة الخامسة.
خطوات تدوير وييك مبينة فى الصورة الخامسة.
تدوير وييك هى ليست حيلة رياضية بل هى مبرهنة ضخمة فى نظرية الحقول البديهية axiomatic field theory حيث ان فيزيائيين نظريين من الوزن الرياضى الثقيل جدا جدا بينوا النتيجة العظيمة التالية التى ينبنى عليها جزء عظيم من نظرية الحقول الكمومية:
(بديهيات ويختمان Wightman axioms التى تُعطى دوال غرين Green's functions فى نظرية الحقل الكمومى النسبى الذى هو فى حالة توازن حرارى عند درجة حرارة ثابتة T تذهب بالضبط تحت تأثير تدويك الى بديهيات اوستروالدر-شرايدر Osterwalder-Schrader acioms التى تُعطى دوال الربط correlation functions فى نظرية الحقل الاقليدى حيث ان مقلوب درجة الحرارة اى β هو بالضبط دور الزمن الاقليدى τ).
لهذا فنحن نقوم بتدوير وييك فهو ليست حيلة حسابية بل هى رياضيات مؤسسة على ادق المعايير.
انظروا الصورة الاخيرة.








الحاسوبية الكمومية و محاضرات فايمان حول الحاسوبية

كتاب فايمان حول الحاسوبية هو الذى ابتدأ اهتمام الفيزيائيين النظريين بالحاسوبية و علومها.
هذه الدراسة فى الرابط كنت قد قمت بها عام 2005 اعتمادا على مرجع واحد هو محاضرات فايمان هذه حول الحاسوبية الكلاسيكية classical computation و المعلوماتية الكلاسيكية classical information من وجهة النظر الفيزيائية و تعميمهما نحو الحاسوبية الكمومية quantum computation و المعلوماتية الكمومية quantum information.
اتذكر انها كانت تجربة ممتعة جدا فى وقت كنت فيها اكثر شبابا و اكثر املا و اكثر طموحا و اكثر اجتهادا و اكثر فى كل شيء.
اذن الدراسة فى الرابط هى خلاصة دراستى لهذا الموضوع فى ذلك الوقت او بالاحرى هى خلاصة دراستى لكتاب فايمان. و قد كفتنى هذه الدراسة لوقت طويل.
كتاب فايمان يمكنكم تحميله من هنا
كتاب فايمان هو فى الحقيقة مجموع محاضرات فايمان حول الحاسوبية الذى قدمه فى اواسط الثمانينات فى جامعة كالتاك Caltech و الذى كان نقطة الانطلاق نحو الحاسوبية الكمومية بالنسبة للفيزيائيين النظريين بالخصوص.
فايمان قام فى هذا الكتاب بتفكيك اسس علوم الحاسوب تفكيكا تاما ثم اعاد تركيب هذه الاسس بنظرة فيزيائية تهدف الى الوصول الى الحاسوبية الكمومية اى كيفية تعميم الحاسوبية الكلاسيكية نحو الحاسوبية الكمومية.
العقيدة الاساسية فى علوم الحاسوب بين علماء الحاسوب الاوائل (المدارس الانجلوسكاسونية) هى ان (المعلوماتية وبالتالى الحاسوبية هى رياضيات) لا اقل و لا اكثر.
وكما قال دونالد كنوث Donald Knuth وهو رياضى و هو احد آباء علماء الحاسوب (ومخترع لغة البرمجة الشهيرة لايتاك Latex) ان علوم الحاسوب مثل الرياضيات تهتم باختراع قوانين اصطناعية على عكس الفيزياء التى تهتم باكتشاف قوانين طبيعية.
لكن عندما دخل الفيزيائيون النظريونن مع فايمان على الخط و رأوا ان كل ذلك العلم (علوم الحاسوب) هو تقريب فقط لشيء اعمق و ادق (هو الحاسوبية الكمومية) تحولت العقيدة الاساسية الى فكرة ان (المعلوماتية هى فيزياء) قبل كل شيء آخر كما صرح بذلك دايفيد دويتش David Deutsch احد رواد الحاسوبية الكمومية و ايضا فلسفة الكمومى (ذات العلاقة الوطيدة جدا لمن فهم الموضوع فعلا).
بعد ذلك جاء ويلر Wheeler -استاذ فايمان الاكثر عبقرية من فايمان- و قلب الطاولة على الجميع و نص على ان (الفيزياء هى معلوماتية و ليس العكس و المعلوماتية ليست رياضيات بل هى جوهر الوجود).
هذه العقيدة هى المشهورة باسم (الشيء من البت it from bit) وتذكروا فان البت bit هو وحدة المعلومات الكلاسيكية كما ان الكيوبت qubit هو وحدة المعلومات الكمومية.
هذه الفكرة هى التى تطغى اليوم -ضمنيا- فى المعلوماتية الكمومية و نظرية الاوتار الممتازة اللذان انصهرا فى بوتقة واحدة هدفها -بالاضافة الى فهم المعضلات الاساسية للحاسوبية الكمومية و المعلوماتية الكمومية ذات التأثيرات التطبيقية-هدفها ايضا غايات اساسية اخرى فى غاية الصعوبة و التعقيد من بينها فهم الثقوب السوداء و الكوسمولوجيا الكمومية و ماهية الزمن الكمومى و البحث عن نظرية الثقالة الكمومية.
و الادوات الاساسية المستعملة فى تحقيق هذه الاهداف هى ميكانيك كمومى متقدم جدا جاءت به الحاسوبية الكمومية و المعلوماتية الكمومية على مدار ال 20 سنة الاخيرة لم تستطع ان تأتى به الفيزياء الذرية و لا فيزياء الجسيمات الاولية على مدار ال 80 سنة الاولى من عمر ثورة الميكانيك الكمومى على يد بوهر و هايزنبرغ و شرودينغر و ديراك و باولى و بورن.
الاستثناء الوحيد هو مبرهنة بال Bell's theorem التى ساهمت بها فيزياء الجسيمات الاولية (لان بال Bell جاء من رحم فيزياء الجسيمات) و لم تستطع لا الفيزياء الذرية و لا الحاسوبية الكمومية مضاهاتها.
اذن هذه كلها اشياء مترابطة الى حد يعجز اللسان عن وصفه (الميكانيك الكمومى, الحاسوبية الكمومية, المعلوماتية الكمومية, نظرية الاوتار الممتازة, الثقوب السوداء الكمومية, الكوسمولوجيا الكمومية, نظرية الثقالة الكمومية و أسس و فلسفة الميكانيك الكمومى).


الحاسوبية الكمومية

هذا الكتاب اضعه مرة اخرى للتحميل لاهميته القصوى.

الموضوع هو (الحاسوبية الكمومية quantum computation و المعلوماتية الكمومية quantum information) وهما من اهم مواضيع الفيزياء النظرية بالدرجة الاولى و علوم الحاسوب بالدرجة الثانية حاليا.
وهو الموضوع الذى سيؤدى الى الذكاء الاصطناعى الحقيقى (لو كان ذلك ممكنا فعلا) الذى يتحدث و يتحمس له الجميع.
المؤلفان هما نيلسون Nielsen و تشانغ Chuang و كتابهما هو اهم كتاب فى هذا المجال (هذه هى الطبعة العاشرة).
أهم ما استرعى انتباهى فى المقدمة هو المقترح من المؤلفين فى استخدام هذا الكتاب فى تدريس (الميكانيك الكمومى) و ليس (الحاسوبية الكمومية) او (المعلوماتية الكمومية).
الميكانيك الكمومى شيء اساسى فى الفيزياء و الفيزياء النظرية و لكنه شيء معقد و استخدام طرق الحاسوبية الكمومية فى تقديمه و تدريسه هى فكرة اقل ما يقال فيها انها فكرة عبقرية.
الميكانيك الكمومى ندرسه عموما انطلاقا من رياضيات المعادلات التفاضلية الجزئية و من اعتبارات الفيزياء الذرية. و هذان المنطلقان هما فى الحقيقة شيئين معقدين جدا فى حد ذاتهما بالنسبة للطالب و بالنسبة للاستاذ يزيدان فى تعقيد المادة.
المعادلات التفاضلية الجزئية و الفيزياء الذرية هى اشياء قديمة ايضا و هذا ليس انتقاصا منهما اذن يجب العناية الشديدة بهما دون اى تقصير.
لكن النقطة من المؤلفين هو تدريس الميكانيك الكمومى باستخدام اعتبارات الحاسوبية الكمومية و هى اعتبارات حديثة و باستخدام رياضيات الجبر فى فضاء هيلبرت وهذا فعلا اقرب الى الروح الحقيقية للميكانيك الكمومى.
المؤلفان يقترحان كبرنامج للميكانك الكمومى لمدة فصل واحد الفصول التالية من الكتاب:
-الفصل الثانى الذى هو مدخل الى الميكانيك الكمومى.
-الفصل الرابع و يعرض فيه المؤلفان نموذج الدارة الكهربائية الكمومية quantum electric circuit (وهو اهم فصل فى الكتاب بالنسبة للمبتدئ و لغير المبتدئ).
-الفصل الخامس يعرض فيه الخوارزمية الكمومية الاولى (تحويل فورييه الكمومى quantum Fourier transform).
-الفصل السادس يعرض فيه الخوارزمية الكمومية الثانية خوارزمية (البحث الكمومى quantum search).
-الفصل السابع حول كيفية بناء الحاسوب الكمومى اى التنفيذ الفيزيائى physical realization للحاسوب الكمومى.
-ثم يقترح المؤلفان اى فصل من الجزء الثالث من الكتاب واننى اقترح الفصل ال 11 حول الأنطروبى entropy.
اذن هذه فكرة جيدة.
لكن بعد تفكير كثير ارى الآن ان الحاسوبية الكمومية لا يمكن ان تعوض فعلا الميكانيك الكمومى على الاقل ليس بالطريقة التى يقترحها المؤلفان اعلاه.
لكن يمكننا ان نعوض مادة (الفيزياء الذرية او الفيزياء الصلبة او الفيزياء النووية او اى تطبيق آخر للميكانيك الكمومى) بالحاسوبية الكمومية (وليس بالمعلوماتية الكمومية لانها موضوع اعقد).
المؤلفان يقترحان البرنامج التالى من الكتاب كمادة حاسوبية كمومية لفصل واحد:
-الفصل الثانى مدخل الى الميكانيك الكمومى.
-الفصل الثالث مدخل الى علم الحاسوب.
-الفصل الرابع نموذج الدارة الكهربائية الكمومية.
-الفصل الخامس خوارزمية (تحويل فورييه الكمومى).
-الفصل السادس خوارزمية (البحث الكمومى).
-الفصل السابع التنفيذ الفيزيائى للحاسوب الكمومى.
-الفصل الثامن الذى يقوم بعرض مشكلة (الضجيج الكمومى quantum noise) التى تؤثر سلبا على الحاسوبية الكمومية.
-الفصل التاسع الذى يقوم بعرض مقاييس المسافة distance measures الخاصة بالمعلومات الكمومية اى كيف نقارن مدى تساوى او اختلاف معلومتين كموميتين.
-الفصل العاشر شفرات تصحيح-الخطأ الكمومية quantum error-correcting codes التى تستعمل للتحكم فى الضجيج الكمومى.
ازى ان هذا البرنامج طويل جدا بالنسبة الى فصل واحد.
اذا تصورنا ان الطلبة الذين سيأخذون هذه المادة قد درسوا الميكانيك الكمومى اذن يمكن ان نتخلص من عبأ تقديم الميكانيك الكمومى.
الفصل الثالث حول علم الحاسوب نحاول اما الغاءه او اختزاله بشكل او بآخر و الرجوع اليه كلما احتجنا شيء.
اذن نبدأ من الفصل الرابع حول الدارة الكمومية الكهربائية ثم الفصلين الخامس و السادس حول خوارزمية تحويل فورييه الكمومى و خوارزمية البحث الكمومى. هذه الفصول الثلاثة هى هى اهم شيء على الاطلاق. و اذا قمنا بهم فى فصل واحد فقد ارضينا العلم و الضمير و اى شيء آخر هو زيادة فى الخير.
بعد ذلك يمكن ان ننتبه الى الفصل الثامن الخاص بالضجيج الكمومى و الفصل العاشر الخاص بشفرة تصحيح-الخطأ الكمومية. هذا ثانى أهم شيء حسب رأيى.
الفصل الخاص (الفصل التاسع) بمقاييس المسافة اما نلغيه اذا كان ذلك ممكنا او نختزله الى اقصل حد.
اظن ايضا ان الفصل الحادى عشر من نظرية المعلومات الكمومية الخاص بالانطروبى مهم جدا اذا تمكنا من الوقت فعلينا ان نقوم به.
المؤلفان يقدمان ايضا مقترح لتدريس مادة المعلوماتية الكمومية مدتها فصل واحد من هذا الكتاب.
لكن برأيى هذا امر اعقد بكثير فالمعلوماتية الكمومية اقرب الى الرياضيات مثلما ان الحاسوبية الكمومية هى اقرب الى الفيزياء النظرية.
اذن ارى انه يجب تدريسها بعد تدريس الحاسوبية الكمومية التى تحتوى على الامثلة الفيزيائية و الرياضية و الحاسوبية و ليست قضية تجريد و تنظير فقط.
اؤكد فى الاخير ان (الحاسوبية الكمومية) هى ميكانيك كمومى (بل هى ميكانيك كمومى متقدم) ليس لها اى علاقة بالفيزياء الحاسوبية computational physics او الحسبنة العلمية scientific computing اذن لا يجب الخلط بينهما.
الحاسوبية الكمومية هى موضوع غريب ايضا من جانب كونها رغم انها ميكانيك كمومى متقدم كما ذكرت الا انه ايضا يمكن فهمها على انها تطبيق للميكانيك الكمومى (وفى هذا فهى تتفوق على جميع تطبيقات الميكانيك الكمومى من مثل المادة الصلبة الى الفيزياء الذرية الى الفيزياء النووية الى فيزياء الجسيمات الخ).

https://drive.google.com/file/d/18Abpy5m8Dv0mZ1XLMNMYSsEEo5bnugxk/view?usp=sharing


معضلة المضاعفة الفرميونية

 لو قمنا بحساب الطاقة باستخدام معادلة ديراك لوجدنا أن الطاقة تعطى بعلاقة اينشتاين الشهيرة.

نفترض للتبسيط بعد فضائى واحد. التعميم سيكون مباشر للابعاد العليا.
بالنسبة للجسيمات ذات الكتلة المعدومة اى ذات السرعة المساوية لسرعة الضوء (اى الجسيمات الكايرالية chiral particles التى تدور اما الى اليمين او الى اليسار و ليس هناك امتزاج بين اليدوانية-اليمينية right-handedness و اليدوانية -اليسارية left-handedness) اذن بالنسبة لهذه الجسيمات فان علاقة الطاقة لاينشتاين تنص على ان الطاقة متناسبة مع زائد او ناقص قيمة كمية الحركة.
اذن هناك حلان (الخطان الاسودان فى الصورة).
نقوم الآن بحل معادلة ديراك على الشبكة الاقليدية Euclidean lattice.
فى هذه الحالة نجد ان الطاقة تعطى بقيمة جب او سينوس sin كمية الحركة و ليس بقيمة كمية الحركة.
اذن الطاقة دالة دورية periodic function فى كمية الحركة (وليست دالة خطية) وهى معرفة فى مجال يسمى منطقة بريوان Brillouin zone.
الحلان الموجب و السالب هما الخطان الاحمران فى الصورة.
نحصل اذن بالاضافة الى الحل الفيزيائى (من الجهة الموجبة) القريب من نقطة المبدأ (انظر نقطة تقاطع الخط الأسود مع الخط الأحمر) على حل آخر غير فيزيائى يسمى الفرميون المضاعف fermion doubler عند حدود منطقة بريوان من الجهة الموجبة (انظر النقطة السوداء عند حدود منطقة بريوان).
بنفس الطريقة نحصل بالاضافة الى الحل الفيزيائى (من الجهة السالبة) القريب من نقطة المبدأ على حل آخر غير فيزيائى عند حدود منطقة بريوان من الجهة السالبة (النقطة السوداء الاخرى عند حدود منطقة بريوان).
بكل بساطة الفرميون المضاعف هو فرميون مغشوش يعيش عند كميات الحركة الكبيرة اما الفرميون الحقيقى فهو الحل الذى يعيش عند كميات الحركة الصغيرة.
الحل المضاعف غير موجود فى النظرية المستمرة continuum theory (اى عند حل معادلة ديراك فى الفضاء-زمن العادى) و لهذا فهو غير فيزيائى.
لكن الفرميون المضاعف غير الفيزيائى يتميز بنفس طاقة الفرميون الكايرالى chiral fermion الفيزيائى الذى انطلقنا منه (كما ترون من الصورة) لكن يتميز بكايرالية chirality معكوسة (اذا كان الفرميون الكايرالى يدور الى اليمين فان الفرميون المضاعف يدور الى اليسار و العكس).
اذن فى بعد واحد نحصل على فرميونين عوض الفرميون الاصلى اما فى بعد اربعة فاننا نحصل على 16 فرميون عوض الفرميون الاصلى وهذه معضلة كبيرة تسمى معضلة المضاعفة الفرميونية fermion doubling problem.
القاعدة العامة اننا سوف نحصل على 2 للاس d فرميون فى فضاء-زمن ببعد d.
لو ناقشنا الامر من منطلق منتشر ديراك Dirac propogator (الذى يمكن فهمه على انه مقلوب مؤثر ديراك Dirac operator الذى يظهر مؤثرا على سبينور ديراك Dirac spinor فى معادلة ديراك Dirac equation) فان هذه الفرميونات المضاعفة تظهر على شكل اقطاب poles فى منتشر ديراك. اذن نحصل على 16 قطب و فقط قطب واحد حقيقى فيزيائى اما البقية فهى كلها مغشوشة غير فيزيائية. أكثر من هذا فان هذه الاقطاب تأتى بكايراليات (مفرد كايرالية chirality) متعاكسة. انظر الصورة الثانية.
وتذكروا فان الطاقة (طاقة الجسيم) تحسب مباشرة من القطب و القطب هو نقطة عدم تعريف المنتشر.
هذه الفرميونات الاضافية تأتى اذن بكايراليات متعكاسة محافظين بذلك -شكليا فقط- على التناظر الكايرالى chiral symmetry للنظرية.
الحقيقة ان هذه الفرميونات المضاعفة تدمر بالكامل اهم خاصية للتناظر الكايرالى وهو الشذة الكايرالية chiral anomaly التى تلعب دورا اساسيا فى فيزياء الجسيمات الاولية.
وهذه قصة اخرى معقدة جدا حقيقة تحتاج الى شرح لوحدها ان شاء الله.
هذه المعضلة (معضلة المضاعفة الفرميونية) تنص بكل بساطة على انه لا يمكننا ان نضع فرميون كايرالى chiral fermion على الشبكة الاقليدية و بالتالى لا يمكن وضع نظرية كايرالية chiral theory (مثلا النموذج القياسى للجسيمات الاولية) على الشبكة.
تذكروا فان التفاعلات الكهروضعيفة electroweak (عكس تفاعلات الكروموديناميك chromodynamics الكمومى) هى تفاعلات كايرالية فى النموذج القياسى. حيث ان الفرميونات تأتى بكايرالية معينة و بعضها (مثلا النوترينو) يأتى بكايرالية واحدة (يدوانية-يسارية دائما).
معضلة المضاعفة الفرميونية هى معضلة ذات اساس رياضى فهى ليست مصادفة فهناك مبرهنة نيلسون-نينوميا Nielsen-Ninomiya theorem التى تنص على انه اذا كان مؤثر ديراك يتميز بالخواص التالية:
-موضعى local.
-هرميتى Hermitian (اى حقيقى) .
-الصمود الانسحابى traslational invariance.
فانه لا يمكننا تجنب معضلة المضاعفة الفرميونية بدون كسر التناظر الكايرالى. انظر الصورة الثالثة.
اذن هذه مشكلة عظيمة لاننا لا نستطيع التخلى عن التناظر الكايرالى الذى تعتمد عليه كثير من الفيزياء و فى نفس الوقت فانه لا يمكن تحمل مضاعفة الفرميونات لان كثير من النظريات الفيزيائية (و على رأسها النموذج القياسى) هى فعلا نظريات تتميز بدرجات حرية degrees of freedom تعطى فعلا و حقيقة بفرميونات كايرالية.
هناك كثير من الحلول و اول الحلول كان حل كناث ويلسون Kenneth Wilson -احد اعظم الفيزيائيين النظريين فى القرن العشرين- الذى جاء به فى السبعينات وهو يعتمد على تعديل مؤثر ديراك بحيث نقضى تماما على المضاعفة الفرميونية لكن فى نفس الوقت نعدل فى شكل التناظر الكايرالى.
هذا الحل الاول و الاشهر يعرف باسم فرميون ويلسون Wilson fermion.
اذكر هنا ان احد نتائج رسالة الدكتوراة التى كنت قدمتها منذ اكثر من 20 سنة هو كان حل معضلة المضاعفة الفرميونية باستخدام ما يسمى الفرميون الغائم fuzzy fermion.




سبينورات ديراك, ماجورانا و وايل

جميع الجسيمات الاولية التى تشكل المادة المضيئة luminous matter فى الكون هى فرميونات (مفرد فرميون fermion) اى تتميز بعزم لف او سبين spin يساوى نصف و بالتالى فانها تعطى رياضيا بما يسمى السبينور spinor (وهو اسم مشتق من كلمة السبين).
فيزيائيا هناك نوع واحد من السبينورات (مفرد سبينور) وهو سبينور ديراك Dirac spinor لكن رياضيا هناك فى الحقيقة ثلاثة انواع من السبينورات المختلفة جدا.
بالاضافة الى سبينور ديراك هناك سبينور وايل Weyl spinor (جسيم معدوم الكتلة) و سبينور ماجورانا Majorana spinor (جسيم معدوم الشحنة).
كما سنبين فان سبينور وايل و سبينور ماجورانا نحصل عليهما انطلاقا من سبينور ديراك باستخدام شرط وايل Weyl condition و شرط ماجورانا Majorana condition على التوالى.
سبينور ديراك يحل معادلة ديراك و هو كائن باربعة مركبات (لكنه ليس شعاع رباعى) وهو دالة موجة جسيم نقطى بسبين نصف و كتلة m و شحنة q.
أما سبينور وايل فهو سبينور بمركبتين (لكنه ليس شعاع ثنائى) و هو دالة موجة جسيم نقطى بسبين نصف لكن كتلة صفر و شحنة q.
مركبات سبينورى ديراك و وايل هى اعداد مركبة complex numbers (فى التكميم الاول فى الميكانيك الكمومى النسبى) تصبح اعداد غراسمانية Grassmannian numbers (فى التكميم الثانى فى نظرية الحقول الكمومية).
الفرق الاساسى بين ديراك و وايل هو انعدام الكتلة فى وايل و عدم انعدامها فى ديراك.
اذن عندما نجعل الكتلة تذهب الى صفر فان سبينور ديراك يتفكك الى سبينورى وايل واحد يسمى يدوانى-يمينى right-handed و الآخر يسمى يدوانى-يسارى left-handed.
انظر المعادلة الاولى فى الصورة الاولى.
و انظر ايضا الى المعادلتين الثالثة و الرابعة فى الصورة الاولى كيف يتحول سبينورى وايل اليدوانى-اليمينى و اليدوانى-اليسارى تحت تأثير تحويلات لورنتز Lorentz transformations لنظرية النسبية الخاصة.
حتى نفهم الفرق بين اليدوى-اليمينى و اليدوى-اليسارى علينا ان نقوم بتعريف المقدارين الفيزيائيين التاليين -و اغلب الطلبة و حتى الاساتذة يخلط بين هذين المقدارين و يعتقد انهما نفس الشيء-:
- الخاصية الأولى هى الهيليسينية helicity و هى مركبة او اسقاط عزم اللف او السبين على اتجاه كمية الحركة momentum.
اذا كان هذا الاسقاط موجب نقول ان الهيليسينية هى يدوانية-يمينية و اذا كان هذا الاسقاط سالب نقول ان الهيليسينية هى يدوانية-يسارية. انظر الصورة الثانية.
الهيليسينية -من التعريف- هى اذن خاصية محفوظة conserved فى الزمن او ثابت للحركة constant of the motion لان مؤثر الهيليسينية يتبادل commute مع مؤثر الطاقة او مؤثر الهاميلتونية.
-الخاصية الثانية هى الكايرالية chirality وهى تعبر عن كيفية تحول السبينور تحت تأثير تحويلات لورنتز: هل يتم التحول فى التمثيلة الدورانية-اليسارية (هذه هى بالضبط المعادلة الثالثة فى الصورة الاولى) او يتم التحول فى التمثيلة اليدوانية-اليمينية لزمرة لورنتز (هذه هى بالضبط المعادلة الرابعة فى الصورة الاولى).
و تذكروا فان زمرة لورنتز Lorentz group هى مجموعة جميع تحويلات لورنتز بالاضافة الى الدورانات.
اما تمثيلات الزمرة group representations فهى جميع الحلول الممكنة لجبرية الزمرة group algebra.
اذن من الواضح جدا ان الكايرالية على عكس الهيليسينية هى خاصية صامدة invariant تحت تأثير زمرة لورنتز.
بالفعل فان الهيليسينية يمكن عكسها بالذهاب الى معالم عطالية اسرع من الجسيم قيد الدراسة حيث ان الراصد فى هذه المعالم سوف يرى الجسيم يبتعد عنه و بالتالى فان هيليسينية الجسيم سوف تنعكس.
كما ترون فان الكايرالية هى مقدار تناظرى اما الهيلسينية فهى مقدار حركى و عليه فان الكايرالية هى اكثر اساسية.
من الجهة الاخرى فان الكايرالية هى ليست ثابت للحركة مثل الهيليسنية. بالفعل فان جسيم يدوانى-يمينى يمكن ان يتطور فى الزمن الى جسيم يدوانى-يسارى.
الكايرالية و الهيليسينية ينطبقان على بعضهما البعض فقط من اجل الجسيمات معدومة الكتلة (وهذا هو مصدر الخلط الموجود فى فهم و تمييز هذين المقدارين).
من اجل هذه الجسيمات معدومة الكتلة التى تتحرك ضرورة بسرعة تساوى سرعة الضوء فان الهيليسنية تصبح صامد لورنتزى لانه لا يوجد راصد يمكن ان يتحرك بسرعة اكبر من سرعة الضوء.
لهذا فانه عندما تنعدم الكتلة فان سبينور ديراك يتفكك كما ذكرنا الى سبينورى وايل مستقلان تماما عن بعضهما البعض لا يمكن لاحدهما ان يتطور فى الزمن نحو الاخر مادامت الكتلة معدومة كلاسيكيا و بافتراض انها تبقى معدومة كموميا.
فى هذه الحالة نقول انه لدينا صمود كايرالى chiral invariance حيث ان الهيلسينية هى بالضبط الكايرالية وهما (مقدار ثابت للحركة محفوظ) و فى نفس الوقت هما (مقدار صامد تحت تأثير زمرة لورنتز).
التحويلات الكايرالية chiral transformations تولدها مصفوفة ديراك الخامسة γ5 (وتنطق غاما-فايف gamma-five) وهى ربما مصفوفة ديراك الاشهر.
فى الحقيقة فان الكايرالية هى بالضبط القيمة-الذاتية eigenvalue لمصفوفة ديراك الخامسة γ5 التى تسمى مؤثر الكايرالية chirality operator.
اذن السبينور اليدوانى-اليمينى يتميز بكايرالية تساوى +1 (قيمة ذاتية لغاما-فايف تساوى +1) اما السبينور اليدوانى-اليسارى فيتميز بكايرالية تساوى -1 (قيمة ثانية لغاما-فايف تساوى -1).
استخدم المعادلة فى الصورة الثالثة لتبيان هذا الامر (تذكر ان مربع غاما-فايف يساوى واحد).
من هذه المعادلة فى الصورة الثالثة يمكننا ايضا تعريف سبينور وايل انطلاقا من سبينور ديراك بالتأثير على هذا الاخير بالمسقطات projectors اليدوانية-اليمينية PR و اليدوانية-اليسارية PL و المعادلة التى نحصل عليها تسمى شرط وايل Weyl condition.
الصمود الكايرالى chiral invariance هو مكسور بالشذة الكايرالية chiral anomaly فى نظرية الحقول الكمومية و هذا التناظر يلعب احد ادوار البطولة فى الفيزياء النظرية.
آخر انواع السبينورات هو سبينور ماجورانا وهو سبينور حقيقى يصف جسيم بدون شحنة.
اذن الانتقال من سبينور ديراك الى سبينور ماجورانا يتطلب اعدام الشحنة الكهربائية مثلما ان الانتقال من سبينور ديراك الى سبينور وايل يتطلب اعدام الكتلة.
سبينور ماجورنا يعطى باربعة مركبات حقيقية كما فى الصورة الاخيرة حيث نقوم اولا بتعريف السبينور مصروف الشحنة charge conjugated spinor اى السبينور الذى نحصل عليه بتطبيق مؤثر تصريف الشحنة charge conjugation الذى نرمز له ب C.
السبينور مصروف الشحنة يعبر عن جسيم يتميز بجميع خواص السبينور الاول باستثناء ان شحنته تأتى معكوسة. اذن هو يعبر عن الجسيم المضاد للجسيم الاول.
نقوم الآن بمساواة السبينور (الجسيم) بالسبينور مصروف الشحنة (الجسيم المضاد) و النتيجة هو سبينور محايد كهربائيا اى لا يحمل اى شحنة كهربائية.هذه المساواة هى الشرط الشهير باسم شرط ماجورانا Majorana condition.
هذا هو سبينور ماجورانا (الذى يتميز باربعة مركبات حقيقية) وهو حل لمعادلة اخرى للسبينورات تسمى معادلة ماجورانا Majorana equation وهو يمكن ان يعطى ايضا بدلالة سبينور وايل (الذى يتميز بمركبتين مركبتين) الذى هو حل لمعادلة اخرى للسبينورات هى معادلة وايل Weyl equation.
كل هذه الامور التى ذكرناها تصلح فى اربعة ابعاد و لو قمنا بتغيير الابعاد فان كثير من هذه الاشياء التى ذكرناها سوف تتغير فالسبينور -على عكس بقية الحقول- حساس جدا للبعد الذى يعيش فيه.





تعجيز ديراك لأولر

عملية الاشتقاق (بالاضافة الى العملية العكسية: عملية التكامل) تُعول عليها جميع الفيزياء و الرياضيات.
و الاشتقاق كان قد اكتشفه كل من أب الفيزياء نيوتن Newton و الفيلسوف الفذ ليبنيز Leibniz فى نفس الوقت و لم تُحسم الى غاية يومنا هذا قضية الى من ترجع الاسبقية فى هذا الاكتشاف الذى ابتدأت به الفيزياء الحديثة.
ثم جاء الفيزيائى اولر Euler و اكتشف ابسط و اقدم خوارزمية عددية لحساب المشتقات مازالت تستعمل الى غاية يومنا هذا.
و اولر كان عبقرى ظهرت عليه علامات النبوغ منذ كان طفلا و هو اذن لا يحتاج الى خورزمة و حوسبة المشتقة فهو استطاع و يستطيع حساب اشياء معقدة جدا من الناحيتين الفيزيائية و الرياضية بطريقة تحليلية لكنه اكتشف طريقته العددية فى حساب المشتقة لانه كان يعتقد ان العدديات و الخوارزميات و الحاسوبيات هى من كمال الرياضيات و ليست فقط التحليليات و التجريديات و البنائيات.
و تذكروا فان الرياضيات بكل فروعها هى لغة الفيزياء.
طريقة اولر فى الصورة الاولى.
لاحظوا فان طريقة اولر تطبق بالخصوص على المعادلات التفاضلية من الرتبة الاولى first order differential equations اى تحتوى على الاشتقاق من الرتبة الاولى.
ثم تقدمت الفيزياء اكثر بعد نيوتن و ليبنيز و اولر و جاءت اول المعادلات الكمومية العظيمة (معادلة شرودينغر Schrodinger equation) على يد احد آباء الميكانيك الكمومى (شرودينغر Schrodinger) و التى ابتدأت ما يسمى الميكانيك الكمومى الموجى.
معادلة شرودينغر وهى فى اغلبها معادلة تفاضلية من الرتبة الثانية second order differential equation (اى تحتوى على مشتقات من الرتبة الثانية فى الفضاء و مشتقة من الرتبة الاولى فى الزمن) ومع هذا فانه يمكن تطبيق خوارزمية اولر عليها بشكل او بآخر بدون اى مشاكل.
اول معادلة كمومية نسبية هى معادلة كلاين-و-غوردن Klein-Gordon equation التى هى معادلة تفاضلية من الرتبة الثانية فى كل من الفضاء و الزمن و مع هذا فانه يمكن تطبيق خوارزمية اولر عليها ببعض التعديل بدون اى مشاكل ايضا.
يأتى بعد شرودينغر و كلاين و غوردن العملاق الكمومى الآخر و آجد آباء الميكانيك الكمومى (ديراك Dirac) و يكتشف معادلته الشهيرة (معادلة ديراك Dirac equation) التى هى المعادلة الكمومية النسبية الصحيحة التى تصف الجسيمات التى تتميز بعزم لف او سبين spin يساوى نصف.
هذه المعادلة موجودة فى الصورة الثانية.
هذه المعادلة هى معادلة تفاضلية من الرتبة الاولى فى كل من الفضاء و الزمن و مع هذا فاننا لا نستطيع ان نطبق عليها خوارزمية اولر بأى شكل بسيط كما سنرى.
بل ان تطبيق خوارزمية اولر على معادلة ديراك يتطلب قدر هائل من التعقيد و جميع الحلول المطروحة الى غاية يومنا هذا فهى جميعا حلول ترقيعية.
بكل بساطة هناك مبرهنات هندسية-طوبولوجية-تناظرية فى غاية التعقيد الرياضى ضد تطبيق خوارزمية اولر على معادلة ديراك (اشهرها مبرهنة نيلسون-نينوميا Nielsen-Ninomiya theorem) .
بل ان هذه المسألة هى واحدة من اعقد المسائل فى نظرية الحقول الكمومية و السبب الاساسى يرجع الى كون دالة الموجة ψ التى تظهر فى الصورة الثانية هى ليست دالة عادية بل هى سبينور spinor.
والسبينور spinor هو مقطع section فى فضاء يسمى الحزمة السبينورية spinor bundle الذى يعيش على الفضاء-زمن.
معادلة ديراك تنص اذن على ان هناك مؤثر يسمى مؤثر ديراك Dirac operator عندما يضرب دالة الموجة ψ فانه يعطينا بالضبط صفر.
هذا المؤثر (مع فضاء السبينورات مع جبرية الدوال العددية على الفضاء-زمن) كما بين الرياضى كوون Conne كافى جدا من اجل اعادة بناء جميع الهندسة التفاضلية للفضاء-زمن.
بل ان كوون بين ان الثلاثى (مؤثر ديراك مع فضاء هيلبرت للسبينورات مع جبرية دوال ملائمة) كافى جدا من اجل اعادة بناء الهندسة التفاضلية لاى متشعب manifold.
معادلة ديراك تُقرأ على انها دالة موجة كمومية فى الميكانيك الكمومى النسبى لكن فى نظرية الحقول الكمومية فان معادلة ديراك تقرأ على انها معادلة حقل سبينورى spinor field كلاسيكى هو بالضبط معطى بدالة الموجة ψ.
ولهذا فان نظرية الحقول الكمومية تسمى التكميم الثانى second quantization اما الميكانيك الكمومى النسبى فيسمى التكميم الاول first quantization.
السبينور هو حقل يتميز بسبين او عزم لف يساوى نصف لان معادلة ديراك كما ذكرنا آنفا تخص الجسيمات ذات السبين او عزم اللف يساوى نصف.
وتذكروا فان تقلبات (مفرد تقلب fluctuation) الحقل field هى بالضبط هذه الجسيمات particles التى نراها نقطية point.
لكن سبينور ديراك الذى يتميز باربعة مركبات يوفر تمثيلة قابلة للاختزال reducible representation لزمرة لورنتز Lorentz group.
بالفعل فان سبينور ديراك يمكن تفكيكه الى زوج من سبينورات وايل Weyl spinors كل واحد منها يتشكل من مركبتين: سبينور وايل يدوانى-يمينى right-handed و سبينور وايل يدوانى-يسارى left-handed وهذه هى التى توفر تمثيلات غير-قابلة للاختزال irreducible representation لزمرة لورنتز.
اذن ديراك هو قابل للاختزال لانه يقبل الاختزال الى سبينورى وايل واحد يدور الى اليمين و الاخر يدور الى اليسار وهذه الاخيرة غير-قابلة للاختزال اكثر.
هذه نقطة تحتاج الى شرح اكبر نتركه لفرصة اخرى ان شاء الله.
لكن أهم خاصية يتميز بها السبينور هى كونه حقل تبادلى-ضدى anti-commuting عكس الحقل السلمى مثلا الذى هو حقل تبادلى commuting وهذا ضرورى حتى نحافظ على شرط ان الطاقة يجب ان تكون محدودة من الاسفل bounded from below (اى حتى يكون هناك طاقة اساسية ground state للجملة).
بعبارة اخرى فان السبينور ψ لا يمكن التعبير عنه فى تكامل الطريق path integral لفايمان Feynman بدلالة الاعداد المركبة complex numbers بل يجب ان يتم التعبير عنه بدلالة اعداد تسمى الاعداد الغراسمانية Grassmannian numbers (نسبة الى الرياضى غراسمان Grassmann) وهى اعداد غير-تبديلية anti-commuting numbers.
هذا يرجع الى خاصيتين كموميتين اساسيتين.
-احصاء فرمى-ديراك Fermi-Dirac statistics الذى ينص على دالة موجة فرميونين (مفرد فرميون fermion) مثلا الكترونين يجب ان تكون تناظرية-ضدية anti-symmetric اى انه لو قمنا بمبادلة الفرميونين فاننا نحصل على اشارة ناقص. اصل هذه الاشارة هو طوبولوجى (طوبولوجيا زمرة الدورانات rotation group). وهذا مكافئ الى مبدأ الاستبعاد لباولى Pauli's exclusion principle الذى ينص على ان الحالة الكمومية الواحدة لا يمكن ان يحتلها الكترونين.
-مبرهنة السبين-و-الاحصاء spin-statistics theorem التى تنص على ان دالة موجة الفرميون تكتسب اشارة ناقص عندما نقوم بتدوير الفرميون دورة كاملة ب 360 درجة. اصل هذه الاشارة هو تناظرى (هندسة زمرة الدورانات). ولهذا فان التكميم القانونى canonical quantization للفرميون على طريقة ديراك يتم باستعمال مبدل-ضدي anti-commutator و ليس مبدل commutator.
يمكننا قراءة عبارة مؤثر ديراك من الصورة الثانية بدلالة مصفوفات ديراك γ و بدلالة المشتقات من الرتبة الاولى فى الفضاء-زمن.
اذن مؤثر ديراك يتعلق على المشتقات من الرتبة الاولى التى يمكن ان نحاول تعويضها بخوارزمية اولر لكن لو فعلنا -ذلك بعفوية- لوقعنا فى مشاكل عظيمة فى نظرية الحقول الكمومية على الشبكة lattice quantum field theory.
اولر نجح بسهولة شديدة مع شرودينغر لكنه سوف ينجح نسبيا و بصعوبة شديدة مع ديراك.
الآن اذا اردنا تعريف مؤثر ديراك على الشبكة الاقليدية Euclidean lattice فاننا نقع فى ثلاثة مشاكل عظيمة جدا فى نظرية الحقول على الشبكة:
-اولا معضلة المضاعفة الفرميونية fermion doubling problem. و هذه المعضلة تنص على انه لا يمكن و ضع النموذج القياسى للجسيمات الاولية (او اى نظرية كايرالية chiral theory) على الشبكة الاقليدية.
-ثانيا انكسار التناظر-الممتاز supersymmetry breaking. وهذه المعضلة تنص على انه لا يمكن وضع نظرية الوتر الممتاز على الشبكة الاقليدية.
-ثالثا معضلة الاشارة sign problem. وهذه المعضلة تنص على انه لا يمكننا استخدام الميكانيك الاحصائى و طرق المونتى كارلو.
والمعضلات الثلاثة لها حلول بل حلول كثيرة لكن جميع هذه الحلول هى حلول ترقيعية و ليست حلول جذرية اساسية نهائية.
اذن هذا مجال عظيم للدراسة.
نترك شرح هذه المعضلات وحلولها الى فرصة اخرى ان شاء الله.




محاضرات دايفيد طونغ

من أقوى العارضين للفيزياء النظرية فى هذا العصر بأسلوب دقيق رياضيا و عميق فيزيائيا مع سهولة لغة و فصاحة اسلوب هو دافيد طونغ David Tong استاذ الفيزياء النظرية فى جامعة كمبريدج.

وهو رجل مازال شابا و مع هذا فهو قد كتب محاضرات موسوعية فى جميع مواضيع الفيزياء النظرية ابتداءا من الميكانيك الكلاسيكى و الميكانيك الكمومى و الميكانيك الاحصائى الى نظرية النسبية العامة و نظرية الحقول الكمومية و نظرية الاوتار الممتازة الى نظرية الحقول المعيارية و نظرية الحقول الكونفورمالية و نظرية الحقول الممتازة الى الكوسمولوجيا و ميكانيك الموائع و فيزياء المادة المكثفة.
كمثال فاننى وجدت محاضراته فى تأثير هال الكمومى من اروع ما يكون ففيها كم هائل من نظرية الحقول الطوبولوجية.
وكمثال آخر فاننى وجدت عرضه للتناظر الممتاز و للتناظر الكونفورمالى من ابسط العروض وهما من اعقد التناظرات فيزيائيا و اغمضها نظريا واصعبها حسابيا بالنسبة للاستاذ و الطالب فى الفيزياء النظرية.
صفحته هى فعلا موسوعة دقيقة جدا من الناحيتين الرياضية و الفيزيائية و اكثر من هذا هى بسيطة جدا لغويا وتعرض الفيزياء النظرية باسلوب سهل ميسر.
هذا رجل عندما اقرأ له فاننى اتذكر قراءاتى القديمة لفايمان الذى كان الفيزيائى الافصح فى تاريخ الفيزياء الحديثة.
وهو ايضا رجل يجعل قراءة الفيزياء و كأنك تقرأ كتاب تاريخ او أدب او بالاحرى و كأنك تقرأ صحيفة ترفيهية لا تحتاج ان تفعل الا ان تقرأ لان الفهم الأولى معه سريع جدا لا تضطر معه الى اجراء المعادلات و الحسابات و البراهين.
انصح جدا بقراءة محاضراته و متابعته على الاوراق قراءة و كتابة و ليس فقط على اليوتوب. فالفيزياء النظرية هى فى الاخير علم يعتمد على القراءة-و-الكتابة-و-التخيل-و-التجربة الذاتية و ليس علم بصرى-سماعى-شفاهى-انتظارى.
لكن حتى تتمكنوا منه فعليكم التمكن من الفيزياء الاساسية و من الرياضيات الأولية و من اللغة الانجليزية وهذه الاخيرة لن تتأتى الا عن طريق نفض الايدى من اللغات الاجنبية الاخرى و على رأسها الفرنسية فهى من اكبر المضيعات للجهد و الوقت.

مبرهنات التناظر الممتاز

نقدم هنا مبرهنتين هائلتين (و حدسية مازالت تحتاج الى البرهان) تتحكمان فى جميع تناظرات نظرية الحقول الكمومية الى غاية التناظر-الممتاز.

اولا مبرهنة كولمان-ماندولا Coleman-Mandula theorem تنص على ان مصفوفة التصادم scattering matrix او ال S-matrix اى المصفوفة-S (التى تتحكم فى جميع تفاعلات القوى الكونية غير-الثقالية) تتميز بثلاثة انواع و ثلاثة انواع فقط من التناظرات:
-صمود بوانكريه Poincare invariance تحت تأثير تحويلات لورنتز Lorentz transformation و الدورانات rotations و الانسحابات translations. هذه التحويلات تؤثر مباشرة فى الفضاء-زمن.
-التناظرات المتقطعة discrete symmetries و هى ثلاثة و هى المركزنة parity وهى القلب فى الفضاء و يرمز له ب P و الانعكاس فى الزمن time reversal و يرمز له ب T و تصريف الشحنة charge conjugation و هى عكس قيمة الشحنة و يرمز له ب C. هذه تؤثر ايضا فى الفضاء-زمن رغم ان تصريف الشحنة غير واضح لماذا هو كذلك.
-التناظرات الداخلية internal symmetries و هى جميع التناظرات التى تؤثر فى فضاء التشكيلة configuration space كلاسيكيا او فضاء هيلبرت Hilbert space كموميا.
من بين هذه التناظرات نذكر مثلا التحويلات المعيارية gauge transformations المؤسسة للقوى غير-الثقالية الثلاثة التى نجح الفيزيائيون فى تكميمها.
اذن مبرهنة كولمان-ماندولا تجعل نظرية الحقول الكمومية ادق اكثر مما هى عليه من دقة اصلا.
جميع التناظرات الداخلة فى مبرهنة كولمان-ماندولا لا تنطوى جبريتها algebra الا على مبدلات (مفرد مبدل commutator) تخضع لها المولدات (مفرد مولد generator) التى تولد التناظرات اعلاه.
ثانيا: تأتى بعد ذلك مبرهنة هاغ-لوبيوسزانسكى-صوهنيوس Haag-Lopuszanski-Sohnius theorem التى تنص على ان التناظر-الممتاز super-symmetry هو التعميم الوحيد لتناظر بوانكريه (اى لمبرهنة كولمان-ماندولا) الذى تحتوى جبريته على مبدلات commutators و مبدلات-ضدية anti-commutators اى على مولدات متبادلة commuting generators و على مولدات متبادلة-ضديا anti-commuting generators.
التناظر-الممتاز يعيش فى الفضاء-الممتاز super-space مثلما ان تناظر بوانكريه يعيش فى الفضاء-زمن.
المولدات المتبادلة هى جسيمات بوزونية bosons و المولدات المتبادلة-ضديا هى جسيمات فرميونية fermions وهذه البوزونات و هذه الفرميونات تشكل معا الجسيمات-الممتازة super-particles.
التناظر-الممتاز هو فى الحقيقة من اروع ابداعات الفيزياء النظرية فى السبيعنات و هو أصل مؤسس متين جدا لنظرية الاوتار الممتازة super-string theory الا انه تجريبيا لم يُكتشف بعد و هذا قد يشكل خيبة امل عند البعض.
لكن من مارس نظرية الحقول الكمومية الممتازة supersymmetric quantum field theory عن قرب فانه يعرف جيدا انه يجب ان يكون التناظر-الممتاز موجودا فى الطبيعة لا محالة فهذه الدقة الرياضية و هذه العظمة الفيزيائية لا يمكن ان تكون صدفة وسيُكتشف اذن التناظر-الممتاز عاجلا ان لم ي
كن آجلا.
التناظر-الممتاز هو من اصعب التناظرات كسرا قاطبة.
ونقصد بالكسر الانكسار التلقائى spontaneous breaking بالديناميك و الفيزياء و ليس الكسر اليدوى فذلك شيء لا يمارسه اى احد.
ونحن نريد للتناظر-الممتاز ان ينكسر لاننا لا نراه مباشرة فى الطبيعة اذن هو مكسور مثلما اننا لا نرى فى الطبيعة مباشرة التوحيد بين القوة الكهرومغناطيسية و القوة النووية الذوقية الضعيفة الذى نعرف يقينا انه مكسور بجسيم الهيغز.
اذن كيف ينكسر التناظر-الممتاز تلقائيا. هل من مجيب.
الآن الذى ينتشر فى الفضاء-الممتاز هو الحقل-الممتاز super-field وهو الحقل المرفق بالجسيم-الممتاز مثلما ان الذى ينتشر فى الفضاء-زمن هو الحقل العادى المرفق بالجسيم العادى.
التناظر-الممتاز و الحقل-الممتاز و الجسيم الممتاز يعيشون فى الفضاء-الممتاز و الجميع ينطوي على ما يسمى الاعداد الغراسمانية Grassmannian numbers وهى حقول و احداثيات فرميونية fermionic اى ذات سبين spin او عزم لف يساوى عدد نصف-صحيح half-integer (اى عدد طبيعى تقسيم 2). وهذا هو بالضبط معنى (ممتاز) فى الاسم.
وهذا مثلما ان تناظر بوانكريه و الفضاء-زمن و الحقل العادى و الجسيم العادى ينطوى جميعها على الاعداد المركبة complex numbers و هى حقول و احداثيات بوزونية bosonic اى ذات سبين او عزم لف صحيح integer (اى عدد طبيعى).
اذن فهم التناظر-الممتاز يحتاج الى تحكم تام بالتمثيلات السبينورية spinor representations (اى الجسيمات ذات السبين الذى يساوى عدد نصف صحيح) لزمرة بوانكريه Poincare group.
أهم شيء هنا هو فهم سبينور ديراك Dirac spinor و سبينور وايل Weyl spinor و سبينور ماجورانا Majorana spinor فهما كاملا لان كل الحساب (وهو معقد جدا و يسمى الحساب السبينورى spinor calculus) يعتمد على هذه السبينورات و على خواص مصفوفات ديراك Dirac matrices فى مختلف الابعاد (بالفعل لا يمكن الاكتفاء ببعد اربعة لان الفهم سيكون ناقصا. السبينورات و مصفوفات ديراك تتصرف بشكل مختلف بشكل طفيف لكنه خفى و مهم فى الابعاد المختلفة).
الآن نصل الى آخر نتيجة.
كما انه يصعب جدا كسر التناظر-الممتاز فان يصعب جدا تعميم التناظر-الممتاز.
التعميم الاول هو طبيعى جدا وهو تعميم التناظر-الممتاز الى التناظر-الممتاز الكونفورمالى conformal super-symmetry مثلما اننا نعمم زمرة بوانكرية الى الزمرة الكونفورمالية conformal group.
التعميم الثانى هو غير هين بالمرة لانه يدخل فى خانة التشوية الذى يحفظ التناظر-الممتاز.
اذن هذا ليس تمديد extension بل هو تشويه deformation وهذا اصعب ومع هذا فاننا نحافظ على التناظر-الممتاز.
هذا التشوية يعطى بالضبط بالهندسة غير-التبديلية non-commutative geometry و نحصل بذلك على التناظر-الممتاز غير-التبديلى non-commutative super-symmetry.
هذه النتجية الاخيرة ليست مبرهنة لكنها حدسية conjecture فهل من مبرهن.

هندسة الفيزياء

https://drive.google.com/file/d/1GNQdNC55MJ89d6oc1jpOLSXd-vPai8P1/view?usp=sharing 

هذا كتاب بعنوان (هندسة الفيزياء The Geometry of Physics) وهو من تأليف (ثيودور فرانكل Theodore Frankel) وهو يعنى بجميع مواضيع الهندسة Geometry و الطوبولوجيا Topology و الزمر Groups التى تظهر فى النظريتين الاساسيتين: نظرية الحقول الكمومية Quantum Field Theory (التى تصف جميع الحقول غير-الثقالية فى النظريتين الكلاسيكية و الكمومية) و نظرية النسبية العامة Theory of General Relativity (التى تصف الحقول الثقالية فى النظرية الكلاسيكية).

تذكروا فان (نظرية كل شيء Theory of Everything) مثلا نظرية الاوتار الممتازة Super-String Theory فانها تهدف الى وصف جميع الحقول الثقالية و غير-الثقالية فى النظريتين الكلاسيكية و الكمومية.
هذه النظرية (نظرية كل شيء) سوف تحتاج الى كل هذه الهندسة بدون ادنى شك بالاضافة الى هندسة اخرى قد تكون معروفة او غير معروفة.
و تجربة نظرية الاوتار الممتازة فى ال 30 سنة الاخيرة قد اثبتت هذا الامر. حيث وقعت اكتشافات كثيرة فى الرياضيات انطلاقا من الفيزياء و بعض الفيزيائيين النظريين تحصل على ميدالية فيلدز Fields Medal فى الرياضيات مثلا ادوارد ويتن Edward Witten.
نحن نتوقع ايضا ان تصف (نظرية كل شيء) حقول الوعى-و-العقل (والا فهى ليست نظرية كل شيء) بل هى يجب ان تصف هذه الحقول النفسية فى النظريتين الكلاسيكية و الكمومية معا. هذه النظرية سوف تحسم السؤال هل الوعى مادى او مثالى او ثنائى (وهذا الرأى الاخير هو رأيى). هذا الكتاب (هندسة الفيزياء) الذى يمكنكم تحميله من الرابط ينقسم الى ثلاثة اقسام كبرى هى كما يلى. فى القسم الاول يتناول بالدراسة المتشعبات و حساب التكامل و التفاضل على المتشعبات و هندسة الميكانيك الكلاسيكى. فى القسم الثانى ينتقل الى مواضيع اكثر عمقا فى الهندسة و الطوبولوجيا بالخصوص فانه سيتناول بالتفصيل كل من الاشتقاق الكوفاريانتى و الانحناء و الهدف هو الوصول الى النسبية العامة. اما فى القسم الثالث فهو سيتناول بالدراسة هندسة التكميم و هندسة الزمر و الهدف هو الوصول الى نوع من المتشعبات يسمى الحزم حيث ان الحزم الليفية تصف الحقول المعيارية (معادلة يانغ-ميلز) اما الحزم السبينورية فتصف الحقول المادية (معادلة ديراك). القسم الاول حول المتشعبات Manifolds و التنسورات Tensors و الاشكال الخارجية Exterior Forms. -المتشعبات Manifolds و الحقول الشعاعية Vector Fields. -التنسورات Tensors و الاشكال الخارجية Exterior Forms. -مكاملة الاشكال التفاضلية Integration of Differential Forms. -اشتقاق ليه Lie Derivatives. -لازمة بوانكريه Poincare Lemma و الكمونات Potentials. -القيود Constraints الهولونومية Holonomic و غير-الهولونومية Non-Holonomic. القسم الثانى حول الهندسة Geometry و الطوبولوجيا Topology. -الفضاء الاقليدى Euclidean Space و فضاء-زمن مينكوفسكى Minkowski Spacetime. -هندسة السطوح Geometry of Surfaces فى الفضاء الاقليدى. -الاشتقاق الكوفاريانتى Covariant Differentiation و الانحناء Curvature. -الجيوديزيات Geodesics. -النسبية Relativity, التنسورات Tensors و الانحناء Curvature. -الانحناء و الطوبولوجيا Curvature and Topology: مبرهنة سينج Synge's Theorem. -اعداد باتى Betti Numbers و مبرهنة دى راهم De Rahm's Theorem. -الاشكال التوافقية Harmonic Forms. القسم الثالث زمر ليه Lie Groups و الحزم Bundles و اشكال شارن Chern Forms. -زمر ليه Lie Groups. -الحزم الشعاعية Vector Bundles فى الهندسة و الفيزياء. -الحزم الليفية Fiber Bundles و تكميم غوس-بونات Gauss-Bonnet Quantization و التكميم الطوبولوجى Topological Quantization. -الرابطيات Connections و الحزم المرفقة Associated Bundles. -معادلة ديراك Dirac Equation. -حقول يانغ-ميلز Yang-Mills Fields. -اعداد باتى و فضاءات التغطية Betti Numbers and Covering Spaces. -اشكال شارن Chern Forms و زمر الهوموطوبيا Homotopy Groups.


طريقة مونتى كارلو تصيب-أو-تخيب

 تصور اننا نريد حساب تكامل دالة f(x) فى مجال [A,B]. مثلا نأخذ الدالة المعطاة بالخط الازرق فى الصورة.

الخطوات بسيطة و تسمى خوازرمية تصيب-أو-تخيب hit-and-miss algorithm.
هذه الخوارزمية اكتشفها جون فون نيومان John von Neumann آخر عظماء الرياضيات و الأب الرياضى للميكانيك الكمومى (بالخصوص المنطق الكمومى) و أب الوعى الكمومى (فهو أول من تكلم عن دور الوعى فى الميكانيك الكمومى) و هو ايضا احد الآباء الاوائل لعلوم الحاسوب.
اذن هذه خوارزمية اكتشفها فون نيومان بالاشتراك مع ستانسيلو اولام Stanislaw Ulam عندما كانا يعملان فى اطار مشروع مانهاتن Manhattan project للقنبلة الذرية-ثم-الهيدروجينية تحت رئاسة الفيزيائى الكمومى-النووى روبرت اوبنهايمر Robert Oppenheimer احد اباء الميكانيك الكمومى فى ناحية تطبيقاته التاريخية الهائلة فى الفيزياء الذرية و الفيزياء النووية.
الفيزياء و الفيزياء النظرية و الميكانيك الكمومى لا يمكن ان يتنفس الجميع بدون الرياضيات و الرياضيات لا يمكن ان تتنفس بدون الحساب و اهم مسألة حسابية هو حساب التكاملات.
اذن نحن عموما نريد ان نحسب تكامل دالة (ليس بالضرورة فى بعد واحد) و عموما فانه ليس امامنا اى مجال لاجراءها بشكل تحليلى.
اذن نستنجد بالتحليل العددى و بالفيزياء الحاسوبية و بالخوزرمة و بالتشفير بهذا الترتيب. من لا يريد ان يفهم يمكنه استخدام التطبيقات المتوفرة و هى كثيرة جدا. و من يريد ان يذهب بعيدا -فى الفيزياء النظرية و الرياضيات و علوم الحاسوب و الذكاء الاصطناعى- فعليه ان يفهم كيف تعمل الخوارزميات التى تعمل فى هذه التطبيقات؟
خطوات هذه الخوازمية البسيطة هى كما يلى.
-نقوم اولا بتوليد عدد عشوائى random (او بالاحرى شبه-عشوائى pseudo-random) x موزع بشكل منتظم uniformly فى مجال التكامل [A,B].
اى مولد generator للاعداء العشوائية سيعطيك عدد عشوائى r موزع بانتظام بين 0 و 1 اذن عليكم تحويله الى المجال [A,B] بالعلاقة x=(B-A)*x+A (تأكدوا من هذه العلاقة البسيطة).
-ثانيا نقوم بتوليد عدد عشوائى y موزع بانتظام فى مجال الدالة اى بين مثلا 0 و القيمة القصوية Max كما فى الصورة.
مرة اخرى نقوم بتحويل العدد العشوائى المولد فى المجال [0,1] الى المجال [0,Max] بعلاقة مماثلة للمعادلة اعلاه.
هذا العدد العشوائى سيلعب دور قيم الدالة.
اذن لدينا ازواج عشوائية مولدة فى مستطيل بطول و عرض يساويان [A,B] و [0,Max].
اذن هناك ازواج عشوائية (x,y) ستقع تحت منحنى الدالة و هناك ازواج ستقع فوق منحنى الدالة.
-الخطوة الثالثة. نقوم بحساب عدد الازتصور اننا نريد حساب تكامل دالة f(x) فى مجال [A,B]. مثلا نأخذ الدالة المعطاة بالخط الازرق فى الصورة. الخطوات بسيطة و تسمى خوازرمية تصيب-و-تخيب hit-and-miss algorithm. هذه الخوارزمية اكتشفها جون فون نيومان John von Neumann آخر عظماء الرياضيات و الأب الرياضى للميكانيك الكمومى (بالخصوص المنطق الكمومى) و أب الوعى الكمومى (فهو أول من تكلم عن دور الوعى فى الميكانيك الكمومى) و هو ايضا احد الآباء الاوائل لعلوم الحاسوب. اذن هذه خوارزمية اكتشفها فون نيومان بالاشتراك مع ستانسيلو اولام Stanislaw Ulam عندما كانا يعملان فى اطار مشروع مانهاتن Manhattan project للقنبلة الذرية-ثم-الهيدروجينية تحت رئاسة الفيزيائى الكمومى-النووى روبرت اوبنهايمر Robert Oppenheimer احد اباء الميكانيك الكمومى فى ناحية تطبيقاته التاريخية الهائلة فى الفيزياء الذرية و الفيزياء النووية. الفيزياء و الفيزياء النظرية و الميكانيك الكمومى لا يمكن ان يتنفس الجميع بدون الرياضيات و الرياضيات لا يمكن ان تتنفس بدون الحساب و اهم مسألة حسابية هو حساب التكاملات. اذن نحن عموما نريد ان نحسب تكامل دالة (ليس بالضرورة فى بعد واحد) و عموما فانه ليس امامنا اى مجال لاجراءها بشكل تحليلى. اذن نستنجد بالتحليل العددى و بالفيزياء الحاسوبية و بالخوزرمة و بالتشفير بهذا الترتيب. من لا يريد ان يفهم يمكنه استخدام التطبيقات المتوفرة و هى كثيرة جدا. و من يريد ان يذهب بعيدا -فى الفيزياء النظرية و الرياضيات و علوم الحاسوب و الذكاء الاصطناعى- فعليه ان يفهم كيف تعمل الخوارزميات التى تعمل فى هذه التطبيقات؟ خطوات هذه الخوازمية البسيطة هى كما يلى. -نقوم اولا بتوليد عدد عشوائى random (او بالاحرى شبه-عشوائى pseudo-random) x موزع بشكل منتظم uniformly فى مجال التكامل [A,B]. اى مولد generator للاعداء العشوائية سيعطيك عدد عشوائى r موزع بانتظام بين 0 و 1 اذن عليكم تحويله الى المجال [A,B] بالعلاقة x=(B-A)*x+A (تأكدوا من هذه العلاقة البسيطة). -ثانيا نقوم بتوليد عدد عشوائى y موزع بانتظام فى مجال الدالة اى بين مثلا 0 و القيمة القصوية Max كما فى الصورة. مرة اخرى نقوم بتحويل العدد العشوائى المولد فى المجال [0,1] الى المجال [0,Max] بعلاقة مماثلة للمعادلة اعلاه. هذا العدد العشوائى سيلعب دور قيم الدالة. اذن لدينا ازواج عشوائية مولدة فى مستطيل بطول و عرض يساويان [A,B] و [0,Max]. اذن هناك ازواج عشوائية (x,y) ستقع تحت منحنى الدالة و هناك ازواج ستقع فوق منحنى الدالة. -الخطوة الثالثة. نقوم بحساب عدد الازواج التى تقع تحت منحنى الدالة و ليكن هذا العدد L. -الخطوة الرابعة. التكامل يساوى L على N حيث ان N هو العدد الكلى للاعداد العشوائية المولدة. اما البرهان فهم يعتمد على البرهان على طرق (الاستعيان الأهم importance sampling) التى هى (سلسلة ماركوفية Markov chain) من امثلتها الشهيرة جدا خوازرمية ميتروبوليس Metropolis algorithm التى هى اشهر طرق مونتى كارلو Monte Carlo methods.تصور اننا نريد حساب تكامل دالة f(x) فى مجال [A,B]. مثلا نأخذ الدالة المعطاة بالخط الازرق فى الصورة. الخطوات بسيطة و تسمى خوازرمية تصيب-و-تخيب hit-and-miss algorithm. هذه الخوارزمية اكتشفها جون فون نيومان John von Neumann آخر عظماء الرياضيات و الأب الرياضى للميكانيك الكمومى (بالخصوص المنطق الكمومى) و أب الوعى الكمومى (فهو أول من تكلم عن دور الوعى فى الميكانيك الكمومى) و هو ايضا احد الآباء الاوائل لعلوم الحاسوب. اذن هذه خوارزمية اكتشفها فون نيومان بالاشتراك مع ستانسيلو اولام Stanislaw Ulam عندما كانا يعملان فى اطار مشروع مانهاتن Manhattan project للقنبلة الذرية-ثم-الهيدروجينية تحت رئاسة الفيزيائى الكمومى-النووى روبرت اوبنهايمر Robert Oppenheimer احد اباء الميكانيك الكمومى فى ناحية تطبيقاته التاريخية الهائلة فى الفيزياء الذرية و الفيزياء النووية. الفيزياء و الفيزياء النظرية و الميكانيك الكمومى لا يمكن ان يتنفس الجميع بدون الرياضيات و الرياضيات لا يمكن ان تتنفس بدون الحساب و اهم مسألة حسابية هو حساب التكاملات. اذن نحن عموما نريد ان نحسب تكامل دالة (ليس بالضرورة فى بعد واحد) و عموما فانه ليس امامنا اى مجال لاجراءها بشكل تحليلى. اذن نستنجد بالتحليل العددى و بالفيزياء الحاسوبية و بالخوزرمة و بالتشفير بهذا الترتيب. من لا يريد ان يفهم يمكنه استخدام التطبيقات المتوفرة و هى كثيرة جدا. و من يريد ان يذهب بعيدا -فى الفيزياء النظرية و الرياضيات و علوم الحاسوب و الذكاء الاصطناعى- فعليه ان يفهم كيف تعمل الخوارزميات التى تعمل فى هذه التطبيقات؟ خطوات هذه الخوازمية البسيطة هى كما يلى. -نقوم اولا بتوليد عدد عشوائى random (او بالاحرى شبه-عشوائى pseudo-random) x موزع بشكل منتظم uniformly فى مجال التكامل [A,B]. اى مولد generator للاعداء العشوائية سيعطيك عدد عشوائى r موزع بانتظام بين 0 و 1 اذن عليكم تحويله الى المجال [A,B] بالعلاقة x=(B-A)*x+A (تأكدوا من هذه العلاقة البسيطة). -ثانيا نقوم بتوليد عدد عشوائى y موزع بانتظام فى مجال الدالة اى بين مثلا 0 و القيمة القصوية Max كما فى الصورة. مرة اخرى نقوم بتحويل العدد العشوائى المولد فى المجال [0,1] الى المجال [0,Max] بعلاقة مماثلة للمعادلة اعلاه. هذا العدد العشوائى سيلعب دور قيم الدالة. اذن لدينا ازواج عشوائية مولدة فى مستطيل بطول و عرض يساويان [A,B] و [0,Max]. اذن هناك ازواج عشوائية (x,y) ستقع تحت منحنى الدالة و هناك ازواج ستقع فوق منحنى الدالة. -الخطوة الثالثة. نقوم بحساب عدد الازواج التى تقع تحت منحنى الدالة و ليكن هذا العدد L. -الخطوة الرابعة. التكامل يساوى L على N حيث ان N هو العدد الكلى للاعداد العشوائية المولدة. اما البرهان فهم يعتمد على البرهان على طرق (الاستعيان الأهم importance sampling) التى هى (سلسلة ماركوفية Markov chain) من امثلتها الشهيرة جدا خوازرمية ميتروبوليس Metropolis algorithm التى هى اشهر طرق مونتى كارلو Monte Carlo methods.واج التى تقع تحت منحنى الدالة و ليكن هذا العدد L.
-الخطوة الرابعة. التكامل يساوى L على N حيث ان N هو العدد الكلى للاعداد العشوائية المولدة.
اما البرهان فهم يعتمد على البرهان على طرق (الاستعيان الأهم importance sampling) التى هى (سلسلة ماركوفية Markov chain) من امثلتها الشهيرة جدا خوازرمية ميتروبوليس Metropolis algorithm التى هى اشهر طرق مونتى كارلو Monte Carlo methods.


جداول التكاملات

 نفس الفيزياء هى الرياضيات و نفس الرياضيات هو الحساب.

وأهم الحساب ثلاثة اشياء: التكامل و التفاضل و المصفوفات.
نواصل التركيز على التكامل.
عندما تصادف تكاملا عليك باحدى الخطوات التالية.
-اما ان تحاول ان تقوم بالتكامل و ايجاد الدالة الاصلية تحليليا. هذا قد يكون ممتع و قد يستغرق بعض الوقت و الاجتهاد.
-او تحاول ان تبحث عن التكامل فى جدوال التكامل.
الكتاب فى الرابط يحتوى على 1600 صفحة من جداول التكامل و السلاسل و المتتاليات و المجاميع و الدوال و كل شيء قد يخطر و قد لا يخطر على بالك.
اذن ابحث عن التكامل فى هذه الجداول و اذا وجدته فقد حُلت المسألة و انتهيت من التعب.
لكن اذا لم تجد التكامل فى هذه الجداول فى هذا الكتاب مثلا او غيره فانه عليك ان تذهب الى الخطوة الثالثة.
-عليك محاولة اجراء التكامل عدديا باستخدام واحدة من البرمجيات الجاهزة المشهورة مثلا ماثيماتيكا Mathematica او مايبل Maple او ماتلاب Matlab.
هذا سيتطلب منك كتابة شفرة بسيطة جدا بلغة تلك البرمجية ثم اجراء التكامل و الحصول اما على العبارة الرمزية او العبارة العددية.
لكن اذا كان المطلوب منك اجراء التكامل عدد ضخم من المرات مثلا و هذا ما يحدث فى طرق المونتى الكارلو Monte Carlo methods فانه عليك ان تذهب الى الخطوة الرابعة.
-اذن عليك خورزمة التكامل شخصيا فى احدى لغات تشفير المستوى-الاعلى High-level languages مثلا السى بلاس بلاس C++ (المسيطر اليوم فى الفيزياء و علوم الحاسوب) او السى C (الذى كان مسيطرا فيما مضى فى علوم الحاسوب) او الفورترون Fortran وهو اقدمها (الذى كان مسيطرا فيما مضى فى الفيزياء).
هذه اللغات البرمجية هى لغات مستوى-اعلى اى ان المجمع compiler وهو البرنامج الذى سوف يترجم الشفرات المكتوبة فى هذه اللغات سوف يترجمها من هذه اللغات مباشرة الى لغة الآلة machine language اى النظام الثنائى 0-و-1.
وهذا عكس الماثيماتيكا و غيرها من البرمجيات الجاهزة التى هى لغات مستوى-ادنى low-level languages اى ان المجمع سوف يترجم شفرتها الى احدى لغات المستوى-الاعلى (على الارجح السى) قبل ترجمة هذه الترجمة نفسها الى لغة الآلة.
لهذا فان استعمال هذه البرمجيات لا ينصح به بل هو لن يصلح اذا كان هناك تكرار فى حساب تكامل عدد هائل من المرات وهذا ما يحدث فى طرق المونتى كارلو.
لكن اقول ان هذا الطريق الرابع (كتابة الشفرة فى لغة مستوى-اعلى) قد يعوض تماما الحاجة الى الطريق الثالث (استخدام التطبيقات) وهو بنفس القدر من المتعة الموجود فى الطريق الاول (الحساب التحليلى).
اما الطريق الثانى (استخدام الجداول) فهو اقل الطرق متعة لكنه اسهل الطرق اذا كان التكامل معروف الى حد كبير.
اذن الرابط يحتوى على كتاب لجداول التكامل يمكن استخدامه فى الطريق الثانية. و اذا فشلت هذه الطريق فعليكم بالطريقين التاليتين.
هناك فى الحقيقة خطوة ضمنية خامسة لم اصرح بها.
قد يكون التكامل نادر جدا و صعب جدا لكنه معروف فى احدى نظريات الرياضيات التجريدية.
اذن يجب ايضا البحث عن هذه الامكانية او على الاقل السؤال عنها عند المختص لانها امكانية ممكنة جدا.
ديراك و فون نيومان

من اعظم الفيزيائيين النظريين بل من اعظم الفيزيائين على الاطلاق هو الانجليزى بول ديراك Paul Dirac المولود عام 1902.
و ديراك هو فى مصاف آباء الفيزياء نيوتن و بولتزمان و اينشتاين فهو يوضع معهم فى خانتهم.
و من اعظم الرياضيين فى التاريخ هو المجرى-الامريكى جون فون نيومان John von Neumann المولود عام 1903.
وفون نيومان هو فى مصاف آباء الرياضيات غوس و ريمان و هيلبرت فهو يوضع معهم فى خانتهم.
وديراك هو رياضى ايضا بأتم معنى الكلمة.
وفون نيومان هو فيزيائى ايضا بأتم معنى الكلمة.
واول كتابين بخصوص اعظم علوم الفيزياء (الميكانيك الكمومى) هما:
- كتاب ديراك المعنون (مبادئ الميكانيك الكمومى principles of quantum mechanics) الصادر عام 1930.
-و كتاب فون نيومان المعنون (الاسس الرياضية للميكانيك الكمومى mathematical foundations of quantum mechanics) الصادر عام 1932.
اكتشافات بول ديراك فى الميكانيك الكمومى و فى الفيزياء الكمومية و فى الرياضيات لا تعد و لا تحصى و نحن لا نبالغ.
اما اكتشافات فون نيومان فى الفيزياء و الرياضيات و الحاسوبية فهى الاخرى لا تعد و لا تحصى و نحن لا نبالغ.
اذن حتى لو حاولنا احصاء و جرد انجازات الرجلين فاننا لا و لن نستطيع و الامر يحتاج فى الحقيقة الى مختص بليغ متمكن عبقرى فى تاريخ الفيزياء و الرياضيات.
لكن فى هذا الكتاب بالخصوص فان ديراك كان مهتما بالفيزياء و مبادئها و قد ادى اهتمامه هذا الى اكتشافه لشيئين عظيمين فى الرياضيات (بعض النظر عن الفيزياء):
- اولا نظرية التوزيع theory of distribution و مثال على التوزيعات هى دالة دلتا ديراك Dirac delta function المشهورة التى كان يضحك عليها فون نيومان.
-ثانيا لكن اهم من هذا هو مناقشة ديراك فى فصل بأكمله لما يسميه (الشرط الكمومى quantum condition) اى ماهى القواعد العامة من اجل الانتقال من جملة كلاسكيية موصوفة بالميكانيك الكلاسيكى الى الجملة الكمومية المقابلة.
هذا موضوع شائك جدا و هو يتعلق بطبيعة الهندسة التى تؤسس لفضاء هيلبرت Hilbert space و ماهى علاقتها بالهندسة السمبليكتية symplectic geometry المؤسسة للميكانيك الكلاسيكى و كذا طبيعة النهاية الكلاسيكية classical limit اى كيف نرجع من الميكانيك الكمومى الى الميكانيك الكلاسيكى و متى ننجح فى ذلك.
هذا موضوع عميق جدا جدا يمس اسس الميكانيك الكمومى و اسس نظرية الحقول الكمومية و اسس نظرية الثقالة الكمومية (التى مازالت مجهولة) و هو موضوع مازال لم يحسم بعد (ابحثوا فى الامر فهو امر ممتع جدا جدا فى الفيزياء النظرية).
من الجهة الاخرى فاننا لا نتوقع ان يكون فون نيومان اقل عبقرية من ديراك فهو قدم فى كتابه و بشكل دقيق فيزيائيا قبل رياضيا اربعة اشياء عظيمة اخرى هى:
-اولا نظرية الرصد و القياس فى الميكانيك الكمومى التى تسمى نظرية القياس الاسقاطى theory of projective measurement حيث ميز فون نيومان تماما بين عملية القياس او الرصد من جهة و عملية التطور الاحادى unitary evolution فى الزمن لدالة الموجة من جهة اخرى.
-ثانيا اكثر من هذا فان فون نيومان حدد بالضبط المحتوى المنطقى للميكانيك الكمومى وهو المنطق الكمومى quantum logic الذى يعتمد على جبريات المسقطات algebra of projectors فى فضاء هيلبرت مثلما ان المنطق الكلاسيكى يعتمد على جبريات بول Boole algebras للمجموعات.
-ثالثا ومن اكتشافات فون نيومان ايضا انطروبى المعلومات information entropy و يسمى ايضا انطروبى فون نيومان و الذى يقابل الانطروبى الحرارى thermal entropy او انطروبى بولتزمان و هذا فى اطار اكتشافه لنظرية ضخمة جدا فى الميكانيك الكمومى تسمى نظرية مصفوفة الكثافة theory of density matrix.
-رابعا اكثر من هذا فان فون نيومان قدم اول برهان ضد نظريات المتغيرات المخفية hidden variables theory التى جاء بها اينشتاين كتفسير للميكانيك الكمومى لكن لم يصدقه احد (اى ان فون نيومان على جلال قدره لم يصدقه احد اما اينشتاين فقد صدقه الجميع) و بعضهم تجرأ و خطأ فون نيومان حتى جاء جون بال John Bell بعده ب 30 سنة و قدم البرهان النهائى ضد المتغيرات المخفية (اى ضد واقعية الواقع) المعروف باسم مبرهنة بال او متراجحة بال.
اذن فون نيومان رأى بعمق رياضى خطأ اينشتاين مثلما ان بوهر رأى بعمق فيزيائى خطأ اينتشاين فى النظريات الواقعية للمتغيرات المخفية و انتظر الجميع حتى جاء بال و حسم الامر رياضيا و فيزيائيا ثم تجريبيا.
اذن فون نيومان قدم فهم اساسى غير مسبوق للميكانيك الكمومى بنظرية القياس و المنطق الكمومى و براهين اللاواقعية (عدم وجود المتغيرات المخفية) و عليه فهو اول من وضع اللبنات الاولى فى فلسفة الميكانيك الكمومى.
اما ديراك فكما انه من الآباء المؤسسين للميكانيك الكمومى فهو من الآباء المؤسسين للميكانيك الكمومى النسبى و من الآباء المؤسسين لنظرية الحقول الكمومية و من الآباء المؤسسين للميكانيك الكمومى الاحصائى.
ديراك بدون اى مبالغة (مع هايزنبرغ و باولى و شرودينغر و بورن و آباهم و اب الميكانيك الكمومى بوهر) قاموا بكل شيء تقريبا فى الميكانيك الكمومى.
اذن اذا كان ديراك قدم التأصيل الفيزيائى و الفهم الهندسى للميكانيك الكمومى فان فون نويمان قدم التأصيل الميتافيزيقى و الفهم المنطقى للميكانيك الكمومى.
واذا كان فون نيومان قد يضحك على رياضيات ديراك فان ديراك اكتشف فى الرياضيات ما يلى:
-اولا التحليل الدالى functional analysis (نظرية التوزيع distribution theory التى ذكرناها اعلاه و نظرية المقياس measure theory او على الاقل اكتشف دورهما فى الفيزياء النظرية).
-ثانيا جبريات المؤثرات operator algebras و فضاء هيلبرت Hilbert space الذى ذكرناه اعلاه.
-ثالثا جبريات كليفورد Clifford algebras (المعروفة عندنا فى الفيزياء النظرية بجبريات ديراك).
-رابعا الحزم السبينورية spinor bundles (المعروفة عندنا بمعادلة ديراك) و الحزم الرئيسية principal bundles (المعروفة عندنا بالحقل الكهرومغناطيسى).
-خامسا الطوبولوجيا (بأول مثال فيزيائى هو المونوبول المغناطيسى magnetic monopole او احادى القطب المغناطيسى)
-سادسا هندسة و تكميم الجمل المقيدة constrained systems (المعروفة عندنا بتكميم النظريات المعيارية gauge theories و النسبية العامة general relativity التى هى نظريات مقيدة ولهذا فهى صعبة جدا و بالخصوص فان النسبية العامة هى نظرية مقيدة مع انعدام الهاميلتونية Hamiltonian وهذا ما يجعلها نظرية كلاسيكية فريدة من نوعها و تكميمها quantization من اصعب ما يكون).
اما فون نيومان كرياضى فان نظرته فى دور فضاء هيلبرت كانت هى الاساس الذى اعتمده الجميع فيما بعد و هو ما يؤدى بشكل طبيعى الى المنطق الكمومى للفضاءات الجزئية لفضاء هيلبرت المعروف باسم منطق الشبكات الأرطو-معيارية orthomodular lattices.
و كخلاصة نذكر ان الميكانيك الكمومى كنظرية فيزيائية و كنظرية رياضية و كنظرية ميتافيزيقية يعتمد على مجموعة من البديهيات تسمى اليوم بديهيات ديراك-فون-نيومان Dirac-von-Neumann axioms.
نختم بكلمة حول الايمان و الموت عند الرجلين.
كلا الرجلان كان اغنوسيا قريب الى عدم الايمان.
لكن ديراك عمر طويلا اما فون نيومان فقد واجه الموت رجلا شابا وهذا ما جعل موقفها من الايمان يكون مختلفا بشكل كبير.
ديراك كان لا يؤمن بوجود الله لكن لا يستبعد وجود الله و كدليل مقبول على هذه الامكانية يقول انه يجب فقط ان يأتى احد و يبين ان انبعاث الحياة فى الكون البدائى بشكل تلقائى هى عملية مستحيلة او باحتمال ضئيل جدا.
اذا كان بالفعل هذا الانبعاث التلقائى للحياة مستحيلا فان ديراك سوف يُقر بأن الله يجب ان يكون موجودا.
اما عموما فان ديراك كان ضد فكرة الدين و بالخصوص ضد التسييس المشاهد و المعروف للتدين و لفكرة الدين نفسها وفى العموم هو كان غير مؤمن يميل الى الاغنوسية فى آخر حياته.
حتى ان باولى وهو كاثوليكى مؤمن فى نقاش له مع هايزنبرغ و ديراك قال متهكما على ديراك (لا يوجد اله فى الكون و بول ديراك هو رسوله).
بنفس الطريقة فان فون نيومان كان اغنوسيا اقرب الى عدم الايمان بالله لكنه أصيب وهو مازال فى الخمسينات من العمر بمرض السرطان الذى تسبب له فيه عمله بمشروع مانهاتن فى الحرب العالمية الثانية.
وقد كان مرتعبا جدا من النهاية و الموت فى الاخير.
وحتى تطمئن نفسه فان فون نيومان عاود و استرجع ايمانه الكاثوليكى (باستخدام رهان باسكال) لكن لم يكن ذلك كافيا فى تحقيق الطمأنينة حسب القس الذى كان قائما عليه و حسب اهله و كتاب سيرته.
اذن عودة فون نيومان بعد مرضه الى الايمان اعتمادا على رهان لم يكن كافيا من اجل تحقيق الطمأنينة التى كان ينشدها فى مواجهة الموت و توفى مرعوبا حسب جميع الروايات.
على ما يبدو فان ايمانه فى آخر عمره لم يكن بالعمق الكافى و تعويله على باسكال هو من باب الرهان فعلا (وهذا غير كافى بالمرة) و ليس من او عن اقتناع حقيقى بعقلانية و برهانية رهان باسكال.

Comments

Popular posts from this blog

الواقع و الزمن 2020

الواقع و الزمن 2019